Anesthesia Oral Boards

Réussis tes devoirs et examens dès maintenant avec Quizwiz!

What is a good rate of urine elimination and why

0.5ml/kg/hr. This is the minimal amount one can get rid of nitrougenous waste without having changes in electrolytes

NYHA classes 1-4

1- asymptomatic in terms of physical activity 2- comfortable at rest, physical activity leads to sx 3- comfortable at rest by ADLs lead to sx 4- CHF at rest

dose of magnesium for torsades

1-2 grams

dose for methylene blue

1-2mg/kg

Whats a normal level of CO in a person and smoker

1-3% and this comes from heme metabolism You go to hyperbaric oxygen when CO level is greater than 15 or symptoms smoker 4-9% Normal half life is 4-6 hours non rebreather will cut this to 1-2 hours hyperbaric O2 will take this down to 20 minutes

Name some treatments for myasthenia gravis

1. Steroids 2. Pyridostigmine 3. Thymectomy 4. immunosuppressants 5. Plasmapheresis 6. IVIG, which is used for myasthenic crisis

What are the advantages and disadvantages of bronchial blockers

1. They can dislodge easily. 2. Difficult to place 3. Cost more than DLT 4. They can be placed next to single lumen and not through it. 5. No tube exchange, better for long cases. 6. Better for difficult airways 7. Can be used to isolate individual lobes

After an airway fire the patient is hypotensive and tachycardic. Whats your ddx

1. Tracheal tear 2. Pneumothorax 3. Airway obstruction 4. Bronchospasm

With a myasthenia pt, what are two things you think of?

1. Try to do the case regional 2. Avoid muscle relaxants

What is your plan for post operative pain control in an OSA patient

1. Tylenol and NSAIDS 2. Surgeon local anesthesia around wound 3. PCA, relief without overdosing in a monitored setting 4. Regional

What are the different depths of burns

1st, 2nd, 3rd 3rd are not painful-pale dry and a burn of fat. Third degree will not heal on their own

When is a toddler most at risk for post op apnea

2 hours after surgery but last up to 12 hours

How many joules do you use in pediatric defib shocks

2 per and then 4 per

How long do you npo supplements like ginseng before surgery?

2 weeks

What is 2,3 dpg and what does it do

2,3 Diphosphoglyceric acid. It resides inside rbc and decreases the affinity for oxygen on hgb

whats the rhythm with two p waves and one qrs like clockwork

2-1 flutter

What is a normal wedge pressure?

2-10

list the pediatric tube sizes for premature neonate 3kg 6 months 1-2 years

2.5 ET for less than <1kg 3.0 for 1-2kg 3.5 for 2-3kg 4.0 for over 3 kg

What is a normal phosphate and magnesium

2.5-3.5mg/dl, 2-3.5

What's a normal cardiac index

2.5-4

What is the normal value of phosphate

2.5-4.5

What is a normal aortic valve area, valve gradient

2.5-6 less than 15 mmHg

How often do you give dantrolene in MH

2.5mg q5 miutes, 36 vials 20 mg vial in 60 ml of sterile water

What is the dose and choice of fluid for treatment of MH with dantrolene

2.5mg/kg every minutes until symptoms resolve mixed in sterile water

With PFTs what are value are you looking for with vital capacity and why

20ml/kg at least, to make a good cough

What is your fluid bolus to a child with a deficit

20ml/kg, then switch to D51/2NS20K once making urine. Put some D5 in there because they're liver sucks at gluconeogenesis

How do you calculate IBW

22*height^2 with height in meters

What is shelf life of a cadaveric kidney

24-48hours

How would you perform a superior laryngeal nerve block for an awake intubation? and translaryngeal nerve block

2ml of 2% lidocaine just anterior to the cornu of the hyoid bone and a trans-tracheal recurrent laryngeal nerve block. A recurrent nerve block is never performed because it can lead to vocal cord paralysis and obstruction so we do a translaryngeal block to target just the sensory fibers. Go through the cricothyroid membrane with 5 cc of 4% lidocaine with a 22 gauge needle.

How long does it take for coumadin to kick in

3 days

If a patient has had a stent or a cabg and are asymptomatic for how long are they likely good to go before becoming sick again?

3 years after a stent and 5 years after a cabg

Going on pump, how much heparin for cabg would you give

3-4 u/kg for goal ACT 300-400

What is your starting heparin dose on bypass

3-4 u/kg with a goal of greater than 300 seconds.

CBF decreases how much per each 1 mm paCO2?

3-4%

What is a normal K level

3.5-5

What is the half life of narcan?

30-60 minutes

What is the onset of duramorph?

30-60 minutes

What is the onset of bone cement implantation syndrome after placement?

30-60 secs

proteinuria 1+ is equal to what?

300mg

What is a normal PVO2

35-45mmhg

How long can you wait on a cadaveric kidney?

36-48 hours.

How long npo lithium before elective case?

36-72 hours or can lead to seizures, arrhythmias and delirium

What size DLT will you use

37 for females and 39 for males, based on height. Can even xray trachea 35 french if under 5'4 and 41 if over 6'2 39 french if trachea diameter between 15-16

What is the compression to ventilation ratio for a newborn, kid under 8 and a kid over 8

3:1, 5:1, 15:2

Whats your nebulizer settings for awake intubation?

3ml of 4% nebs for 15 minutes

Half life of CO on RA and half life of CO on nonrebreather

4 hours 1 hour

What is the dose of magnesium for eclampsia

4 mg bolus over 30 minutes then 1-2mg/hr. Reduce if kidney disease.

what is the normal value for calcium and how do you replete

4.5-5.5.

Whats the most important thing about hypothermia

400% increase in oxygen consumption with shivering

Goal of ACT for bypass? and then what do you do if you're not reaching your target

425 give more heparin give recombinant AT3 and ffp

How many weeks is a baby susceptible to for retinopathy of prematurity

44 weeks

How long do infants have a risk of post op apnea what about premature infants. What are some of the risk factors

44 wks and 60 for prematures. Anemia and premature, hypothermia there's still a risk whether you do general or regional with sedation

What is a good dose of magnesium

4g load over 5 min and then 1-2g/hr

Do you have to change your sugammandex dose in a patient with end stage renal

4mg/kg is what studies are showing

What is the parkland formula

4ml/kg/hr * BSA burned. First half over 8 hours

What is your neonate fluid rate

4ml/kg/hr and replace insensible loss with 6ml/kg/hr

What is the dose of succinylcholine IM

5 mg/kg

What size LMA would you place for a child who is 20-30 kilogram and approximately what age is this?

5 year old and a 2.5 LMA

What are normal and toxic Mg levels, list them

5-9 is therapeutic, 10-15 is BP and DTR drop, >15 respiratory arrest, and >20 is cardiac

What is considered mild hyperkalemia

5.1-6

What is the definition of a massive blood transfusion

50% blood volume in 4 hours or 100% in 24 hrs

How do you treat a carcinoid crisis

50-100mcg of octreotide q10 min up to 1mg and hydrocortisone. Aprotinin which is a kallikrien inhibitor has been used as well.

What heart rate is considered adequate beta blockade

50-60

What's a normal cerebral blood flow rate

50ml/100g of tissue/min

What is your awake intubation technique

5cc of 4% lidocaine via atomizer, if they have a limited mouth opening then use a nebulizer with 5cc of 4% for 30 minutes

How much epi is in 3ml of 1:200,000

5mcg per ml 5*3 is 15 mcg

Pediatric amiodarone dose

5mg/kg over

How long do you have to wait until you can use heparin after a catheter is placed

6

What is a normal pulse pressure variation?

6-10. When greater than 12 you start to think about volume

When is the risk of vasospasm the highest?

6-8 days post op

Risk factors for preop anxiety in a kid

7 y/o, shy, anxious parents

What is a normal fetal pH

7.25

What is a normal fetus pH

7.25

What level of HgA do you need to prevent sickling?

70%

How much PEEP is too much to extubate on, no peep? is peep of 10 ok to extubate?

8 or less is ok. there's no fine rule about this

What is a normal pulse pressure variation and what is it used for

8-12, and it indicates where one is under mechanical ventilation on the frank starling curve.

How do you calculate SVR and whats a normal value

80* (MAP-RAP)/CO = SVR, Normal is 800-1200

List the EBV for neonate, infant, child, man, woman

85 neonate, 80 infant, 80 child 1-6, child 7-10 75, man 75, woman 65.

In a PDA repair what is your target O2 sat

87-95% because of risk of retinopathy

What is the max dose of intralipid

8ml/kg

What is the EBV of a premature infant, infant, under 2, and children over 2

95ml/kg, 85ml/kg, 75ml/kg, 70ml/kg. For example an 8 year old male will be 70m/kg

Post op oliguria electrolytes, pre-renal vs intra renal, urine sodium

<10 in prerenal and >40 in intrarenal

What is a aortic valve area and gradient for severe aortic stenosis

<1cm and greater than 40mmHg

What is the urine osm in a pt with renal Osm

<400 mOsm/L

What is the bun/cr ratio in pre-renal etiology

>20 AND 10-15 for intra-renal

What is the urine sodium in a pt with intra-renal oliguria

>40 meq/L

Why do preterm neonates develop necrotizing enterocolitis

A PDA after birth due to increased pulmonary vascular resistance then becomes a left to right shunt. Then this shunts blood away from the systemic circulation towards the pulmonary circulation leading to decreased organ perfusion. This then leads to necrotizing enterocolitis

How do you distinguish between RV failure and tamponade?

A TEE probe

How does a TEG work

A TEG analyzes the viscoelastic properties of the clot Whole blood placed in an oscillating cup to imitate the sluggish venous flow and a pin is placed where clot form. The pin helps convert mechanical energy to current and this goes to a computer

What is an aortic balloon pump

A counterpulsation device that sits in the aorta net result is that there is a decrease in myocardial work and improvement in oxygen supply to myocardium.

What is achondroplasia and what are some complications with OBGYN

A disease of cartilage formation. Patients will have defomity of the spine and airway so spinal should be avoided tracheal narrowing thickening of laryngeal structures large tongue and tonsils atlanto axial instablity spinal stenosis leading to a high spinal They have kyphosis, cervical stenosis, cauda equina atlanto axial subluxation foramen magnum stenosis preferred epidural over a spinal small rib cage-> restrictive lung dz for regular cases the best way to go is awake foi for a c section an epidural technique is the safest way to go

How is the definitive diagnosis of OSA made

A formal sleep study

What is hepatopulmonary syndrome?

A hypoxic pulmonary syndrome that has symptoms of dypsnea particularly platypnea which is sob relieved with lying flat but worsened with sitting up or standing. It is a syndrome of vasodilatation*

What is the mechanism of autonomic dysreflexia

A local insult creates full body vasoconstriction and then an inhibitory signal is sent out but is unable to go below the spinal cord lesion -> vasodilatation above, increase cutaneous flow, flushing

Contraindication to a DLT left side

A mass in the left main bronchi

Whats the one monitor you need for any CRANI?

A precordial doppler...don't forget this...as its the only monitor that detects a VAE

What is the major risk factor for placenta previa

A previous uterine scar

for mediastinoscopy for a mass, what kind of ET tube should you use?

A reinforced tube

What is octreotide and how does it work

A somatostatin analog and a potent inhibitor of GH, insulin, and glucagon, vip

With preclampsia why would you prefer an epidural over a spinal?

A spinal does have a risk of herniation if there's an increase risk of ICP that has gone undetected.

Would you delay a renal transplant for a potassium of 6.0

A value of 6.0 can be lethal and is never normal.

Tell me the FDA pregnancy risk categories

A-No risk to fetus B-No risk to animals C- Some risk to animals D-risk to humans

How can you figure out cord compression from a lateral neck xray?

AADI, atlantoaxial dental interval, if greater than 4mm it's bad

Working up asthma? what lab should you get?

ABG

You have a lot of bleeding in the drains post op after a CABG, whats the first lab you order?

ACT and a TEG If ACT is 10% higher than baseline then give more protamine

Why do we use ACT or ptt

ACT is for aggressive heparin administration if you try to use ptt for large heparin it won't even registrer a number. ACT can also give rapid results that ptt cannot A normal ACT is 70-150 ACT for cardiac is greater than 400 for a carotid is greater than 250

How does plavix work?

ADP receptor antagonist

What medications can you put down an ET tube and what are some negative aspects about it

ALONE V atropine, lidocaine, narcan, epi, vasopressin negative aspects are drugs can come out with compressions, interrupts ventilation

What is your ddx for post op vision loss

AON and PON which is painless due to hypoperfusion retinal artery occlusion-painless Acute angle glaucoma is painful

Indications for ECMO

ARDS Bridge to transplant exhaustion of medical management retractable arrhythmia

What kind of monitoring devices do you need for a liver transplant

ASA, art line, central line, foley, TEE

List some acyanotic lesions

ASD, VSD, PDA, Coarct

What's a normal AST, ALT, ALKP, PT and PTT

AST 5-40 ALT 5-40 ALK 35-130

Cause of peak pressure and decrease BP

AUTOPEEP

What size ET tube can you place through an LMA 5?

Add two, so you can place an ET 7 through there

How would you know if you're having an adrenal crisis intraop

Addison's disease is an auto-immune disorder of the adrenal cortex. signs: low blood pressure, N/V, syncope, low Na, low glucose, metabolic acidosis, hyperkalemia,

Can you use an afoi in the ED or an emergent situation? Will they aspirate

As long as they keep spontaneously breathing, and you don't perform a vagus or recurrent laryngeal/transtracheal nerve block they won't aspirate

during aortic dissection repair what are some things you can do to increase distal perfusion

Ask surgeon reimplant arteries to vital organs, ask surgeon to place a shunt, induce hypothermia

How do you determine if a recurrent laryngeal nerve injury bilateral

Ask the patient to say EE, they will be aphonic if bilateral.

How would you extubate an OSA patient

Assuming no adverse intraoperative events or significant volume shifts, i would extubate with emergency airway equipment on standby, patient in the sitting position to optimize pulmonary mechanics, adequate reversal, and have the patient spontaneously breating with normal tidal volumes and respiratory rate. Once the patient is responding to commands and protecting the airway I'll extubate.

Would you extubate this patient?

Assuming no further surgical intervention in the 24-48 hours and that they have met extubation criteria

What is term for how will you induce this patient

Assuming the patient is optimized and a normal airway I will perform a smooth controlled induction using blah blah blah

How to do assess htn preoperatively for elective cases

At 180/110 i cancel the case at 160/100 I cancel if there's comorbidities Lastly i cancel for 140/90 if its a cardiac case or a carotid, or pheo surgery

At what level would you place your epidural for post op pain.

At the level of the incision

If you can't intubate a patient asleep, whats the point in intubating them awake

At times if the patient is sitting upright and if the airway maintains its muscle tonicity it may be easier to intubate this patient.

Drugs you can place in the ET

Atropine Lido Oxytocin Narcan Epi Vaso

After aortic cross clamp is released, HR drops to 40, what do you do

Attempt to treat, feel for a pulse and apply a transcutaneous pacer

How does a hx of htn affect your anesthetic technique

Autoregulation is shifted and you have to consider keeping their MAP within 20% even tho its elevated

With burn patient's what is your fluid of choice? Can you use albumin?

Avoid albumin as this can leak thru capillaries and cause even further hypovolemia. NS is a good choice

With burn patients what is your choice of fluids?

Avoid colloids as to not disrupt oncotic pressure .

How would you treat pulmonary hypertension in a patient with a congenital diaphragmatic hernia?

Avoid situations that increase pulmonary hypertension: hypoxia and hypercarbia -administer nitric oxide -HFOV which improves ventilation Keep the patient sedated and paralyzed to reduce stress induced catecholamine secretion prostanglandin E1 Last resort is ECMO

During a resection of a pheo, what medications would you avoid?

Avoid succinylcholine due to fasciculations, histamine releasing drugs like morphine and atracurium, and avoid drugs that can cause an increase in sympathetic activity such as atrophine, ketamine,and ephedrine.

How would you intubate a peritonsillar abscess

Awake nasal intubation on the contralateral side oral intubation has the risk of rupture, dislodging, aspiration

How do you diagnose ARDS

B/L pulm infiltrates, wedge less than 18, PA/fIO2 < than 200

What are some clinical applications for CPAP and what is the difference of BIPAP vs CPAP

BIPAP is used when people need more support such neuromuscular disease or CHF/copd. CPAP is typically just for OSA. Also with BIPAP is better tolerated and you can adjust ventilation and oxygenation separately

After a trauma the patient is in the ICU and hypotensive whats the cause

BLEEEEEEEDING

How to diagnose pickwickian syndrome

BMI > 30, PaCO2 > 44, no alternate explanation for hypoventilation. Pts often have co-existing OSA but they don't go together.

What are the normal pediatric vital signs for a 2 year old

BP 100/60 HR 80-150 RR 20-30

What are some things that do and don't effect SSEP

BP, temperature, acidosis, anemia, hypoxia,

What is the difference between semi open, semi closed, open, and closed systems

Based on if there's a reservoir bag and if there's rebreathing Open: no and no Semi-open: Reservoir yes and no breathing Semi-closed: Reservoir yes and partial rebreathing Closed:reservoir yes, and complete rebreathing

Would you get an echo in a sickle cell pt getting an appy?

Baseline echo is only needed if the patient is having signs of CHF, Dypsnea on exertion, or if you hear a loud murmur on exam. Otherwise no baseline echo is needed.

In a post CABG bleeding pt, why will doing a TEG is inaccurate?

Because you gave the patient a bunch of heparin so TEG will be off unless you add heparinase to the sample

For a carotid would you perform a regional or general anesthetic for this case and why

Benefit of regional awake pt for cerebral and cardiac status. No hemodynamic instability associated with induction. Disadvantages include need to convert, movement during the case. Assess pts anxiety level. May have to convert

What kind of beta blocker do you want to avoid in COPD patients?

Beta 2 blockers

What abnormalities are associated with Mg overdose

Between 5->8 meq/L is associated with increased PR interval, widening of QRS complex

What is the treatment for uterine atony

Bimanual compression and massage. oxytocin, methergine, hemabate, ligation of hypogastric arteries and hysterectomy

What precautions do you need to make with a patient who is doing ECT

Bite block parasympathetic response initially avoid etomidate as it can prolong seizure duration an adequate seizure in ECT is 30 seconds

What are two conditions where you would want to avoid epidurals for cases?

Bleeding and severe aortic stenosis.

after a TURP, the patient is in the PACU and complaining of blurry vision that progresses to blindness. What is going on

Blindness is associated with glycine use in TURP. Glycine is similar to gaba and activate inhibitory neural pathways in the eyes. The management is supportive and reassure the patient that this is transient, still get an ophth to eval

How does amicar, aprotinin and TXA work?

Blocks plasminogen activator and plasmin release

What are some causes of PONV

Blood glucose if diabetic volume status opioids that pt has been given

What are the differences between phentolamine and phenoxybenzamine

Both are non-selective. Phenoxy is irrev and used orally. Phentolamine is iv and reversible. Phenoxy is usually taken at home.

How do you decide if it is CSWS or SIADH

Both have decreased sodium, so order urine osmolytes and a 24 hour urine collection. CSWS high a high urine output and SIADH is minimal. Osm in CSWS is usually low and high in SIADH. Urine salt electrolyte is elevated in both

What is the different and what changes are there from a supra renal vs an infrarenal clamp?

Both have several changes Placing the clamp -> increased SVR-> Increased afterload and decreased CO -> MI Increased wall tension -> Increased LVEDP -> Decreased coronary perfusion There's actually an increase in preload when applied increasing the LVEDP even more so When placing the clamp watch for EKG changes, FiO2, propofol bolus. Always have an echo probe in placed when applying an aortic cross clamp*** The higher the clamp, the higher the SVR

What is kernicterus?

Brain damage in a newborn from elevated levels of jaundice.

Which one of these would you give IE prophylaxis to: VSD, HOCM, Pulling deciduous teeth, or placing orthodontic brackets, bronch with biopsy

Bronch with biopsy. Also manipulation of gingival tissue would be warranted.

What risks are there to a premature infant

Bronchopulmonary dyplasia, nec enterocolitis, apnea spells, renal/liver dysfunction, patent PDA, ret of prematurity, vacterl

Indications for DLT

Bronchopulmonary lavage, bronchopleural fistula, lung abscess, hemorrhage, communicating empyema pneumonectomy, lobectomy, thoracoabominal aneurysm, esophageal resection

What is your dose for a thoracic epidural for rib fractures

Bupivicaine 1/8th percent 8-10cc/hr

Why not use ketamine on induction with burn pts

Burn pts are somewhat insensitive to ketamine with NMDA receptor down regulation.

What are some of your concerns with a trauma

C spine injury Full stomach cardiac injury retroperitoneal bleeding

Ddx for increased mediastinal drain output post CABG

CABG Graft dysfunction which includes graft clotting, suture rupture, kink, tamponade.

What labs tests would you automatically order if you have a preclamptic patient?

CBC, BMP, liver fxn, UA, 24hr protein, coags. Don't forget the LFTS!!!

What labs would you want to order for a case with an HIV patient?

CD4 need to be above 200 for elective case Chem, cbc, coags cxr, ekg Prefer autologous transfusion, cell saver over transfusion as you wouldn't want to transfuse CMV or an opportunistic infection

PAC indications

CHF Low EF Left sided valvular heart disease CABG Aortic cross clamp COPD ARDS Complex fluid management Shock venous air embolism Burns acute renal failure high risk obstetrical care eclampsia abruption sitting crani

Name the symptoms of end organ perfusion in preclampsia

CNS, RUQ pain, HELLP, oliguria

What is indomethacin and how does it work and what are its side effects

COX inhibitor and decreases prostanglandin E1 levels it has a possible effect of IVH. So if you see IVH in a stem then indomethacin would be a contraindicaion

Youre in the pacu and the patient is not oxygenating correctly, what should you try before intubating?

CPAP

Causes of high urine output

CSWS Diabetes insipidus Osmotic diuresis from glucose/diabetes

Tip for SAH

CSWS vs SIADH, Triple H therapy, SEPS, EEG, TIVA, Cushing Response. Neuroprotection low temp, barbs, MAP, Hunt Hess classification

What is your criteria for coming off bypass

CVP Cold Conduction: avoid hypoxia, hypercarbia, medications, light anesthesia, good plane Cardiac output Cell/hgb Calcium/electrolytes Coagulation:products ready but not given until after protamine V Ventilation visualization of the chambers and heart with echo Pressors ionotropes and pacer ready potassium

What a normal value for a CVP and PA

CVP 3-8 PA 15-30/4-12

A pt had a stent placed 3 weeks ago and needs urgent surgery, what is your management of the platelet therapy?

Continue it unless bleeding risk is greater than stent thrombosis risk

CRRT there's two forms of it and what are they

Continuous venovenous hemofiltration (CVVH) and CVVHD.

What are the physical exam findings for pulmonary edema

Crackles, wheezing, dull percussion

What are some procedures where VAE is high?

Crani, cervical laminectomy, cesarean, total hip, laparoscopic procedures, and central line placement

What can using too much nitroprusside do?

Create cyanide toxicity which has symptoms of htn, tachycardia, tachyphylaxis, metabolic acidosis, htn, arrhythmias, elevated lactic acid it creates cyanomethemoglobin

Explain croup vs epiglottis

Croup 6 months to 3 y/o big tube, overinflated cuff, prolonged surgery viral hospitalization not required Epiglottis 3-6 yrs bacterial immediate intubation

Contraindications to succinylcholine

Crush injury upper motor neuron injury stroke, tumor , mass with residual deficit severe abdominal infection burns neuromuscular disorders

What are some adv/disadv of cuffed and uncuffed ETTs

Cuffed adv: less reintubations, lower OR pollution, more accurate peak pressures, much less tube exchanges. With the cuff there's risk of overinflation and tracheal stenosis and damage.

How would you tell if your preclamptic patient has elevated ICP?

Cushing response altered mental status nausea and vomiting

What is the difference between cyanotic and acyanotic heart lesions

Cyanotic blood is shunted from the right to the left and acyanotic is the other way

Ddx oozing from surgery site

DIC, bleeding, hypothermia, dilutional coagulopathy

DDx for premature labor bleeding

DIC, tear, abuse, previa, abruption, uterine rupture, AFE as they might make you ooze.

Why would you want to avoid preoperative sedatives in a TBI patient?

Decrease in RR leading to increased PaCO2 leading to increased ICP

What are some harmful effects of hypothermia

Decrease in plasma insulin, leading to hyperglycemia, decreased intravascular volume due to cold-induced diuresis, decreased platelet function, decreased drug metabolism. Can also become htn and tachycardia

Do you still need to perfect an RSI on a preggo if she hasn't ate in days, why?

Decreased GI transit time gravid uterus, narcotics

In a pregnant woman should you stop her methadone during pregnancy

I would not stop the methadone as patients on chronic methadone therapy risk preterm labor, fetal demise, and fetal distress if abruptly stopped.

How would you place the airway in a thyroid goiter pt.

If there's a history of dysphagia, dyspnea, and mass effect then awake intubation

For a TURP would you want an arterial line placed?

If there's multiple cardiac risk factors because there's going to be a massive amount of fluid absorption that can create hemodynamic instability

Would you place your IV on a 12 y/o before or after induction?

If they're comfortable with needles then before, otherwise afterward.

Why do anesthesiologists care about rib fractures?

If you add ppv then you can create a ptx

Why should you never do an epidural on a CHF patient?

If you induce a sympathectomy you will make them have venous pooling and can decrease their EF even more so.

How does a history of cirrhosis effect your anesthetic? Name 5

If you see ascites then the patient is likely intravascularly depleted, and due to the mass effect of the ascites the patient is likely to be a full stomach. Therefore an RSI and cardiac stable induction would be recommended. MAC values are usually req'd to be higher than normal because most cirrhotics have a history of chronic alcohol abuse. Lastly avoid drugs that utilize hepatic metabolism.

What kind of airway will you do on a burn pt?

If you suspect any kind of inhalational injury then you will do an awake fiberoptic. There's nothing wrong with doing a MAC and having a quick DL look as well.

Why is prone positioning a treatment in ARDS

Improved FRC Better drainage of secretions improved oxygenation

Obese difficult airway vs asthma....explain

In an asthmatic a deep extubation is a great tool. However, which road will you pick in an obese asthmatic? I'll extubate fully awake due to the difficult airway and deal with the bronchospasm if it occurs.

What is the urine:serum creatinine ratio in a pt with oliguria inta-renal vs pre-renal

In pre-renal urine:serum creatinine <20 meq/L and intra-renal >40 meq/L

What is the pathophysiology of sickle cell disease

In the beta globin protein there is a change from valine to glutamine which causes a defect

For a anterior mediastinal mass, or biopsy. Where would you put your IVs

In the legs are you can get compression or SVC and occlude your upper extremities IVs.

Pneumothorax vs PE how do you tell

Increase ETCO2 with ptx with an acute drop with PE

What conditions shift the hgb curve to the right?

Increase in 2,3 dpg temperature acidosis fetal hgb shifts the curve to the left

How does CO2 effect blood pressure.

Increase in CO2 effects blood pressure. Decrease in CO2 can drop blood pressure

How does a cross clamp effect CNS?

Increase in ICP -> decreased perfusion of spinal cord

What are some adverse effects to the aortic cross clamp

Increase pressure above the clamp Decrease pressure below the clamp MI, renal and mesenteric ischemia, spinal cord ischemia/paraplegia

What are some ways to treat an elevated ICP

Increase the head of bed Mannitol 0.25-1g/kg (Takes 20-30 minutes to act) Hyperventilate to a etCO2 to 30 CSF drain Initiate a barbituate for protections

List all the physiologic changes with applying an aortic cross clamp

Increase: CVP, ICP, SVR, PA, Afterload, Preload, Myocardial demand due to increased LVEDP, Increased wall tension

Contraindications to an epidural

Increased ICP, coagulopathy, hypovolemia, sepsis, severe aortic stenosis

What is the down side to polycythemia

Increased SVR, vaso-occlusive crisis, dysfunctional platelets

What are some cardiac considerations with a downs pt

Increased risk of cushion defects, ASD, VSD, PDA, tetralogy of fallot, phtn leading to eisenmenger

what are nitrous oxide effects on PVR and SVR

Increases PVR and maintains SVR

What is the immediate effect of mannitol?

Increases regional cerebral blood flow due to the increase of cardiac output and this results in vasoconstriction in the cerebrum where autoregulation is still in place.

During TEF repair pneumoperitoneum develops, how does this create desaturation

Increasing gastric distention could cause the stomach to rupture and place pressure on the diaphragm

What are your major concerns with a burn patient

Infection, hypovolemia, difficult airway, renal failure

What are the effects to the patient from being on bypass

Inflammatory response, neurologic injury, coagulopathies, platelet dysfunction, fibrinolysis, renal impairment, and arrhythmias

You're about to extubate but are having difficulty, how do you know if its due to your narcotics or inhalational agents using your CO2

Inhalation agents are high RR and low TV narcotics will be low RR and high TV

How do you perform superior laryngeal nerve block?

Injecting local anesthetic at the greater cornu of the hyoid bone

How would you treat preclampsia elevated ICP?

Intubate hyperventilate elevate head of bead lasix as a last resort and mannitol mannitol works as a osmotic but also decreases CSF production Also avoid flumazenil and narcan in cases like this because it will create a symp discharge and elevate ICP and also lower the seizure theshold

What are some down sides to intubating through an LMA and what's great about an AINTREE tube exchanger with fiberoptic

Intubating with just the fiberoptic and ET tube doesn't allow you to remove the LMA and the ET tube actually doesn't go much further pass the LMA The AINTREE is different from a tube exchanger because it allows a fiberoptic to be loaded within it

What are the risks and benefits to acute normovolemic hemodilution

It acute increases cardiac output due to compensation and there's an increase in stroke volume due to a decrease in SVR Which increases 2,3 dpg which decreases hgb affinity for oxygen

How do you treat a cholinergic crisis

It appears as salivation, sweating, abdominal cramps, brady, fasciculations. Treatment is supportive/intubation, atropine.

How does amiodarone work

It blocks potassium, sodium, and calcium channels and has some beta blocking activity. It is a class 3 anti-arrhythmic drug it lengthens the cardiac AP and decreases chronotropy negative ionotropic properties as well

What is one side affect to muscle that etomidate can do?

It can cause masseter muscle spasm

Why no succinylcholine with open globe?

It can cause the contents of the eye to extrude

What is wrong with using ephedrine with hyperthyroid patients?

It can have an exaggerated response with ephedrine or indirect acting agents.

What are the benefits of EEG and disadvs of it

It can not eval subcortical injuries and it can miss regional injuries and there can be a lag time

How does a chest xray help with asthma?

It can show edema, foreign body, pneumonia, pulmonary hyperinflation, bronchial wall thickening

What is systolic anterior motion of the mitral valve?

It causes LVOT and a systolic murmur

What is the mechanism that mannitol works so quickly

It causes cerebral vasoconstriction

How does hypothermia create R->L shunting in a newborn

It causes increased PVR

What is the mechanism of nitroglycerine

It converts to nitric oxide which then stimulates cgmp production -> vasodilate

How does adenosine work

It creates cell membrane hyperpolarization bt modifying potassium channels

What effect does lateral positioning have on ventilation

It creates more v/q mismatch and it decreases FRC as abdominal contents push into the thorax. Once the chest is opened ventilation occurs even more and V/Q mismatch occurs

would you use succinylcholine when intubating a neuro patient ?

It depends for a mass sure no problem upper motor neuron injury is a contraindication and this includes stroke usually one is oke if under 48 hours. This effect can last up to six months. So if a stroke victim comes in RSI with roc and have suggamandex ready If it's for an intracranial mass then succinylcholine is ok and should be used if the airway is difficult or if they have a full stomach.

After a rigid bronchoscopy for a foreign body will you intubate this patient or allow them to just wake up?

It depends, I will discuss with the surgeon the extent of edema. Then place an ETT and see if there's an adequate cuff leak at 25-30 mmHg. If no leak we will leave the patient intubated over night until the swelling has decreased.

Should a myasthenia patient take their pyridostigmine before the case?

It depends, if they have severe myasthenia gravis then yes, as you don't want any respiratory difficulties prior to surgery but if you do take it, you risk a cholinergic crises, especially after giving neostigmine for reversal. If you don't take it you may develop an anti-cholinergic response during the case. If mild-moderate then no, but beware this may lead to an anti-cholinergic reaction later

How do you use pulse pressure variation?

It determines where one is on the frank starling curve. A low PPV means you're low on the curve and the body will response to an increase in preload. A high PPV means you're on the right of the curve and fluid won't increase cardiac output. Overall increasing your preload will decrease your PPV.

During aortic cross clamping for an aneurysm, why not always place the clamp below the renal arteries to make them less ischemic?

It does not matter because there is still a very high renal artery resistance

Why would you add narcotic to an epidural?

It has synergistic effects with the local anesthestic

How does heparin work

It increases the reaction between AT3 and thrombin

What is stage 3 htn and why does it matter

It is 180/110 and this is a level that studies began to show end organ damage becomes more prevalent and stage 3 is associated with increased morbidity and mortality

Suggamandex side effects and tips

It is a cyclodextrin works on iminosteroids works by chelation eliminated in urine unchanged When you have to reintubate in the pacu it will be unpredictable for 3 hours -If you have to reintubate you have to use 1.2 mg/kg if under 3 hours. After three hours you can go back to 0.6 mg/kg Allergies: hypersensitivity, bradycardia avoid with lactation, non hormonal contraception for 7 days

What are the causes of IUGR and why should you care

It is a huge risk factor for fetal acidosis and asphyxia during delivery and prematurity causes -htn, diabetes, chf, copd, chronic hypoxia, smoking, alcohol

What is the pathology of malignant hyperthermia

It is a mutated ryr receptor that releases calcium that leads to actin and myosin cross bridging. MH is autosomal dominant

Why is it that you have to give mannitol over 10 minutes

It is a smooth muscle relaxant and can increase ICP if given too fast.

What are the advantages and disadvantages of ECMO

It is a strategy to improve oxygenation/ventilation and myocardial dysfunction VV Ecmo places a double lumen catheter into IJ VA pumped to a membrane oxygenator and then back into the common iliac Contraindications inability to anticoagulate, multi-organ failure Indications: reversible respiratory failure cardiogenic shock, bridge to a cardiac assist device

What is PPV?

It is a value that helps correlate where the patient is located on the frank starling curve. So if your PPV value is high then you are on the left side of the frank starling curve and the patient may benefit from receiving volume.

What is MUDPILES?

It is an acronym for the causes of increased anion gap metabolic acidosis Methanol Uremia Diabetic ketoacids Paracetamol or tylenol Infection Lactic acid Ethylene glycol Salicylates

What is post op delirium

It is disturbance of perception, orientation, and psychomotor behavior with a fluctuating clinical course that develops in hours to days.

What are the risks of using Kcentra

It is made from human blood so risk of infection. Contraindicated with patients with DIC and those with HIT, as it contains some heparin.

What is one major reason to not use thiopental?

It is not cardiac stable like etomidate

When considering preoperative steroids for replacement therapy, why choose hydrocortisone?

It is short acting and you don't want to develop further adrenal suppression with a dose of a long acting drug like decadron

Why don't we use LR for crani's?

It is slightly hypotonic

Why is isoflurane the best choice for a cabg?

It is the most potent vasodilator of the coronaries of the volatile anesthetics

In a trauma situation, why is factor 7 used as a last resort

It is very expensive and is used for severe bleeding

How does chronic pyridostigmine use alter extubation?

It makes it more difficult as the body may be depleted.

Why can't you give pre op oral diabetes medication

It may result in hypoglycemia.

You have an elevated MA that is unresponsive to platelets. What could this mean

It means that you have fibrinolysis and you might want to try amicar

What is the significant of LVH in the perioperative setting

It means you have a decreased CPP but also an increased myocardial oxygen demand

What is rhogam and how does it work

It prevents the formations of D-antibodies in an Rh positive mother

what is procrit or epoetin alfa?

It stimulates the body's endogenous hgb production, good for jehova's witnesses and sickle cell patients

Why is pulmonary hypertension a contraindication to liver transplant?

It typically has effects or leads to right heart decline. If you can improve the right heart function and pulmonary hypertension medically then a transplant would then be amicable. The main reason is that it decreases survivablity after transplant.

Why is isoflurane the agent of choice in neurosurgery

It uncouples cerebral blood flow and metabolic rate the best. Isoflurane decreases CRMO2 the most

Traumatic brain injury, why not use nitrous

It will lower my FiO2 and can worsen my ICP.

What's the diffference from pickwickian syndrome to OSA?

It's a central process and said to be hormonal. It has increased bicarbonate concentrations leading to apnea

What is the debakey classification and what is it used for

It's a classification used for aortic dissections. It has three types Type 1 begins in the ascending and descends to the abdominal aorta Type 2 begins and ends in the ascending aorta Type 3 begins left of the left subclavian and extends to the diaphragm Type 1 and 2 are surgical emergencies and type 3 is a medical emergency

What does mannitol transiently raise ICP if you bolus it?

It's a smooth muscle vasodilator

What is a three legged stool ?

It's an assessment of VO2max, , ppoFEV1, and DLCO It follows the 15, 30, 40 rule VO2 max is 15ml/kg/min which can be assessed by 2 flights of stairs, 6 minute walk

Would you do an exchange transfusion or a simple transfusion

It's the same

What are your thoughts on using PEEP during a crani?

Its a bad idea as PEEP can decrease venous return increase ICP.

What are the disadvantages of hetastarch

Its a colloid and it decreases factor 8 and VWF, and interferes with platelet adhesion. The only positive is that its a colloid

MEN 1, MEN2A, MEN2B

MEN1: pit adenoma, parathyroid hyperplasia, pancreatic tumors MEN2A: Parathy hyperplasia, medullary thyroid cancer, pheochromocytoma MEN2B: Medullary thyroid CA, pheochromocytoma

ddx for increased PaCO2

MH, low flow, CO2 cannister exhausted, stuck inspiratory valve, hypoventilation, sepsis

What the difference between malignant hyperthermia and thyroid storm

MH: tachy, htn, arrhythmia, temperature, mixed acidosis, increase in myoglibin and CK Thyroid: not associated with elevated CK, rigidity, lactic acidosis

What are the post op complications from a carotid endarterectomy

MI Stroke Stridor-phrenic nerve, hematoma, hypocalcemia, recurrent laryngeal Htn-> cerebral hyperperfusion syndrome hypotension hematoma-> airway obstruction

What is your biggest worry with aortic stenosis?

MI, due to increased LVEDP

Number one case of mortality in a carotid case

MI, think of an echo probe

What TEE view is best for looking at RVH and right ventricle and pulmonary stenosis

MID esop right ventricle inflow outflow

OB and multiple sclerosis. How does pregnancy affect the disease state

MS relapse decreases in pregnancy avoid infection, fever, succinylcholine

What is your treatment for torsades?

Mag 1 gm q5 as needed

What are your concerns about a sitting crani

Maintaining stable hemodynamics to ensure adequate cerebral perfusion pressure Intraoperative brain relaxation to facilitate surgical access Absolute avoidance of further increases in intracranial pressure

You have a hypotension in a trauma, whats your next steps

Make sure patient is not hypocarbic, hypoxic, malignant rhythm, check position of a line transducer, inform surgeon and look at surgical field for hemorrhage, then fluids and bolus as a temporizing measure until a definitive diagnosis is made

How does diabetes affect your anesthetic care

Manifestation questions means go by systems Cardiac: increase risk of MI and arrhythmias so EKG Pulmonary: none GI: Consider full stomach due to gastroparesis Hematologic: anemia of chronic disease so cbc and electrolytes so I would want the patient type and screened Renal: BUN/CR and adjust medications accordingly Neuro: increased risk of stroke, and peripheral neuropathy so have consider if certain regional techniques are viable options

Whats your extubation criteria

Mechanics: tidal volume, secretions, RR, FRC, VC, NIF >25, normothermia, stable Oxygenation: PaO2, PF ratio, sat, PaO2 70 on FiO2 40 Ventilation: PaCO < 55, ETCO < 50, normal pH I choose FiO2 of 40 because that's a nasal cannula

DDx for bradycardia in a carotid endarterectomy

Medication or manipulation to the carotid sinus and if regional was performed a sympathectomy

Normal hgb and Hct for men and women

Men 42-52, Women 37-47, Men 13-17, Women 12-15

How often can you give methergine and hemabate?

Methergine every 2 hours hemabate every 30 minutes for a max of 8 doses

What is bone cement implantation syndrome, what is the name of the compound in bone cement that caues problems

Methylmethacrylate causes medullary hypertension which can lead to embolization of marrow, fat, and cement. The methylmethacrylate monomer itself can cause a drop in SVR Bone cement syndrome symptoms hypotension arrhythmias hypoxia-from pulmonary htn -> shunting (cause of death)

What TEE view is best for assessing venous cannula or an ASD

Mid esop bicaval

Which valvulopathy is a contraindication to placing a PA catheter on someone?

Mitral stenosis, it can lead to PA rupture most likely

Whats an EEG

Monitors the activity of the surface of cortical cells vs SSEP which is able to evaluate more deep brain structures

What is the acute treatment for MI

Morphine, oxygen, aspirin, nitrates if pressure allows, and beta blockers if heart rate allows.

Where are most functional carcinoid tumors located

Most pts with intestinal carcinoids tumors do not have symptoms because more vasoactive substances are cleared by the liver. However, once the liver has mets then carcinoid tumor then becomes functional.

What blocks can you do while the patient is asleep

Motor blocks, fascial blocks

What is preclampsia and how do you categorize it

Multi-organ disorder that presents after 20 weeks and remission 48 hours after surgery There's preclampsia and severe preclampsia preclampsia: SBP 140 and/or diastolic 90 with proteinuria 300mg/24 hours. Must have 2 bp reads 4-6 hours apart. Severe preclampsia: Is preclampsia plus one of the following: -bp greater than 160 or diastolic 110 -end organ damage -renal dysfunction showing 5g protein/24hours -CNS -Liver -HELLP -RUQ pain -pulmonary edema -low platelet

How would you induce this patient?

My goals for induction are : blank blank blank and to achieve these goals Will use drug A for blah drug B for blah and drug C for blah

What kind of peep do you want for a neuro case?

NONE!!

What is the fluid of choice in neuro anesthesia?

NS, LR has slightly hypotonic

Would you use a pulmonary artery catheter?

Nah just a TEE

You intubate a child for rigid bronch? Will you place lidocaine in the airway

Nah the ENT surgeon will perform a DL and intubate with a ventilating bronchoscopy as the circuit will be attached to the side port of the bronchoscope. It is never ok to place lidocaine in the airway as it causes one to cough, except in an awake intubation

What are your main concerns for the sitting position

Neck hyperflexion and cervical dislocation external pressure on the eyes from headrest poor visualiation, acces, and confirmation of ETT and breathing ciruit jugular venous compression more prone to cerebral ischemia from venous pooling

A patient with carcinoid is hypotensive, which pressor do you choose neo or ephedrine?

Neo, indirect acting beta agonists stimulate the carcinoid. Beta agonists stimulate but alpha agonists do not. So your choice of pressors should be phenylephrine and vasopressin

What are thoughts on neuraxial anesthesia in a pt with HSV

Neuraxial anesthesia should not be performed on patients who have an untreated primary hsv infection or who are prodromal

Post op concerns of endarterectomy

Neuro exam hemorrhagic stroke htn MI Neck hematoma stridor being recurrent nerve or phrenic nerve palsy

You have a 3 y/o with an intracranial mass, what would you like to know? He's going for MRI

Neuro hx, seizure, ICP is this kid an aspiration risk, if not LMA difficult stick febrile MRI Compatible equipment and crash cart close LMA or ET? If no aspiration risk and no signs of increased ICP then LMA

What are some systemic manifestations of renal failure?

Neuro: Encephalopathic, peripheral neuropathy, seizures Cardiac: HTN, CHF, cardiomyopathy, CAD, hyperdynamic circulation Pulmonary: pulmonary edema may occur GI: Gastroparesis, pancreatitis Hematologic: anemia, coagulopathy, metabolic acidosis, low salt/calcium, high K

What are the systemic manifestations of OSA

Neuro: Hyper somnolence, increased sensitivity to anesthetic agents. Cardiac: Htn, LVH, pulmonary HTN Pulmonary: Increase V/Q mismatch from decrease FRC and atelectasis. This usually due to the comorbidities that accompany it. GI: Usually upward displacement of stomach due to abdominal pressure -> GERD Renal: htn nephropathy

ROS for liver cirrhotic

Neuro: encephalopathy, if changes order head CT Cardiac: high output, low SVR, dehydrated intravascularly pulm: hepatopulm, low frc, ascites renal: hepatorenal, hyponatremia gi: ascites, gerd, delay emptying heme: coags, low platelet from hyperspenism altered drug metabolism hepatopulm is from shunts and hyper cardiac output is from shunting too the low SVR is from splanchnic vasodilatation

How does diabetes effects your anesthetic management

Neuro: risk of stroke, TIA Cardiac: orthosis-poor cardiac reserve, autonomic neuropathy, silent MI, resting tachycardia, orthostatic hypotension. Pulm: difficult airway due to neck glycosylation Renal: renal dz Endocrine: glucose monitoring, potassium monitoring, pH GI: Gastroparesis, GERD, full stomach, early satiety

What are the effects of the sitting position, go over it like ROS

Neuro: ulnar nerve injury there way the arms are positioned. Injury to the cervical nerves of the neck, ischial nerve injuries due to sitting. Decreased cerebral perfusion due to gravity. brachial plexus injury stretch Cardiac: prone to venous pooling and hypertension neck flexion from the position can decrease venous return. Pulmonary: neck flexion can kink the ETT. Keep two finger breadths between chin and sternum. Increase in FRC and improved ventilation as the diaphragm is pushed downward increase in FRC but offset by lung compliance GI: reduction in hepatic and mesenteric blood flow

Night before insulin dose, how do you manage it

Night before 80% of long acting 50% of NPH and hold regular if pm glucose is <120 Morning of surgery No regular, aspart or lispro give 1/2 NPH and 1/2 long acting is BS greater than 120

What's your insulin management before surgery

Night before and day of long acting: 80% and 80% NPH 80% AND 50% Short acting same and hold

There's a child at risk of MH, how will you place his IV?

Nitrous oxide

With epiglottis, would you want a xray?

No I wouldn't delay as this is a surgical emergency. if there was already a lateral neck xray I would see thumb print sign

Does thyroid state effect MAC values?

No Mac is unaffected by thyroid

A child is vomiting and the mom asked can he receive phenergan, how do you respond?

No as that has a black box waring for children under 2 years due to respiratory distress.

At 5.2 would you delay a renal transplant case?

No at 5.2 this pt likely lives here and its expected. I would continue if she was asymptomatic and would like medications ready to treat hyperkalemia

If doing a beach chair shoulder surgery would you use an LMA?

No because it can become dislodged and we don't have direct access to the airway during this time. You also can attempt to do shoulder surgery purely with a regional block

After giving a child post op racemic epi for stridor should you discharge to the floor?

No because racemic epi has a rebound effect. So the child must be monitored at least 3 hours.

If a kid has a first degree relative with MH susceptibility, do they need a halothane contracture test?

No because the disease is autosomal dominant we're going to assume full precautions

Could you use ketamine for wound debridements from burns?

No because the requirements from ketamine are so high and also NMDA receptors are down regulated when the victim is burned.

With a TEF patient would you demand a gastrostomy tube?

No because this can create a situation where gas passes through the lungs and out of the stomach like a shunt

In myasthenia, do you continue the morning pyridostigmine dose?

No clear answer. Recommend continuing it in order to avoid any respiratory difficulties before induction. Disadvantage is prolonged motor block with succinylcholine and difficulties with reversal when using neostigmine. Also the possibility of cholinergic crisis with the use of succinyl choline. Pyridostigmine blocks plasma cholinesterase, leading to prolonged succinylcholine.

What is post op cognitive dysfunction?

No clear etiology fluctuating mental status that occurs between post op days 1-3. The diagnosis of POCD requires preop and post op neuropsych testing. RFs include low education level, hx of alcoholism, history of stroke without residual impairment.

How do you rule out a C spine? Is an xray enough?

No it is not enough, you need an MRI as there may be ligamentous injury. 1. Absence of cervical tenderness 2. Absence of neurological deterioration and paresthesias 3. Lack of distracting injuries

For post op oliguria, would you send urine labs if you just gave the patient diuretics, for example a renal transplant

No it would completely alter the real results

Would you perform a regional technique for a liver transplant?

No liver dysfunction is associated with multiple coagulation derangements therefore, regional anesthesia is a high risk procedure

Key points for a TBI

No nasal intubation only reverse coumadin if ICH shown on CT CT scan reqs 1. GCS lower than 15 2. GCS 15 but LOC in the field 3. Neuro changes

Would you extubate a patient after a TEF repair

No neonates are prone to post op apnea Id also be worried about the tension placed on the anastomotic site if the patient was allowed to breathe spontaneously so I would want to keep this patient sedated and relaxed until the anastomosis has healed (5 days) and ensure adequate pain control

How do diagnose LBBB

No q waves, tall R waves, wide QRS, Deep S waves in V1-3 (M) R'R/notched R wave in V6

Would extubate a SAH on POD1

No rebleeding, SAH was small, evacuaed, no edema.

What are some tips for performing a pheochromocytoma resection

No succinylcholine, no desflurane, cardiac evaluation always needed.

Would you obtain PFTs

No they have not shown to affect anesthetic management in most cases. I can obtain most of my information from a detailed

You have a patient with increased ICP do you want to place a bolt first

No we will assume that the ICP is high and once a crani is performed the bolt values will be off, so instead we will set MAP goals with the surgeon

Is a femur fracture an emergency?

Only if its open. Patients only have 6 hours to be washed out and they can do that in the ED temporarily.

Will you perform a regional block for this ORIF

Only if the patient has full neurologic function and no risk of compartment syndrome

You have a patient with a post dural puncture headache. Do you perform a blood patch that day?

Only if you witness the CSF. Do you discharge them home? hell no

When do a TE fistula repair what are two other conditions that you must rule out? This is an urgent and not emergent surgery

Open PDA and VACTERL The cardiac part of VACTERL includes Tetralogy, open PDA, VSD, ASD with open PDA being the most common.

You still can't intubate them, what do you do?

Options now include:Invasive airway access, feasibility of other options, awake patient. Invasive: percutaneous tracheostomy, cricothyrotomy Feasibility of other options: regional, local, face mask, LMA Wake the patient up: awake intubation, cancel case If you wake them up and can't perform an awake intubation successfully, you might need an awake invasive

Name the four parts of tetrology of fallot

Overriding aorta, RVH, right ventricular outflow track, VSD

What is the criteria to be put on ECMO

PA-A greater than 600 for 8 hours O2 index of 51 for 5 hours (FIO2*Mean airway pressure/PaO2 Reversible cause

What is retrolental fibroplasia and what are the risk factors

PCA less than 44, hypercapnia, hypoxia, TPN, anemia

What is a continuous machine murmur

PDA

What are the two main treatments for ARDS

PEEP 5-10, as more than that can create lung shunting Decreased Fio2 so that the patient doesn't get oxygen toxicity

Contraindications to the seated position

PFO, ventral-atrial shunt, Pulmonary AV malformation, cardiac instability

Side effects of etomidate

PONV, pain with injection, adrenal suppression, myoclonus

What kind of monitoring will you use before placing ecmo cannulas and why

PTT ACT is for high heparin dosages and ptt for lower

Treatment for thyroid storm

PTU 6-8 weeks, beta blockers, k-iodine, steroids. Avoid k-iodine in preggos

How does PTU and methimazole work?

PTU inhibits thyroperoxidase which assists in creating thyroglobulin the precursor hormone. It also helps to block the conversion of T4 to T3

How would you treat a thyroid storm preoperatively

PTU, methimazole and potassium iodine for 10-14 days

What are some drugs to help treat a thyroid storm

PTU, methimazole, and steroids. Steroids help prevent conveersion of T4->T3

While doing a mediastinoscopy you lose pulse ox reading, what is your ddx

PTX which is a complication of mediastinoscopy, but this work up would strongly depend on what my EtCO2 is reading. If that also dropped I'm thinking about obstruction of the airway. Using a rigid bronch, push the ETT pass the airway mass. Another complication is recurrent laryngeal nerve injury

What are a few complications using a mediastinoscopy scope

PTX, compression brachiocephalic artery, recurrent laryngeal nerve injury.

In congenital diaphragmatic hernia the O2 drops to 80%, what's your ddx

PTX, severe phtn, comrpession of great vessels, blood loss, hypothermia, allergic reaction

What factors determine cerebral blood flow

PaCO2: 1ml/100g/min for each 1 paCO2 PaO2: Not much control here until the PaO2 reaches 50-60 pH: Temperature hematocrit

Signs of fat embolism

PaO2 <60, tachy, temp, fat in urine, drop in Hct or platelets, fat globules in sputum, petechiae cxr should show bilateral infiltrates

What happens when a blood gas is exposed to air

PaO2 stays the same, PaCO2 drops After 30 minutes, PaO2 drops and PaCO2 increases

Describe the symptoms of a corneal abrasion

Pain with blinking and blurry vision

What are the causes of PONV

Pain, hypoxia, dehydration, anesthetic agents

What are the contraindications and relative contraindications to regional anesthesia

Patient refusal, sepsis with instability, uncorrected hypovolemia, ongoing hemorrhage, coagulopathy Relative contras elevated ICP, back injury with neurological deficit, chronic back pain with neuropathy, localized infection at injection site.

What is your algorithm for increased peak pressure

Patient to machine listen to the chest palpate for subcutaneous emphysema ET tube in the original place, is the pt biting the tube place suction to just mucus plug and kinking hand ventilate Endobronchial intubation -drop a fiberoptic bronchoscope

How do you calculate a FENA

Pcr*Una/Pna*Ucr

What is the treatment for ARDS

Peep and decrease FiO2, permissive hypercapnia and decreased tidal volumes, mechanical ventilation, PA catheter

What are some drugs that can have an exaggerated response to indirect agonists like ephedrine, ketamine, and demerol

People who are on TCAs.

What is PTCA

Percutaneous transluminal coronary angioplasty

What is the risk of hypothyroid in OR

Pericardial and pleura effusions coma, depressed consciousness hypothermia bradycardia, hypotension that only resolves with the treatment of thyroxin impaired gastric motility-aspiration risk

Celiac plexus block, what drugs may you use? and what's the benefits of both

Phenol or alcohol. The celiac plexus resides at L1 in the retroperitoneal space. Alcohol directly demyelinates the nerve while phenol causes ischemia and toxicity to the nerves

You just placed a interscalene block and now the patient complains of shortness of breath. Why?

Phrenic nerve palsy

Trouble placing a femoral and arterial art line, what do you do?

Place an axillary, transduce the CVP, try a micropuncture kit, use an ultrasound

If a child developed a tet spell what would you do?

Place the infants knees into their chest. This decreases SVR and increases preload. Increase in preload is the goal Chemically: give a fluid bolus and give phenylephrine

With a TEF patient, what measures can be taken to minimize further aspiration

Placing a NGT into the blind pouch and suctioning it, head of bed 30 degrees and kept NPO.

How do you diagnose a pheochromocytoma

Plasma and urine metanephrines. VMA-vanilylmandelic acid. Can also use CT and MRI

Pickwickian Sequelae

Polycythemia, cor pulmonale, somnolence, reduced lung volumes, increased closing capacity, decreased FRC, V/Q mismatch, RV failure, chronic CO2 retention.

What are the positive and negatives to amrinone

Positive Vasodilator, pulmonary too increase in contractility Negative thrombocytopenia

What are the positives and negatives to using levophed

Positive first line in septic shock Negative This medication only increases cardiac output mildly so it's the first choice in septic patients decreases end organ perfusion

What are the positives and negative to dobutamine

Positives ionotrope increase stroke volume decrease filling pressures B1 strong, B2 mild Increases cardiac output First line in right heart failure* Negative No real change in blood pressure due to its B2 effects, while increasing cardiac output, so shouldn't be used alone Increases myocardial work and O2 consumption

What are the positives and negative of phenylephrine

Possible decrease in cardiac output Should be avoided in spinal cord injury as it has shown to decrease blood flow compared to other pressors

Would you perform your regional block in the OR? pre or post

Pre as you need to assess neurologic function. Post has ok as long as the patient has adequate clarity to assess for paresthesias.

How does OSA effect anesthetic management

Pre-op: assess for comorbordities. Pre-op thorough airway exam because most are a difficult airway. Intra-op: minimize sedatives because they have an increased sensitivity to CNS depressants so minimal to no versed. Thorough pre-oxygenation due to low FRC, difficult airway cart in the room and consider an awake intubation if away was non-reassuring. During surgery use multi-modal pain regimen to minimal opioids including regional, and local wound infiltration. Post-op: extubate in head up position to improve pulmonary mechanics and extubate after fully awake. CPAP machine in the pacu and monitor closely for episodes of apnea and desaturation.

What monitors would you want for a TEF repair

Precordial stethoscope, and another over the stomach, gtube to waterseal and monitor for bubbles, arterial line in the umbilical artery of the fem, atropine with laryngoscopy

Post op oliguria in renal transplant

Prerenal: very unlikely unless the patient was just dilated Intra: most likely and think plumbing whenever you have oliguria-poor anastomosis, PE, clot, graft collapse. Place an ultrasound probe on the stomach with color doppler and see if the patient has flow. Post renal: flush foley and if you get back what you put in If due to toxins do not forget you can also dialyze the patient intraop

What are the risk factors for uterine rupture

Previous uterine scarring, use of prostaglandins, inductions, uterine trauma, forceps delivery, breech version, uterotonics, fetal macrosomia, multiple gestation

What is primary hypothyroidism and what would the labs look like

Primary is of the organ so high TSH and low T4

Name some medications the prolong NMBDs and decrease

Prolong Volatile, anti-arrhythmics, aminoglycosides Diminish phenytoin, calcium, steroids

Describe restrictive pericarditis in CVP

Prominent a and v waves with steep x and y descents and produces the square root sign

What are the pros and cons of a preop transfusion in sickle cell patient

Pros: reduces amt of HgS Cons: increase in viscosity which creates stasis which is a risk factor for sickling. Pre op transfusion is controversial but the target goal is 10 Hgb

Contraindications to neuraxial for OB

Pt refusal coagulopathic poor anatomy hemodynamic instability

Contraindications to a CEA

Pt refusal previous stroke on the same side complete obstruction

What are the indications for bacterial endocarditis pplaxis

Pts undergoing skin, mucosa, or respiratory tract 1. Prosthetic valve/prosthetic material used for repair of valve 2. Previous IE 3. Unrepaired cyanotic heart dz 4. Completely repairs congenital heart defect with prosthetic material during first 6 months after. 5. Repaired CHD with residual defect 6. Cardiac transplant who develop valvulopathy

What are the 4 P's to SOB

Ptx, pneumonia, PE, pulmonary edema

In DLT with hypoxia do you clamp the pulmonary artery of vein?

Pulmonary artery

ddx of blood in the ETT

Pulmonary contusion, bronchial artery injury

TEG values and what do they mean

R is onset K is strength Alpha is speed MA:Point of maximum clot formation and correlates with platelet function Lys30: percentage of clot decrease 30 min after MA and represents fibrinolysis Normal R is 4-8 min Normal K 1-4 min, time to achieve a certain level of clot strength Normal Alpha is 45-75 degrees Normal MA is 55-75 mm Normal Lys30 is 0-8%

What are the TEG values and what products would you give

R value is elongated, give ffp K value is decreased give cryo Alpha value is decreased give cryo MA decreased give platelets

In a TEG what is an R, K, alpha angle, MA

R value is the time from start to clot formation K take taken to obtain a certain level of clot strength alpha is the slope of the angle between these two lines. Alpha is the speed of fibrin build up and cross linking MA represents the overall strength of the clot

What is a normal RA and RV pressure

RA 3-8, RV 15-30/3-8

What are your respiratory parameters for extubation

RR 10-30 breaths per minute, sat greater than 95% with FIO2 > 40%, VC greater than 10ml/kg of ideal body weight, and TV >5 ml/kg of ideal body weight.

Tips for crani

RSI due to cushing ulcers CSWS VS SIADH question Mannitol side effects Positioning questions Decreasing ICP treatments Monitors need to include a TEE or precordial doppler and SSEP/MEPs Central Line

Will you use succinylcholine for a crani intubation emergently?

RSI with ROC if the air way is reassuring and suggamandex is available. Otherwise succinycholine, not securing the airway quickly will raise the ICP way more than succinylcholine

What is the induction of choice for TEF

RSI with gastrostomy

If you have a difficult airway but don't want to do an awake because its a trauma patient and/or you do not have cooperation, whats a great way to secure this airway

RSI with rocuronium and have suggamandex ready

What kind of induction would you perform on a myasthenia gravis patient?

RSI with succinylcholine. These patient are always a full stomach due to aspiration risk and even though they are not sensitive to succinylcholine doesn't mean you still can't use it.

What are the four images for a FAST scan

RUQ, LUQ, subxiphoid, and suprapubic

What kind of room setup would you want for an aortic dissection patient

Rapid infuser several units of blood/platelets/ffp right and one in the leg arterial lines so you can monitor BP before and after the aortic cross clamp two different pulse ox central line, TEE, foley, crash cart in the room, cell saver

Why is a spinal bad in MS

Reactivation of CNS plaques

What is your ddx for wheezing

Reactive airway, PNA, asthma, aspiration

During aortic dissection surgery the surgeon removes the cross clamp and the bp plummets, whats your next steps

Reapply the clamp, fix the BP and remove the clamp incrementally

What is your ddx for post op stridor after a thyroidectomy

Recurrent laryngeal nerve injury, laryngospasm, tracheomalacia, hematoma formation, inadequate reversal, residual anesthesia, hypocalcemia.

What are the advantages and disadvantages of norepinephrine

Reflex brady can occur, alpha 1 stim can decrease renal, mesenteric and peripheral blood flow, but also increase myocardial oxygen consumptions

What are the advantages/disadvantages of performing a TURP with regional and general anesthesia.

Regional Advantages:ability to monitor mental for TURP, MI, bladder perf. Reduce need for opioids for post op pain control avoidance of instrumenting the airway with associated hemodynamic changes Disadvantage of regional intraop anxiety and awareness develop high spinal/epidural inadequate level of blockade Benefits of general secure airway lack of awareness assure proper depth Disadvantages of general are inability to monitor mental status increased need for systemic opioids for postop pain have to perform an induction

What causes cerebral salt wasting syndrome

Release of natriuretic peptide by the brain

You are unable to ventilate the patient, whats your first step

Reposition the patient

What is your differential for a myasthenia pt that feels weak in PACU?

Residual anesthetic, narcotic overdose, hypothermia, acidosis. Perform a tensilon or edrophonium test to evaluate myasthenic crisis vs cholinergic criss.

What is acute chest syndrome

Respiratory symptoms, fever, pain, hypoxia, infiltrates on chest xray, sputum production, cough, shortness of breath. Acute chest syndrome can occur post op also days later. DDx pneumonia, PE, pulmonary edema, ARDS

What changes are there in pulmonary mechanics of an obese pt

Restrictive pattern

Retrobulbar vs peribulbar

Retro: meds into the muscle cone blocks nerve and ciliary ganglion uses less volume but deeper Peri injections above and below orbit block is outside the muscle cone less dense, but harder to completely block

What are the contraindications to a PA catheter

Right heart mass that can be dislodged Prosthetic tricuspid or pulmonary valve that can be damaged Tricuspid or pulmonary valve that has vegetations LBBB-it can go into complete heart block WPW and ebsteins anomaly because it can lead to a lethal arrhythmia

What are the down sides to a PA Catheter

Risk of infection, pneumothorax, vascular/cardiac injury, and bleeding

What paralytic and dose would you use for a burn patient?

Roc, but increased dose as burn patients are insensitive. Once you reach 25% BSA, the dose may have to be 3-5x as much!!

How much blood is in 1 unit of Hgb

Roughly 250cc of blood loss will drop Hgb one. 4 units is about 1 L

What is an S3 heart sound and S4

S3 Heard during early filling and can be normal in children represents a dilated ventricle and blood that is speeding into the ventricle due ventricle systolic failure S4 is heard during atrial kick against a stiff ventricle

Cause of S3 heart sound and s4

S3 is early diastole and due to very compliant ventricle and is a sign of systolic CHF S4 is late diastole and stiff ventricle is a sign of diastolic heart failure

Post op after SAH you notice sodium is 123, what are some causes

SIADH and cerebral salt wasting syndrome SIADH no urine, overloaded, low serum sodium CSWS, high urine output, volume depleted , low sodium CSWS and SIADH both have elevated urine sodium

You doing a trauma for a traumtic brain injury, and in the PACU you're paged for a sodium of 124, whats your ddx

SIADH, CSWS, dilutional from resus

If you have a traumatic brain injury, what are the two things that can occur in PACU with urine?

SIADH, and CSWS

Whats the difference between Sepsis, Severe Sepsis, and Septic Shock

SIRS (systemic inflammatory response syndrome) must have 2 or more -Fever -WBC greater than 12 or less than 4 -RR > 20 -HR > 90 Sepsis SIRS + Infection Severe Sepsis Response to fluids -End organ damage Septic Shock not responsive

What extra monitors do you need for a triple A repair

SSEP MEPS, epidural for post pain, lumbar drain, TEE, rapid infuser, cell saver

What are some EKG findings with tamponade

ST changes and low voltage QRS complexes

What are the normal ecg changes of a patient on digoxin

ST depression, prolonged PR but narrow qt, t wave flattening/inversion, u wave qt interval narrowing. so the QRS becomes narrow, st depression, t wave flattens

What are the normal murmurs of pregnant

ST depression, systolic regurgitation, LVH, sinus tach, t wave flattening

Name some normal changes on EKG for a pregnant woman

ST depression, t wave flattening, systolic murmur

If you have a case of mediastinal mass, what should automatically come to mind

SVC syndrome

How do you calculate SVO2

SVO2 =SAO2 - (VO2/13*Hgb*cardiac output) VO2 is oxygen consumption The most accurate value is drawn from PA port on a swan

What are the effects of low phosph

Seizures, rhabdo, hemolysis, liver failure, arryhtmias

Whats the cause of PSVT

Sepsis, thyroid, medications, PE, volume shifts, intrinsic heart dz

What is HFOV

Several small tidal volumes placed at a constant pressure but it uses a machine piston so there's positive and negative pressure changes. vs HFJV which keeps intrapulmonary pressures low and may be idea lfor bronchopulmonary fistulas. Also you can attach a catheter and ventilate certain portions of the lung if need be and works by bulk convention. HFJV uses tidal volumes 1-3ml/kg with passive exhalation sdfx of HFJV: ptx, pneumomediastinum, inadequate volume/exchange contraindications is not having an airway patent enough for passive exhalation and relies on the lungs elasticity. The most common complication is hypercarbia

What are some issues that need to be addressed when a patient presents with chronic htn who is on decadron and HCTZ

Shift in the autoregulation curve of MAP

A patient has elevated bilirubin and jaundice after losing 2L of blood acutely, why?

Shock liver has likely occurred. You need to call this ischemic hepatitis.

How would you differentiate between cerebral ischemia and VAE?

Similarities: ETCO2 can drop for both, EKG changes VAE: Precordial doppler, echo, end tidal N2 Ischemia: light reflex, blown pupil, brainstem reflexes

What is the difference between sinus tach and SVT

Sinus tach is 100-150 and SVT is usually higher sinus tach usually still see p waves SVT is a sudden onset

What is the dose of adenosine in SVT if the patient is "clinically unstable"

Six in central line, 12 in peripheral

What are the factors of survivability when it comes to burns

Size, extent (full thickeness), inhalation injury leading to vent dependence, hypovolemia leadings to ATN

What is Pierre Robinson Sequence

Small jaw, glossoptosis, airway obstruction, cleft palate

Mechanism of hydrochlorothiazide

Sodium channel blockers in distal loop, leads to increased calcium

What are transient neurologic symptoms? TNS

Something one can get from a spinal, particularly with lidocaine. It is a painful condition of the buttocks and thighs that does not show any neurologic symptoms. It an start a few hours after a spinal and can last for several days. It is exclusively a pain syndrome.

What are some sx of increased ICP

Somnolence, HA, n/v, ocular palsies, papilledema, and mass effect on CT. Cushings triad on VS.

What is the alpha angle in a TEG

Speed of clot formation

TIPS about triple A

Spinal cord ischemia Double Lumen Tube Renal insult

For a TURP would you perform a regional or general anesthestic

Spinal, as I would want this patient awake so I could assess neurologic status

There are two ways to do a foreign body retrieval in a child, spontaneous ventilation or paralyze with interrmittent ppv, what are the adv/disadv of both

Spontaneous advs: no interruptions of childs respirations better gas flow, better ventilation and better V/Q mismatching Spontaneous disadvantages OR pollution ignoring full stomach precautions requires higher anesthetic concentrations potential for coughing, dislodging of the body Control advs easier to regulate depth aspiration minimized Control disadvs fb may dislodge

What monitors will you use for an OSA pt

Standard ASA, twitch monitor, second IV, foley

What are some predictors of renal failure for CPB

Starting Cr of 1.9 and combined/complex surgeries

What are some of the values you can receive from PFTs

Static and dynamic values Static: ERV, IRV, VC, Dynamic: FVC, FEV1, FEF25-75, maximum voluntary ventilation (MVV), Gas exchange tests: nitrogen, diffusion capacity etc

Whats the ASRA guidelines for IV heparin

Stop 4-6 hours before block and check a ptt using after procedure-no guidelines Stop 4-6 hours before catheter removal Restart after catheter removal 1 hour

a pt develops TURP syndrome and they're hyponatremic, how do you replete their sodium

Stop irrigation and start running 3% NS with goal of correcting 0.5meq/L/hr until symptoms resolve or sodium i greater than 120 and then switch to NS.

What is your work up for magnesium toxicity

Stop the gtt order a mag level check reflexes check last and current EKG give calcium and a diuretic

Describe cardiac tamponade on a CVP wave

Strong x descent and attenuated y descent. This is because the right ventricle collapses and y disappears because the ventricle doesn't fill

What are your concerns with a patient with down syndrome airway

Subglottic stenosis large tongue small jaw hypothyroid mandibular protrusion small neck atlanto-axial instability OSA

What are some risks of performing laparoscopy

Subq emphysema, CO2 emboli, ptx, hypotension

How would you do an RSI in an myasthenia patient?

Succinyl choline dose would be 2mg/kg. If the patient is not an aspiration risk then intubate with fent, proprofol, and lidocaine.

Advantages of the sitting position

Surgical exposure

Indications for ICD

Sustained VT, LV dysfunction, low EF, Long QT, HCM, cardiac arrest due to VT/VF

How do you know if you intraoperative fluid shifts

Swelling of conjunctiva, mucous membranes, skin turgor, JVD, edema

Your patient goes into atrial fibrillation, and is hypotensive, what's your next step

Sync cardioversion, labs: H/H, ABG, electrolytes, cardiac enzymes

List the physiologic effects of OSA

Systemic and pulmonary htn, cognitive impairment, arrhythmia, LVH

What kind of cardiac ekg changes would you see with SAH

T wave inversions/flattening, ST segment elevation/depression, u waves and QT prolongation

Whats your regional epidural technique for knee surgery

T10 level check a level .25% bupiv bolus with 1/8th infusion

At what level/vertebral body that if doing an epidural will the patient start to develop ome muscle weakness

T6 level, this not only will lead to anxiety but risk of aspiration

Where would you put your epidural for a laparotomy?

T9, middle of the incision

What is the treatment difference between TRALI and TACO

TACO you diurese, TRALI you do not

How do you diagnose a VAE

TEE is the most sensitive, the most sensitive is end tidal nitrogen

In pediatrics, why avoid nitrous in foreign body aspiration

The main reason is that it will increase the amount of air entrapment of affected lung like a balloon then inhibit methionine synthetase

In mital stenosis your LA dilates, why is this significant

The more dilated the more arrhythmogenic, if greater 5 cm then more likely cardioversion won't work

Where do the posterior spinal artery derive from?

The posteior inferior cerebral arteries

How does TURP syndrome occur

The prostatic uretrha is rich rich in venous sinuses that become exposed during resection. Since its performed using a cystoscope the field must be clear with irrigation fluids which leads to hyponatremia. The absorption volume is dependent on the number of open sinuses, , flow rate of fluid, hydrostatic pressure, and duration

if doing a routine surgery for a htn patient and you do an EKG and see LVH and q waves, how do you proceed

The q waves are indicative of past ischemia or infarct and tells me they have CAD. We would now compare this EKG to an old one to see changes are new, if so patient would need to undergo cardiac evaluation with echocardiogram or stress test.

Describe the differences in pediatric breathing mechanics of the chest and lungs

The ribs are highly cartilagenous. This makes it difficult for them to rely on accessory muscle use and they have to depend much more on the diaphragm.

If doing a mediastinoscopy for biopsy of a mass, which arm would you place the arterial line in and why?

The right radial to evaluate for brachio-cephalic compression. Pulse ox on the right hand and left foot.

What size ETT do you place in a neonate? when does the Age/4 +4 rule start

The rule starts at two years of age and a neonate tube is usually a 3.5

The question of will you transfuse?

The the bleeding profuse, is the surgeon able to control it and are the vital signs changing

How does ACT work

The time it takes for whole blood to clot when an activator is applied

If your carcinoid pt had a crisis which is now controlled would you want to continue the gtt in ICU?

There are no guidelines so it would depend on length of surgery, severity of tumor, type of procedure, chance of undetected metastases. Most people taper it over a week.

Aside from TURP syndrome, what's a big concern about doing a TURP?

There's going to be a massive fluid load absorption by the patient so one has to be aware if there's any cardiac risk factors involved.

How does an oxygen analyzer work

There's many different types but they all measures partial pressure of oxygen relative to atmospheric pressure

A patient has a carotid plaque, which side will you place your central line

There's no right answer. if you go plaque side you may dislodge plaques, if you go normal side and pierce the carotid you may have global hypoperfusion.

What is a pediatric anesthesia circuit and what are the differences from an adult circuit

These are referring to the Mapleson anesthesia circuits. They're classified based on gas inflow and overflow valves relative to patient connection. in neonates and infants you can use these semi-closed mapleson circuit systems as the closed circle systems have too much resistance from CO2 absorber, unidirectional valves and breathing tubes.

Why is ketamine not commonly used in ICU patients?

They are catecholamine depleted and likely to act as a only a depressant

What are some things you must know about ascites

They are catecholamine depleted, hypovolemic (intravascularly depleted due to third spacing and ascites) so you must do a cardiac stable induction.

When a patient has ascites will you tap then before induction?

They best method is to tap them a day or two ahead of time. If you tap right before induction they may have rapid reaccumulation of ascitic fluid from the intravascular space leading to decreased BP. Also if you tap it, you can drop the tension the ascites has on the vasculature leading to a drop in preload. On the other side if the patient is having difficulty breathing then you wouldn't bring them to the OR anyway! but they would need to get tapped.

What are some considerations when using PFTS for thoracotomy patients

identifiy patients at risk of post op morbidity and mortality identifiy patients in need of short term or long term post op ventilation and we do this by calculating lung function We calculate the predicted post op FEV1 and the DLCO

What is RVH on an echo

if the wall greater than half a cm

Do you open the hematoma on an obstructed patient after a carotid endart

if there's time, open the wound if no time, intubate

CABG and tamponade...you can't use that answer why

if they chest is open there's no such thing as tamponade, only until the chest is closed.

What are your concerns about regional in a hyperthyroid patient

if they're currently undergong a thyrotoxicosis do not perform and neuraxial because they're volume and catecholamine depleted and you'll drop them

What is the ACLS algorithm for a fib

if unstable you cardiovert 150-200 joules start IV Heparin if possible You don't chemically cardiovert an unstable patient if stable there's two routes if the patient is low risk: ie on OAC for greater than 3 wks or afib <48 hours then chemical or electrical cardiovert if the patient is high risk then echo or OAC for 3 weeks then cardiovert chemically

What is the cause of preclampsia

imbalanace of prostaglandins from the placeneta

what is the algorithm for PEA and asystole

immediately start CPR get an advanced airway continue CPR for 2 min while IV access is established Give epi 1mg every 3-5 min, may sub 40 units of vasopressin after 2 min check rhythm evaluate and go for H's and T's compressions should be at least 2 inches deep Ventilate 10 breaths/min with advanced airway

What is the down side to precedex

in a hypovolemic patient the response of hypotension and bradycardia can be exaggerated.

For a large anterior mediastinal mass where would you place your central line

in the femoral artery, less likely to compress and you would need b/l lines for

What are the causes in a cardiac case of giving heparin and the ACT does not increase?

inadequate dose, expired heparin, ACT machine busted, kinked IV tubing, heparin resistance

What is the definition of shock

inadequate oxygen delivery to match normal cell demands cause end organ dysfunction

If hypokalemic, why correct potassium?

incase you have to give diuretics or hyperventilate

What are the normal hemodynamic changes of pregnancy

increase in intravascular volume approx 50% third tri and 75% at birth anemia-even though rbcs increase 20% SV increases and HR increases 25% SVR decreases MAP decreases

Why can't we give nitrous while doing a crani

increase in size of venous air emboli, increase in cerebral metabolic rate, pneumocephaly

What are the effects of bicarb

increase sodium, decrease potassium and calcium

Peds physiology

increased MV/FRC -faster induction immature BBB increased total body water lower GFR, can't concentrate urine until 1yo Fetal Hgb so left shift Cardiac: fixed stroke volume, heart rate dependent due to non-compliant ventricle, higher parasympathetic tone so prone to bradycardia

What effects would you expect to see when the aortic cross clamp is placed for a dissection case?

increased bp, Increased wedge, increased CVP, increased preload and afterload Decreased cardiac output and ejection fraction distal to the clamp is decreased renal and mesenteric blood flow Decreased oxygen consumption, increased SvO2, metabolic acidosis, and increased catecholamine release

What are some mechanisms your body uses to compensate when anemic

increased cardiac output redistribution of blood increased oxygen extraction shift of oxygen/hgb dissociation curve when chronic

Complications of any regional block

infection nerve damage hematoma LAST

How do you treat post op hypoglycemia

infusion of d5 with frequent finger sticks for 3-4 hours. This usually occurs by decreased catecholamines leads to insulin sensitivity

What are your concerns with the prone position, how do you protect someone in this position

Thoracic/chest roll to take pressure on off the abdomen to prevent shunting. -Pad the elbows, knees, and ankles -neutral neck -check eyes, ears, nose

Why is airway always a concern in myasthenia

Thymus compression of airways, and they always have GERD so must be NPO

What conditions are associated with myasthenia gravis

Thymus hyperplasia, thymomas, rheumatoid arthritus, pernicious anemia, thyroid disease

What are the indications for a cranial trombectomy

Time of symptom onset, goal is 90 minutes, but can still be performed if less than 24 hours, early ischemic change on imaging, severity of stroke symptoms, location and level of pre-stroke functioning

What are the two main indications for HFJV

To access with surgical access and to optimize pulmonary function

Why have a PAC and a TEE in a CABG?

To have the PAC post op

What is the purpose of a chest roll and a hip roll in the prone position

To keep the abdomen free hanging. For example in a cervical laminectomy if you have compression on the abdomen then it can increase bleeding

What is the difference between the traditional liver technique and the piggy back

Traditional: complete vascular occlusion of the IVC, hepatic art/vein, SVC. As a result venous return drops in half and severe hypotension may occur. Piggy back technique: vena cava is partially occluded making the surgery more challenging but continuing active preload and avoiding bypass

What is TACO

Transfusion associated circulatory overload

What are the two things that lead to ARDS

Trauma and transfusions

What is a TAPSE

Tricuspid annulus plane systolic excusion, correlates with EF of the right ventricle

Tell me about your intraoperative and post op care of a myasthenia patient

Try to use short acting opioids and agents so that the risk of post op intubation is minimal Post op You have to make sure a ventilator and ICU bed is ready if you are unable to extubate a patient and use short acting opioids

Anesthesia and altitude, what are the changes

Turn all gases down except desflurane you have to turn up Also there's a higher chance of post dural puncture headache, higher chance of bowel and bladder distension Normal hgb goes from 15->22

During foreign body aspiration retrieval in a child the surgeon loses the foreign with their forceps and it slips causing hypoxia and the child is now hypoxic, you ask him to push the fb distally and it doesn't work, now what?

Turn the child lateral or prone in efforts to dislodge, and if this doesn't work then place the child on bypass This occurs when the fb is at the carina or obstructing a main bronchi

Name three conditions that give subglottic stenosis

Turners, acromegaly, and downs

What is a type 1 protamine reaction, what is a type 2 and a type 3

Type 1 hypotension due to histamine Type 2 Anaphylactoid reaction Type 3 TxA mediated pulmonary hypertension

What are the five kinds of TE fistulas and which is the most common

Type C is the most common at a rate of 75% Type A: Esophageal atresia with no connection to trachea. Type A is just a disconnected esopagus Type B: upper part of esophagus makes a fistula with lung and there's a blind esophagus coming from the stomach, the esophagus is opposite compared to type C Type D: Two fistulas. Upper esophagus makes a connection to the lung and so does the lower esophagus but the esophagus itself doesn't connect Type E is the H type fistula, no esophageal atresia here. Type D is the only one with two fistulas

What labs would you want for an SAH

Type and cross 4u, cbc, metabolic panel.

What are the 5 labs you should order for post op oliguria intra-renal causes and what are 4 common reasons for it

U/A with casts, FENA, Osms, urine NA, BUN/Cr. Causes substances, thrombi, contast, vessel occlusion.

What is the UNa and UoSM in a pre-renal oliguria

UNa < 40, and UoSM > 500

A woman comes into the ED who is pregnant, what is one test that needs to be done

Ultrasound looking for placenta abruption/bleeeding/rupture

You receive a blood gas of 7.3, PaCO2 of 50, and a PaO2 of 60, whats the result

Uncompensated respiratory acidosis with hypoxia

Why do CHF patients get elevated BUN/CR

Underperfusion

What is reperfusion syndrome in liver transplant

Unknown mechanism, it occurs in the first 5 minutes of reperfusion that is associated with high K and hemodynamic flunctuations ex low bp, bradycardia, SVT, elevated PAP

What are your hemodynamic goals in aortic regurg

Up, Up, Down. Maintain preload, avoid bradycardia, and reduce afterload

Lab differences betwene intra-renal and pre-renal oliguria

Urine osmolality, urine sodium, FENA, urine to serum creatinine ratio.

You have a patient with renal dz so you must do a roc RSI but consider this can prolong roc effect so what do you do?

Use suggamandex

Heart rate begins to drop after bypass...what do you do?

Use the pacer!!

What is your differential for post partem hemorrhage

Uterine atony

What are some options to control peripartum hemorrhage when medical options have exhausted

Uterine balloon tamponade uterine arterial embolization surgical iliac artery ligation emergency hysterectomy

What are the disadvantages from the seated position crani

VAE Hypotension airway obstruction from a flex neck cervical nerve damage

Side effects of mag

VAST Vasodilation anticonvulsant sedative tocolytic

What does VATS, EVAR, and TVAR stand for

VATS-video assisted thorascopic surgery EVAR-Endovascular aneurysm repair TEVAR-Thoracic endovascular aortic repair

What is the difference between VBG and ABG

VBG has higher bicarb and PaCO2 values and pH is slightly lower. The major drawback is the paO2 is not accurate

What do you think of when you hear of a holosystolic murmur

VSD, MR, TR

How does the beach chair position in shoulder surgery change your management

Venous pooling so well hydrated and pressors ready, cerebral perfusion is no longer accurately represented with the blood pressure cuff.

What are some contraindications to the sitting position

Ventriculoatrial shunts, pulmonary AVMs, foramen ovale

What kind of maintenance anesthesia would you perform for a renal transplant?

Volatile desflurane as it is less nephrotoxic . Avoid morphine, demerol because metabolic is neurotoxic and can cause seizures. For blockade use cisatracurium.

What are some treatments of hypothermia

Warm IV fluids, ambient room temp, forced air warmer, OGT with warm fluid, open surgical lavage

Patient with a drug eluting stent needs emergent surgery, what is your recommendation if under 365 days

We need to wait atleast 180 days if urgent. You have to weigh the risk of stent thrombosis vs surgical delay.

When do you correct hyponatremia and how?

When Na is 120 meq/liter, start 3% saline at no more than 100ml/hr and increas 1-2meq/hr with a max of 10 meq in 24 hours

How do you calculate total calcium when correcting for albumin and when do you do it?

When albumin is less than 4. Total calcium = ionized calcium + (4-albumin)*.8)

Explain the secondary gas effect

When nitrous is taken up into the capillaries from the alveoli it increases the concentration of other gases in the alveoli The concentration effect is the FIO2 of the gas effects the rate of PA

When you can you ask a trauma patient to move there neck?

When the c spine has been cleared and after primary and secondary survey. In addition to these I perform my own brief neuro exam.

In cardiac tamponade when do you paralyze the pt?

When the chest is open and the heart is accessible to the surgeon

What is placenta accreta and its risk factors

When the chorionic villi invade parts of the myometrium RFs include AMA Previa in the presence of a uterine scar, ie prior c section or uterine surgery

When does the area under a curve of an art line or a flowtrack not really indicative of volume status

When the patient is not in sinus rhythm

What is the value of MAP on your arterial waveform

Whether you're arterial waveform is overdamped or underdamped, MAP is the same

Whats your ddx for post op fever and tachycardia?

Wind, water, walk, wound, wonder drugs

Post op patient with tamponade, do a window bedside or take to a delayed OR?

Window bedside

What kind of vent settings would you want in the ICU for a patient with an open belly vs a patient with a closed belly

With an open belly you would want to avoid coughing and bucking, so I would keep the patient deeply sedating with assist control With a closed belly it is much easier for the patient to take a breath so SIMV

Aortic root dissection, how does this occur with CABG?

With cannulation of the aorta

In a myasthenic patient how will you induce them if you have to?

With propofol and remifentanil. If I have to use an NMBD I would use .1x ED95 dose

In pediatrics foreign body aspiration what kind of history do you want or would you like to do

Witnessed or unwitnessed Size/shape was CPR performed or medications given Baseline breathing and lung status-asthma/URI etc Exam: tripoding posture, drooling CXR

What is your worry with a pt with hyperkalemia getting a kidney transplant?

Worried about them receiving potassium from the transplanted kidney

What kind of surgery is required for a PDA repair?

a massive thoracotomy Note that some PDAs are too big for Indomethacin to work

What is myxedema

a mixture of mucopolysarrcharides in the dermis which creates swelling in that effected area.

What is the difference between the pediatric and adult spinal cord

a neonate dural sac ends at S3 and spinal cord at L3 In an adult dural sac ends at S1 and spinal cord at L1

What device after a cabg can you not forget to have?

a pacer

In a PDA repair what monitors will you need

a right upper extremity arterial line incase there's a tear of the PDA into the left subclavian and sometimes they have to clamp off the subclavian. Pulse oximeters on the right hand and lower extremity to have pre and post ductal readings.

What are PFTs

a series of test that evaluate respiratory mechanics, lung parenchymal function, cardiopulmonary interaction

What are the Roizen guidelines

a setup of criteria to see if someone is optimized for pheochromocytoma resection 1. Supine blood pressure 160/90 2. Presence of orthostatus hypotension no less than 80/45 3. EKG without ST or T wave changes 4. No more than one PVC every 5 minutes

In a pheo resection do you anticipate hyper or hypotension

a small % of pts remain hypertensive in he pacu. If this persists for more than a week then its like incomplete resection occurred. However, most pts experience hypotension. This is from persistent alpha blockade in the preop setting. Best treat is fluids and ruling out cardiac issues.

What is TURP syndrome

a state of overhydration from open venous sinuses and absorption of fluid leading to CHF, pulmonary edema, in association with hyponatremia

What are the complications from performing a cervical block?

a stellage ganglion block, dural space, seizure, infection, horners

If a tet spell occurred in the OR what would you do

abdominal compression, fluid bolus, phenylE, 100% oxygen increase depth of anesthesia or esmolol helps reducing infundibular spasm bypass is the last resort

How do you diagnose pyloric stenosis

abdominal ultrasound and the patient will have a hypokalemic, hypochloremic metabolic alkalosis. But if the patient continues to dehydrate they'll be a shift later to acidosis

What is placenta previa

abnormal placental implanatation in front of the presenting fetal part Abruption is separation of the placenta from the decidua basalis before delivery

Describe atrial fibrillation on CVP wave

absent a wave

Whats the difference between drug abuse vs drug addiction

abuse: continued use despite negative consequences addiction: a compulsion to use the drug

What are 3 major and more common causes of non gap acidosis

acetazolamide, hyperalimentation, diarrhea

What is the ddx for pyloric stenosis

achalasia of esophagus, duodenal atresia, hernia, meckel's, malrotation

Triggers for G6PD ?

acidosis, hypoxia, same as sickle cell

How do you diagnose DKA in a pt

acidosis, ketones in urine and blood, blood glucose, dyspnea, stomach pain

What are your preoperative concerns regarding a pituitary tumor

acromegaly: difficult airway, glottic stenosis -CAD, cardiorespiratory failure, HTN, OSA 70%, insulin resistance Thyroid, adrenal, and reproductive gland dysfunction leading to low cortisol and hypothyroidism nasal cpap is contraindicated afterwards due to pneumocephaly risk, there's also risk of VAE in this surgery Risk of diabetes insipidus afterwards

How do steroids increase blood pressure

activate RAAS, ADH inhibits nitric oxide which normally activates cyclic guanosine monophosphate

What is recombinant 7A

activates factor 9 and 10 side effects actually include bleeding, fever, pain bleeding into a joint contraindicated in head bleeds***

What are contraindications to PFTs

active hemoptysis, unstable angina, t/abdominal or cerebral aneurysms

What is the treatment for phantom limb pain

acupuncture, hypnosis, biofeedback

How do you use a reticulocyte count to determine acute and chronic anemia

acute anemia-high retic count chronic -low acute would be hemolysis chronic would be folate, b12, iron

Whats the difference between acute and chronic digoxin toxicity

acute: arrhythmias, N/V/D, K, poor rhythm chronic: color vision, halos

In triple AAA repair whats steps can be taken to protect the spinal cord

adequate BP Hypothermia CSF drainage shunt across cross clamp to improve distal blood flow

Describe the steps you would take when preparing to come off bypass

administer versed turn on machine/alarms/monitors correct lab abnorms check TEE make sure heart is de-aired initiate ventilation have pacer on standby

After 48 hours you decide to extubate a epiglottis patient, describe how you will do it

after ensuring the patient is afebrile, positive leak test, transfer to the OR, ENT on standby, neck preppred and draped, induce general anesthesia and perform a DL to observe if edema and erythema has resolved, and then extubate

Whats your ddx for hypotension

afterload-overuse of anesthetic gas, opioids preload-filling, bleeding, PTX contractility-MI, tamponade, drop a TEE rhythm-EKG

What are three risk factors for pulmonary artery rupture when floating a swan?

age, hypothermia, anticoagulation

What is croup

aka laryngotracheobronchitis is a subglottic viral etiology. inspiratory stridor, barking cough, hoarseness

What is the down side to a transfusion

alloimmunization Fe overload Infection

Tell me about mitral stenosis

almost always due to rheumatic dz relies heavy on atrial kick risk of a fib 25% of these patients will have LV dysfunction so be aware of pulmonary htn and CHF

Mechanism of methergine

alpha 1 properties but intracellular calcium oxytocin does the same

Whether its pregnancy or femur/tibia fracture, you have to think of what? kyphoplasty

amnioitic fluid emboli, fat emboli, cement implantation syndrome

What is a watershed area

an area of tissue that lies on a border between to areas that is susceptible to ischemia

What is myasthenia gravis

an autoimmune disease can effect ocular, bulbar, and respiration function

What is lambert eaton syndrome

an autoimmune disorder effects proximal muscles, improved strength with repetition and associated with other conditions such as thyroiditis, sarcoiditis, small cell lung cancer

How do you determine if its previa or an abruption causing the third trimester bleed?

an ultrasound will help but aside from that previa is painless bleeding while abruption is not. Previa rarely develops into shock and DIC where abruption usually goes into shock and DIC

After resuming ventilation after bypass the blood pressure drops, whats a possibility?

avulsed or kinked graft, heart block, arrhythmia

In a carotid endarterectomy case whats the best way to monitor the pt

awake

What the main term for how would you provide maintenance for this anesthesia

balanced technique

What drug can you use for cerebral protection in SAH or crani

barbituates as they reduce CMRO2 and institute hypothermia.

What factors determine spinal spread

baricity, and positioning

Why do we do epidurals in kids asleep

because they can't tolerate it

What are the ASRA guidelines for lovenox

before procedure 12 after procedure restart, never before catheter pull, 12 hours after pull 6 hours

What are the ASRA guidelines for low dose heparin sc

before procedure: 6 hours after procedure or catheter in place you can restart heparin immediately before catheter removal:6 hours When you can restart heparin after catheter pull: immediately

How would you treat post op thyroid storm

beta blockers, steroids, cooling, support, oxygen

What are the benefits to the seated position crani

better exposure less retraction and damage less bleeding less cranial nerve damage less facial swelling

How does albumin help with a fat embolism

binds to fatty acids

Biphasic vs monophasic defibrillation

biphasic creates less tissue damage requires less joules and has a higher probability of conversion

post op hypotension ddx

bleeding, check incision urinary retention hypoxia, hypercarb, rhythm pain, anxiety lack of home medication or beta blocker

Down side to regional

bleeding, infection, nerves, pt aware

Causes of hyperkalemia

bleeding, transfusion, hematoma, inability to excrete, ishcemia

Mechanism of lasix (loop)

blocks sodium, k, Cl decreased calcium is a side effect side effects include increased calcium

Pt has nausea and vomiting after ENT surgery or Uvulopalatopharyngoplasty whats the main cause

blood dripping into the stomach and now nauseous so tx with reglan

If you have no pulse ox, how do you tell oxygen saturation

blood gas, pale skin, color of the blood, gums

Describe the features of hemorrhagic stage 2 shock

blood loss 750 -1500 elevated HR normal bp RR 20-30 Class 4 is greater than 2L

What dose of diltiazem for afib

bolus .25mg/kg over 2 min then 5-15mg/hr

Half way through a hip or knee arthroplasty the BP drops what is it

bone cement implantation syndrome

What is treacher collins and pierre robinson syndrome

both have micrognathia and difficult airways but pierre robin syndrome is known for glossoptosis.

what is truncus arteriosus

both the ventricles and the outflow tracks show blood

Contraindications to cell saver

bowel content, infection, malignant cell, amniotic fluid, jehova's witness, bone cement

Side effects of beta blockers

brady, bronchospasm, hyperkalemia, alkalosis

How would you tell if a patient is having a myxedema coma in the operating room

bradycardia, hypotension refractory to pressors, hyponatremia, hypoglycemia, myxedema of the hands, lower extremities, and face

What are some signs that can be seen on cxr for an asthmatic

bronchial wall thickening

What are your concerns with a premature neonate

bronchopulmonary dysplasia, liver dysfunction, lack of feeding-lack of nutrients and poor clotting, meningitis, necrotizing entercolitis, post op apnea, retinopathy of premature, hypoglycemia, electrolytes imbalance (lack of feeding), IVH, anemia, renal and hepatic function, chronic steroid use pulmonary status: compliance, pulmonary htn

What is your ddx for post op pacu stidor in a adult

bronchospasm, laryngospasm, edema, foreign body

What tools do you have for pre-renal vs renal

bun/cr ratio

What is a good dose for a caudal block

bupivacaine .25% 2mg/kg This is done in the lateral position because usually you have to sedate the child so this is done to protect the airway

What precautions will you take to avoid recall

bzs stay greater than 1 mac

What is one thing you must always consider with a trauma

c spine injury

4 trauma keys

c spine, aspiration, hypothermia, hypovolemia/hemorrhage, ptx

Whats the first thing you do post op where there are issues

call surgeon, call the OBGYN

What is the danger of a history of coarctation of aorta

can become a dissection

How does maternal epilepsy affect the fetus

can lead to fetal hydantoin syndrome. This includes intrauterine growth restriction, skull, and facial abnormalities, potential cleft lip/palate, hypoplasia of nails and digits. Also a seizure can lead to fetal asphyxia

What are some risks to placing a pulmonary artery catheter

cannulating the carotid, bleeding, infection, PA rupture, arrhythmias, right bundle branch block

What are the risks of placing a pulmonary artery catheter

cannulation of the carotid, PTX, Vessel rupture, PA rupture, coiling, knot, infection, myocardial perforation

Contraindications to albumin

cardiac failure, renal failure, allergy, neuro injury

Tell me your concerns about duchennes

cardiomyopathy, aspiration, sleep apnea cause pulmonary hypertension, MVR -Don't give sedative premeds as they may already have pulmonary hypertension leading to right heart failure -Pneumonia -Poor laryngeal reflexes, aspiration -Poor cough -difficult airway due to tongue hypertrophy -Induce with ketamine -cannot use volatiles or succ with duchennes as even volatiles have shown to cause hyperkalemia**

Patient has vtach with a pulse and blood pressure is unstable

cardiovert

What monitors will you place in a Traumatic brain injury patient

central line for CVP monitoring, possible VAE treatment and resus, arterial line for H/H and ABG sampling

What are the advantages and disadvantages of roller pumps and centrifuge pumps in a bypass circuit

centrifuge have less air emboli and bubbles and trauma to rbcs and it is preload dependent roller pumps do not allow any retrograde flow

How does htn affect your anesthetic

cerebral curve shifted HTN leads to LVH and increased cardiac work load

List all the monitors to evaluate cerebral function during a case

cerebral oximeter, transcranial dopplers, EEG, SSEP, EEG

Post op cabg, pacemaker not working, whats your next step

change battery, get a new pacer, proper connections. Then pace via swan, transcutaneous, or venous

You cannot intubate a pt, now what do you do

change blade size, miller blade, bougie, intubating LMA, blind intubation, light wand, fiberoptic, video laryngoscopy.

What are your anesthetic considerations for a patient with hypothyroid disease

check labs because should be euthyroid check anemia check neuropathy check/trend blood pressures as its associated with low adrenal function airway exam and difficult airway due to possible macroglossia hypothyroid can lead to myocardial depression, consider giving hydrocortisone if this happens decrease spontaneous ventilation reduced plasma volume hypoglycemia impaired drug metabolism

What do you do with intraoperative recall

check record apologize change policy

Here is a blood gas, is it normal?

check the PaO2/Fio2 ratio. Even with a decreased Pao2, it may be normal if ratio is greater than 200

Patient's bp is gradually decreasing...what do you do?

check the drains

What something the patient should have done before undergoing anterior mediastinal mass resection

chemo or radiation to shrink it

Name two drugs that can give AT3 deficiency preop

chronic heparin use and nitroglycerine

When the stem says the patient has hepatitis or long standing hepatitis what should you think of

cirrhosis and then from there just think ascites, spleen/platelets, decreased FRC, high output, GI bleeding-varice, hemorrhoids, caput medusae

What complications are associated with massive transfusion

citrate toxicity, infection, dilutional coagulopathy, platelet dysfunction, hypothermia, acid base imbalance

What is charge syndrome

cleft lip/palate, micrognathia, laryngomalacia, subglottic stenosis and a TE fistula because of the TEF you have to have an echo

During a cerebral aneurysm repair is there anything you can ask the surgeon to do to decrease the transmural pressure?

clip a feeding vessel

What labs must you order if the patient has a previa, abruption or uterine rupture?

coags and type and cross

What are some tips for scoliosis

cobb angle restrictive lung blood loss, usually 200 cc per level hypoxia-> pulmonary vasoconstriction -> right heart failure

Whats another way to say normal not normal

common and not common

pH 7.35, paO2 65 PaCO2 of 55, how do you interpret

compensated respiratory acidosis with hypoxia

what kind of positional injuries can you incur from the lateral position

compression and stretch injuries of the brachial plexus

In an RA patient what does it mean when a patient extends their neck and get vertigo or light headed

compression of the vertebral artery indicating a C1-2 subluxation

Why not use succinylcholine in an open globe

contents extrusion

What does an oxygen analyzer do

continuously measures the FiO2 in the breating circuit Theres paramagnetic types -the strong paramagnetic properties of oxygen as compared to other gases. Sample and room air (reference) gas are drawn into a gap in a strong electromagnetic field which is rapidly switched on and off. The alternating pressure induced between sample and reference gases is detected by a pressure transducer, and amplified to produce a DC voltage signal and Galvanic types

What is cardiac ventriculography

contrast placed into the heart ventricle and get back EF, stroke, CO

What are your cpap and bipap settings

cpap of 10 and bipap 10/5

Compare croup and epiglottis

croup is 6months to 3 years usually doesn't require prolong hospital viral, self limiting epiglottis 3-6 years bacterial emergency

What products would you give to a hemophiliac in an emergency

cryo, factor 8, ddavp, and send a TEG

Crystalloid or colloid to a burn patient?

crystalloid

What change do you make in the dose when repleting potassium?

cut dose in half

When evaluating fluid status of a renal pt why do you need a cxr and an ekg

cxr to evaluate fluid status and listen to the lungs, ekg to evaluate for electrolyte arrhythmias

What is intussessception associated with

cystic fibrosis and heinoch-schonlein purpura IGA vasculitits

Post op fever ddx

day 1-2 aspiration chorio dehydration atelectasis day 3-5 UTI day 4-6 DVT day 5-7 wound infection

What cardiac changes does the heart undergo when a patient is burned

day 1-2, heart depresses and SVR tightens. Day 2 the reverse happens. Think of it as CO is depressing the heart.

Is dead space increased with extension tubing? what is increased

dead space is not, but there is loss of tidal volume and increased airway resistance so the disadvantage is it may be harder to break bronchospasm.

What are some premedications that you can give prior to foreign body retrieval aspiration in a kid?

decadron, racemic epi, and humidified oxygen

What are your anesthetic concerns about a neonate for GI surgery

decompress with NG no nitrous de air IV lines to avoid embolism use fentanyl instead of morphine maintain afterload to avoid shunt reversal

During a neuro case the surgeon argues that the dura is tight, how can you help

decrease blood pressure, diuretics, HOB up, hyperventilate

side effects of hetastarch

decrease factor 8 and VWF, platelet adhesion

What is the definition of a early fetal decel

decrease in 15 beats per minute onset to nadir is greater than 30 seconds The nadir of the contraction matches the nadir of the heart rate For variable decels, its less than 30 seconds onset to nadir For late decels its greater than 30 seconds

What physiologic changes may occur with epidural

decrease in after load and preload with reflexive tachy, vasodilatation with decrease in temp leading to shivering, bladder distention, contraction of bowel due to opposed parasympathetics, feeling of dyspnea

What can regional anesthesia do to your preload?

decrease it

How does methgb affect the body

decreased O2 carry capacity, impaired delivery

How would your teg appear if you were hypercoagulable

decreased R, increased alpha, decreased K, large MA

How can you use your physical exam skills to determine if a child as a foreign body aspiration

decreased breath sounds on the right side and mediastinal shift toward the normal side like a ptx

What are some side effects of amiodarone

decreased cardiac function, hypotension and bradycardia Long term therapy can cause pulmonary and hepatotoxicity, thyroid toxicity like toxicosis and blue-gray skin color changes

What are the effects of hypothermia

decreased insulin -> hyperglycemia cold induced enuresis, decreased drug metabolism, decreased platelet dysfunction

Why is a restrictive lung picture detrimental to the lungs

decreased lung compliance creates V/Q mismatch pulmonary hypertension decreased FRC Rapid desaturation Hypoxia may require increased peak airway pressures to maintain tidal volume decreased tidal volume decreased everything

Why are pregnant patients a full stomach

decreased motility, decreased LES tone, displaced stomach

what epi do for blocks

decreased systemic absorption increase duration and intensity

What is the exact mechanism

decreases water in parenchyma of cerebral cells side effects can be hypovolemia, hypotension, volume overload, acidosis, low K, high Na

A pyloric stenosis pt has an elevated hematocrit, explain

dehydration

Side effects of lasix

dehydration and electrolyte abnormalities, aplastic anemia, muscle cramps

What is your concern with cocaine and ecstasy

dehydration and temperature

Whats your differential for fever after placenta delivery

dehydration, opioid withdrawal, DIC, sepsis, MH, sepsis, opioid withdrawal

Why does pre op anxiety in a child matter

delay induction, slow recovery, the surgery can lead to long lasting behavioral issues, it can cause post op delirium, hallucinations, confusion. prevent with premeds

Post op in the ICU the patient is not having spontaneous ventilation, what do you do

delayed emergence workup

What is alpha thalassemia

deletion of alpha gene, which leads to excessive beta globin.

What is beta thalassemia

deletion of beta globin chain leading to severe anemia to asymptomatic, this causes a reduction in hemoglobin A which is normally two alphas and two betas hemoglobin A2 is two alpha and two delta

When do you do prophylactic endocarditis treatment

dental procedures 4 criteria previous prosthetic materal in the first 6 months after procedure unrepaired or partial with congenital heart disease transplant with valvulopathy no need in MVP, HOCM Take one hour before procedure and amoxicillin 2 gram or ampicillin 2 gram and keflex 2 gram

Can you perform a inguinal hernia repair under regional

depends on the size and dimensions if large it usually requires muscle relaxant

After doing a block the patient complains of pain in the PACU, How will you assess

describe pain, pinprick testing.

Whats the tx for diabetes insipidus

desmopressin

what does a line isolation monitor do

detect the max current that can go to ground

What fluids do you want to avoid in neuro

dextrose containing solutions as it leads to cerebral acidosis, diuresis avoid LR due to hypotonic Also avoid LR in resuscitation as it has calcium that bind to the citrate in blood and cause it to clot

Symptoms of magnesium toxicity

diarrhea, N/V, lethargy, muscle weakness, respiratory depression, decreased urine output

For non-obstetrical surgery when do you start fetal monitoring

differs by institution but usually at 24 weeks.

What three things must you have in the room when intubating a difficult airway

difficult airway cart ENT surgeon Neck prep

How would you secure an airway in a pierre robin kid

difficult airway cart in room, neck prep and draped with cric kit open. po midaz and perform a sedated awake fiberoptic while maintaining spontaneous ventilation with titration doses of ketamine

What are some intraoperative complications of the OSA patient?

difficulty airway hemodynamic instability rapid desaturation post op apnea diabetes CAD obesity hyper/hypoglycemia

Preparing for MH

disable and remove all vaporizers new circuit and reservoir bag Flush circuit 10L for 30 min Change CO2 absorbent Consult manufacturer for specifics and for placement of charcoal filters remove succ from room MH cart in room MH meeting with staff Lines ready contact ICU for a bed

Sudden loss of ETCO2 and you think its a VAE possibly, whats your ddx?

disconnected ET tube, PE, fat embolism, MI, arrhythmia

What the big worry about placing a PAC when the patient has a pacer

dislodging the pacer wires

What are the three phases of liver transplant and the worries about them

dissection-bleeding and volume, hypocalcemia anhepatic-drop in preload and cardiac output reperfusion-ischemic metabolites, plus volume load causing right heart failure. Vasodilation, vasoconstriction of lungs, right heart failure

What is CRPS 1 vs 2

distinguished by etiology. CRPS 1 results from injury anywhere in the body where 2 is a direct nerve injury.

What is the ddx for post op jaundice

divide to pre intra and post pre: over production of bilirubin due to hemolysis or resolution of hematoma, hemolysis. Can be from hemolytic reaction, and can be from going on bypass Intrahepatic: TPN, hypoxia, ischemia, decreased cardiac output, antibiotics, intrinsic undiagnosed disease. Post hepatic: stones, direct surgical injury, stricture use alk phosph and ultrasound

Two ways to prevent fat emboli in long bone fractures

do not wait to get surgery and early mobilization afterward

RFs for placenta previa

do vaginal exam in OR -Hx of previa hx of c section hx uterine surgery AMA multip

drugs that cross the placenta, drugs that do not cross

do:narcan coumadin ketamine propofol don't paralytics including succ heparin insulin

What are your thoughts on epidurals at outpatient surgery centers?

don't do them, they may have a complication such as a post dural puncture headache or urinary retention and then you'l have to admit them

What is the etiology to neuroleptic malignant syndrome

dopaminergic blockage in the hypothalamus

What are the requirements for prophylaxis infectious endocarditis

dose is amoxicillin 50mg/kg and if allergic clindamycin 20mg/kg 1. Prosthetic cardiac valve or prosthetic material used for cardiac valve repair 2. History of infective endocarditis 3. Unrepaired cyanotic congenital heart disease (CHD) 4. For the first 6 months after completely repairing a congenital heart defect with prosthetic material or device 5. Repaired CHD with residual defects at the site or adjacent to the site of a prosthetic patch or prosthetic device 6. Valvulopathy after cardiac transplantation CHD is congenital heart

Side effects of fever, side effects of hypothermia

fever: tachycardia -> increased oxygen consumption -> MI -Shift in hgb curve Hypothermia: shivering-> increased oxygen consumption, decreased drug metabolism-> hard to extubate -shift in hgb curve

With a cabg how do you treat heparin resistance

ffp, or recombinant human AT3

What does cryo contain?

fibronectin, VWF, factor 8 and 13, fibrinogen

What are your latex allergy precautions

first case sign on door nitrile gloves rubber stoppers foley epi ready no premeds as these will take away the first signs of anaphylaxis

Muscles of the ulnar nerve

flexor digitorum profundus-flexes two fingers and flexor carpi ulnaris-adducts when ask patient to open hand they can't extend those two fingers and thats the ulnar claw

Three things you do with a VAE

flood field PPV 100% O2

VAE treatment

flood field trendelenberg left lateral aspiration from Central line

During a crani you see drop in ETCO2, tachycardia, hypotension, what do you do

fluid the surgical field, call for help, 100% fio2, gentle compression of jugular veins, turn left side down, aspirate air, support

First step in pregnant distress or FHR drops

fluids, LUD, oxygen

What does FAST scan stand for

focused assessment with sonography for trauma

What kinds of lines do you need with severe burn patients

foley, central line to monitor fluid status, arterial preinduction line

Your patient was hypovolemic, how do you know if you properly resuscitated him/her

foley, vs, cbc, electrolytes, acid base status

After PDA closure, should this pt receive infectious endocarditis pplaxis, and for how long

for 6 months for any surgical procedures

How long should you monitor an MH patient

for 72 hours and may need a dantrolene infusion

How do you perform a superficial and deep cervical block

for superficial you inject 10ml along the posterior border of the SCM. For the deep you palpate C2 transverse process just below the mastoid process and inject at each transverse process down until C6.

Would you use 0.5% or .25% bupi in your block for surgery

for surgery you have to 0.5%, for post op pain 0.25%

When is it a good time to use a subcostal TAP block

gallbladder and liver procedure

Whats the difference in mortality with general vs regional

general has a mortality rate 16x higher than regional

Severe hypothyroid pt, will you do general or neuraxial?

general, these pts are severely catecholamine depleted and any neuraxial can bring on a sympathectomy thus if you have refractory hypotension in a hypothyroid pt, give synthroid

Two methods to diagnose malignant hyperthermia

genetic testing and caffeine halothane contracture test

Bleeding after bypass

get a TEG and an ACT

What's a very definitive way to determine if a mitral stenosis or any patient is fluid overloaded before extubation?

get a chest xray

If glyco and atropine don't work for badycardia then what

get a pacer

With dialysis patients what is a great way to tell if they're volume up or down

get the dry and wet weight also consider keeping the patient intubated and dialyzing them

Describe 1st degree AV block with tricuspid stenosis on CVP

giant a wave

While trying to establish an adequate ACT for bypass you're sitting at 270, what do you do now

give a bolus of heparin

Is a hip fracture an emergency? if a patient was on eliquis/warfarin how would you manage this

give the patient kCentra or plasma. Always give vitamin K with plasma

How to dx and tx fetal opioid withdrawal syndrome

give weaning doses of morphine dose .125-.15mg/kg in the first three hours dx: agitation, poor feeding, crying, dilated pupils, myoclonal jerks, excessive sucking

Two labs you want with cystic fibrosis patients

glucose and coags poor hepatic function and an unable to absorb fat soluble vitamins Gestational diabetes is very common for cystic fibrosis preggos

With low risk surgery what labs do you require?

glucose and pregnancy test

In a burn patient, after 24hours of the parkland formula, what fluid do you give next?

glucose to help diurese the kidneys and ffp to help replacement coags after you dilute the patient

What is a sulfonylurea and how does it work

glyburide, stimulations insulin secretion

How do you know your central line is placed in the proper place

go enough until you have a biphasic wave form ie right atrium then pull back a few cm and you'll be in the SVC

post op hypertension in a crani

go see pt, inform surgeon and ask MAP goal r/o pain, anxiety, h/h/mrhythm focal exam to rule out neurological anisocoria etc then treat accordingly with nitroglycerine

What are some advantages of infraclavicular block

good for catheters because they don't dislodge as the other places

What causes epiglottis

haemophilus influenzae type B

When to redose heparin during bypass, and how do you know when it's heparin resistance

half life is 2 hours redose is 100u/kg When you've done 500unit/kg and ACT not greater than 400

How long does narcan last?

half life is 30 minutes so 2.5 hours

What is the treatment for post op delirium

haloperidol 1-2 mg IV

During a case with a carcinoid pt you notice an increase in peak airway pressure. What do you do

hand ventilate to assess for compliance, check for ET migration. Increase anesthesia depth and if no improvement and you notice skin flushing and tumor manipulation give a 100 mcg bolus of octreotide

What is the mechanism of lovenox

has antithrombin bind directly to factor 10

Post op concerns about an OSA pt

have CPAP ready

The patient refuses an awake fiberoptic, what do you do

have a detailed discussion with the patient about the risks and benefits or doing the procedure differently

What is an early decel?

head compression and it is a normal response from uterine compression creating vagal response to intracranial pressure

Side effects of an epidural

headache, urinary retention, anaphylaxis, N/V, nerve injury, abscess, hematoma

How does hydroxyurea work

helps stimulate fetal hgb

What is carboprost

hemabate which can cause bronchoconstriction and pulmonary htn methergine can cause htn

What should always be on the ddx for post op apnea after a carotid surgery?

hematoma

What is the worry about anthrax

hemorrhagic mediastinitis. No need to quarantine due to no person to person transmission

For neuro IR what are some normal heparin gtt rates and ptt values

heparin IV 900 units an hour, for PTT of 70-90

What are some disadvantages of regional anesthesia

high spinal, LAST, decreased thoracic muscle strength, sympathectomy, nerve damage

What are three non traumatic cases where you can get fat emboli

hip replacement, knee, IM nail

What drugs should you avoid with tetralogy of fallot

histamine releasing drugs that may drop SVR Goal in TOF is to keep preload up to avoid RVOT spasm

What are signs of ICP preoperatively

history of HA, N/V, somnolence/altered mental status, gait disturbance Exam:cushing triad, fundoscopy Imaging: midline shift

Risk of preclampsia

history of it smoking htn pregestational diabetes twins

How do you evaluate someone's cardiac status?

history, physical, imaging The new HPI

What are the signs of anaphylaxis whats the treatment after pt is stable

hives, urticaria bronchospasm hypotension tachycardia tryptase level benadryl steroids

You place an interscalene block and the pacu nurse later calls you because she believes the pt is having a stroke, explain

horner's syndrome

Complications of a PA catheter

horners, phrenic nerve, cardiac injury, carotid injury, valve injury, PA rupture, infarct main worry is if there's LBBB and is a contraindication

What are the possible complications to a cervical block

horners, seizure, high spinal, hemiparalysis of diaphragm, epidural space.

Risk factors for placenta abruptions

htn preclampsia PROM hx of c section twins ends of age cocaine and tobacco

What is your management of croup

humidified oxygen, racemic epi,

your patient in the pacu has upper airway stridor, how will you treat it?

humidified oxygen, steroids, racemic epi. If vital signs become unstable and consciousness wavers then I'll intubate and keep ventilated overnight until edema subsides

If a pt is on steroids and has myasthenia would you give them before surgery and how much?

hydrocortisone 100mg q8 hours. Patients on steroids have chronic adrenal insufficiency.

What stays in the epidural space longer, hydrophilac or hydrophobic opiods?

hydrophilac opiods like suf and fentanyl will bind with epidural fat to stay there longer

After neuromuscular block reversal the patient continues to be weakness and respiratroy distress, whats the ddx

hypercarb, hypox, hypotension, rhythm, myasthenia gravis, cholinergic crises

For delayed emergence, the very first thing to rule out is

hypercarbia, hypoxia, malignant rhythm

Ddx for post op htn

hypercarbic, hypoxic, arrhythmia, pain, anxiety,MI, bladder overload, inadvertent medication and post op delirium.

What are the physical exam findings for a ptx

hyperesonant percussion, no breath

Side effect terbutaline

hyperglycemia, low K, pulmonary edema

What are the side effects of terbutalineW

hyperglycermia low potassium (terbutaline is sweet tasting)

Whats are the H's and T's

hyperkalemia acidosis hypothermia hypoxia hypovolemia hypoglycemia tension tamponade toxins thrombosis

What electrolyte imbalances occur with mannitol

hyperkalemia and hyponatremia

What are the side effects of bicarbonate

hypernatremia, low Ca, low K, hypotension

Whats your concern with rocuronium and a patient with a neuromuscular disorder?

hypersensitivity to rocuronium

Contraindications to plavix

hypersensitivity, active bleeding, coagulopathy

What is the acid base status of a pt with a pyloric stenosis

hypoK hypoCl met ALKALosis

How does ac cirrhotic get ascites?

hypoalbuminemia, portal hypertension, water retention due to ADH secretion.

What are some complications associated with a massive blood transfusion?

hypocalcemica, TRALI, infection, ARDS, hyperkalemia, acid/base, dilutional coagulopathy

what does pyloric stenosis place a neonate at risk of?

hypoglycemia

Causes of emergence delirium in a kid

hypoglycemia hypoxia residual gas

What kind of metabolic abnormality would you find in pyloric stenosis

hypok hypochl metabolic akalosis if not corrected progesses to acidosis.

Two major effects from removing an aortic cross clamp

hypotension and hypoxemia from the return of a bunch of desaturated blood

What are the complications of celiac plexus block

hypotension do to sympathectomy hemorrhage from vascular injury abdominal crmping and diarrhea organ injury paraplegia from artery of adamkiewicz

What is a common finding just after pheo resection and how will you manage it?

hypotension due to decreased level of catechols. Decrease anesthetic agent, stat H/H and pressors

Becks triad

hypotension, JVD, Muffled heart sounds

If a child vomits in the pacu whats your ddx

hypoventilation, hypovolemia, hypotension, residual anesthetic, hypoglycemia

Tips for burns

hypovolemia hypothermia central line need 3x as much roc renal injury from dehydration

ddx for bradycardia

hypoxia, hypercarbia, drug effects, myocardial infarct, vagal stimulation, high spinal, increased ICP

Pt becomes agitated...whats the first thing

hypoxia, hypercarbia, rhythm

What is your ddx for post op nausea

hypoxia, hypoventilation, hypotension, anxiety, pain, opioid effect.

Long bone fracture dx

hypoxia, neurologic symptoms, petechiae tx fluids, support, albumin

How does aspiration lead to low BP

hypoxic pulmonary vasoconstriction leading to right heart failure

How would you induce a patient for a congenital diaphragmatic hernia

inhalation induction maintaining spontaneous ventilation and avoid positive pressure ventilation which will compression respiratory function further

What must you always do while intubating a trauma pt

inline stabilization

Whats is bad about a preop or intraop fever

insensible losses tachy instability

How do you control insulin in an emergency surgery?

insulin gtt

What are the complications associated with the stellate ganglion

intavascular, horners, dural, phrenic nerve, recurrent laryngeal nerve palsy

Complications that may arise in a newborn with CF

intestinal obstruction, thickened meconium which may obstruct in the mid-ileum. Can result in volvulus, atresia, necrosis, perforation, and meconium peritonitis.

What are the contraindications to using indomethacin

intracranial bleeding, renal dysfunction side effects include decreased mesenteric, renal and cerebral blood flow

If you can't get an IV on a patient what is your next step?

intraosseous line and then central line

Work up for sepsis

intubate, fluids, ABG, CXR, Blood cultures

What is the induction dose of vecuronium

intubating dose is 0.1mg/kg

How would you evaluate someones asthma status

intubations, ED visits, alleving factors, exercise, triggers, how often albuterol, accessory muscle use, degree of improvement from inhaler, recent URI, cxr

What are the heart effects of digoxin

ionotropy decreased dromotropy increased vagal tone dromotropy is conduction

You have an elevated bilirubin, whats your work up

is that conjugated or unconjugated then pre intra post pre: hemolysis, reabsorption of a hematoma intra: metabolism of products, congenital defect, lfts, shock liver if there was an episode of hypotension post: RUQ ultrasound, stones, sludge, ERCP GI consult

The only time you really need PFTs

is while doing a VATS and a pneumonectomy

What is your spinal dose of bupi for a knee

isobaric bupi 0.5 3cc

What volatile agent will you use for this case?

isoflurane because its cheap and you have cardioprotective properties

What gas would you use for maintenance in a crani?

isoflurane, it has the smallest amt of neuronal activity

What is the mechanism for racemic epi

it activates alpha receptors causing vasoconstriction resulting in reduction of mucosal edema

Thoughts on plavix during a carotid endarterectomy or antiplatelet drugs

it actually might be beneficial to have these drugs on board during vascular surgery

whats nitric oxide mechanism

it binds to GC to make cGMP

Why is it a good idea to avoid opioids and bzs preoperatively while doing a crani

it can elevated PCO2 and increase ICP

How long do you wait after an MI or CVA to have surgery

it depends on the surgery

How do you handle stress dose steroids

it depends on the surgery, if minor then just the usual dose if moderate surgery then 50 mg before incision and then 25mg q8 for 24hours if major surgery then 100 mg before incision and then 50 mg q8 for 24 hours and then taper by half each day

how does HBO help with CO poisoning

it helps regenerate cytochorome oxidase

Why is mannitol contraindicated with renal and CHF patients

it increases circulating blood volume But unlike mannitol if does not increase cerebral blood volume

Why do sickle cell patients take hydroxyurea

it increases the amount of fetal hgb which does not sickle

Is pyloric stenosis an emergency

it is one of those conditions where you medically optimize first.

What is an aplastic crisis

it is where the body is unable to produce blood cells in sufficient numbers including RBCs, WBCs, and platelets.

What's the danger in lowering an SAH repair patient's BP post op

it may already be needed to treat vasospasm

Why is isoflurane a good volatile for a crani

it preconditions for an ischemic event

How does metformin work

it sensitizes the body to the effects of insulin, it decreases the production of glucose by the liver and enhances the peripheral uptake of glucose

Whats the interaction with g6pd deficiency and methylene blue

it will cause hemolysis instead give vitamin C as a treatment

What is aprotinin

its antifibrinolytic

Would you give narcan in neonatal resuscitation?

its contraindicated in opioid addicted mothers don't forget about glucose as a cause bicarb hasn't shown to have any real benefit either

What are your concerns with myotonic dystrophy

its known for prolonged contraction and relaxation with muscle wasting concerns are cardiac conduction abnormalities aspiration hypersensitivity metabolic hyperkalemia, hyperglycemia avoid succinylcholine

What a down side to cisatracurium

its metabolism is temperature dependent, so can linger with a hypothermic pt

Narrowest part of pediatric airway?

just below vocal cords

How do you do an awake foi

keep eyes on the patient the entire time and spontaneously ventilate the entire time

What are two major anesthesia post op concerns in spine/neuro surgery

keeping the patient intubated until they move all four extremities. If they can't then they may be a hematoma, edema, etc. and post op vision loss

What two induction agents can interfere with SSEPs

ketamine and etomidate

What are two premeds that you aren't normal?

ketamine and glyco

You have a patient who has a difficult airway trauma and is combative, how do you intubate?

ketamine fiberoptic

How long does mannitol last

laSIX both lasix and mannitol last 6 hours

What is your ddx for myasthenia gravis

lambert eaton, multiple sclerosis, botulism, myotonic dystrophy

Whats the beckwidth wiedermann syndrome

large tongue hypoglycemia omphalocele check heart

Contraindications to a nasal intubation

large turbinates hx epistaxis small nares

What is your ddx for croup and tx

laryngotrachobronchitis, epiglottis, foreign body. racemic epi and IV decadron.

What is the definition of a variable and late decel

late decels don't begin until 20-30 seconds after contraction

Would you switch to a single lumen tube after a DLT

less edema, less necrosis, increased mean airway pressure.

Why is ropivacaine less cardiotoxic than bupivacaine

less effects on the sodium channels

Why is biphasic defibrillation better

less joules, more target, pass through heart twice, and higher rate of conversion

What is the AVA in severe aortic stenosis

less than 1cm2, Jet velocity greater than 4m/s, mean gradient greater than 40.

What is the urine sodium in pre-renal oliguria

less than 20 meq/L

Neonate ET tube size

less than 2000g-2.5ETT 2-3kg 3.5 over 3g 4.0ETT

Where is the stellate ganglion located

level of C7 vertebrae just anterior to the transverse process, below the subclavian artery

How do you extubate a severe asthmatic

lidocaine, narc, and albuterol prior to extubation

You place an arterial line and the hand turns black, what do you do

lidocaine, regional block, call surgeon

What are the benefits of epidural fentanyl

lipophilac, less rostral spread

O2 sat drops, what do you do?

look listen feel look for bilateral chest rise, patency of ETT, lumen of ETT Listen for breath sounds Manually bag to assess compliance

Surgeries that are low and moderate risk and high risk

low eye GI dental hernia ent IR Cysto intermediate gyn uro intacranial spine High major joint bowel resection kidney transplant

Risk factors for torsades

low K, low mag, acidosis, hypoxia

What are your concerns about a pt with congenital diaphragmatic hernia

lung hypoplasia atelectasis from compression Shunting for PDA and foramen ovale PHTN Must have a preductal pulse ox

What is beckwidth weiderman syndrome

macroglossia, hypoglycemia and open stomaches like omphaloceles in newborns

ACLS for torsades

magnesium has been shown to stabilize the cardiac membrane but mechanism unknown If patient is stable give magnesium 1-2 grams if unstable

In terms of SVR compare Nitrous to volatiles

maintains it better but increases cerebral metabolic rate

What steps would you take with a pt with a latex allergy

make sure anesthesia circuit are latex free injection ports, circuit, rubber stoppers on vials remove all latex free gloves from the OR First case of the day latex cart have benadryl, decadron and epi ready

How does urokinase and streptokinase work

makes more plasmin. It helps break clot

Causes of pericardial effusion

malignancy pericarditis infection uremia injury

Three goals of cerebral aneurysm patient

managing transmural pressure, avoid hypoxia, hypercarbia

During cross clamp what are some medications that can be used to help with renal perfusion

mannitol, lasix, dopamine mannitol also scavenges oxygen free radicals

carotid hematoma question

mark it with a pen, is the pt stridorous, dyphagia, cbc and coags, difficult airway cart

What blood products would you want for a liver transplant

massive transfusion so 10u prbcs, 5-10u ffp, 10plt, 10 cryo

definition of pulmonary hypertension

mean arterial pressure greater than 25, wedge greater than 15

What are SSEPs

measures the somatosensory system from the peripheral nerves to the sensory cortex while MEP measure the stimulation of the motor cortex

Causes of a poor ACT value after giving heparin for bypass

mechanical human error pathilogical like dilution or AT3 deficiency if AT3 give argatroban or lepirudan

What are some risk factors for retinopathy of prematurity

mechanical ventilation, <32 weeks gestation, respiratory distress, hpoxia, acidosis

What is a normal hgb in men and women

men 13-15, women 12-15

whats the mechanism of metformin and glyburide

metformin-decreases glucose production by the liver. glyburide-sensitizes the pancreas to release more insulin

What condition can nitrates cause and how do you diagnose and treate it

methemoglobin which causes heme to not release oxygen tx methylene blue Since it doesn't let go PaO2 is normal but he sat is decreased sx brown urine, dark/chocolate blood, respiratory depression, shock Methylene blue works by blocking nitric oxide Nitropruss -> cyanide toxicity tx is creating methgb Nitrates will cause methgb

What are some medication triggers for G6PD deficiency

methgb can trigger it, ASA, hydralazine, hypothermia, hypoxia, hyperglycemia

What is the biggest worry in pediatrics with a foreign body aspiration?

migration of the fb, if the child is breathing with stridor the fb can easily lodge distally and further respiratory distress can occur. For premedication you can give glyco to minimize secretions

If a myasthenia patient goes into a cholinergic crisis, what will it look like

miosis, salivation, sweating, bradycardia, diarrha, incontinence, stomach pain. Tx is atropine

If there are burns on both of the patient can you still place your arterial line in one of those arms?

no, place it in the femoral

Will you extubate your burn pt after the procedure?

no, risk of sepsis, ARDS, volume overload, additional surgical procedures

Would you do shoulder surgery with just a block

no, you may have to convert so secure the airway

Whats an ideal fluid in a TURP procedure

non-toxic inert cheap isotonic transparent

What surgery can be done with a stellate ganglion block

none

Does sevoflurane or volatiles cause birth defects in 1st trimester

not as much as the process of surgery

What is TRALI and what is the mechanism

not fully understood. Possible that a transfused Ig attaches to the host neutrophils

What can you give to reverse plavix?

nothing only platelet transfusion

If the surgeon wants to use heparin for a procedure, how long does he have to wait after epidural catheter placement

nothing to 1 hour

To help avoid carcinoid crisis what drug should the pt be on preoperatively

octreotide

If you believe a pt has carcinoid crises what drug would you give them

octreotide 250-500 mcg bolus and then an infusion 100-500 mcg an hr

What is the benefits of on pump vs off pump cabg

on pump surgery is blood less and hemodynamics are easier to treat. Also avoiding the bypass machine is a huge benefit

Whats the pediatric weight rule

one year starts at 10kg and then go up 5 while counting odds

You notice the patient aspirated in the OR do you bronch?

only if you see aspiration of solid material and don't lavage either

Medications safe to place in an arterial line

opioids, succ, atropine, NMBDS, versed

What meds can you put through an art line

opioids, succ, nmbds, versed

What are the causes of overdamp and underdamp

overdamp-narrow/compliant tubing underdamp-increased SVR, hypothermia, tachy

Everytime you get a blood gas what should you calculate

oxygenation, acidosis, ventilation, Pa/FiO2 ratio

How does oxytocin, hemabate, and methergine work

oxytocin-increases intracellular calcium side effects: tachy, hypotension methergine-acts on serotonin and alpha 1 recs hemabate or carboprost: binds prostag E2 leading to smooth muscle contraction

Plasmalyte vs LR

pH of LR is 6.5 and of plasmalyte is 7.4. Plasmalyte has magnesium LR has calcium large amount of LR can lead to alkalosis

pH stat vs alpha stat

pH stat is temperature corrected with the goal of paCO2 40 and pH 7.4. It has increased cerebral blood flow is used in pediatric cases. Embolic load is higher alpha stat is not temperature corrected and better for adults and maintains better autoregulation

The 4 side effects of etomidate

pain on injection N/V Seizure adrenal

If you see a laparotomy what should you think about

pain, regional difficult extubation/can't take a good breath Vent settings prolly not SIMV

What are the signs of pyloric stenosis, how do you diagnose it

palpable olive mass with visible peristalsis, male, non-bilious projectile vomiting. Definitive diagnosis is abdominal ultrasound.

After an aneurysm repair, whats 3 big worries about applying an aortic cross clamp?

paraplegia, hemorrhagic stroke, myocardial infarction

For ICP what may happen if you hypervent the patient for too long

paroxysmal cerebral injury

What are the preop considers to myasthenia gravis

past intubations, current condition of disease, medications, prolonged intubations, muscle groups effected

During pheo manipulation the pt becomes hypertensive, what do you do

patient awake, hypercarbic, hypoxia, malignant rhythm. Start nitro, nicardipine or esmolol.

DDx for ascites

pbc, hepatitis, cirrhosis

Name some medications that can cause drug induced fever

pcn, cephalosporins, vanc thiazides, lasix, hydralazine

How does zofran work

peripherally it reduces the sensitivity of the vagus nerve and centrally it works in the chemo trigger zone of the midbrain

Why can't we use IM morphine

pharmacokinetics are variable and we are unable to predict the dose

Contras to a liver replacement

phtn, severe CAD, AIDS, malignant dz with mets

Whats your drug of choice for treating anti-cholinergic crisis for example in myasthenia

physostigmine 1.25mg IV because it crosses the BBB

What are the contraindications to caudal

pilonidal cyst, infection coag anomalies of spine

You have a mucus plug thats not resolving with treatment whats the next step

place a double lumen in and lavage it

What kind of monitors would you place in a cardiac pt undergoing non-cardiac surgery?

place an ECHO PROBE!

If a patient desats during a TEF repair whats your next step

place on 100% O2, confirm ETT placement by auscultation, listen over the epigastric area for air leak, suction ETT, notify surgeon, manually ventilate to assess compliance and provide gentle atelectasis

What is the biggest complication of a congenital diaphragmatic hernia surgery intraop

pneumothorax of the contralateral lung

What are your preoperative concerns in a IVDU mother preggo

poor access multiple drug abuse withdrawal syndrome, systemic derrangement poor prenatal care that predisposes to higher peripartum complications htn and cocaine use predispose to abruption for the fetus id be concerned about IUGR, intraventricular hemorrhage, congenital abnormalities, low birthweight fetal dependence and withdrawal syndrome

With neonate cases always think of two things

post op apnea and retinopathy of prematurity

What is the risk of permissive hypotension in a spine case

post op blindness

Spinal vs epidural

post op pain less systemic side effects spinal is denser block which is bad if using for thorax post op pain relief

Benefits of epidural

post op pain control improved respiratory mechanics lower incidence of DVTs, improved GI function

If you have a tetralogy of fallot patient list all of the drugs you want to avoid

ppv, volatile gas on induction as it drops SVR, avoid propofol as it drops SVR, avoid succinylcholine and histamine

What monitors would you want for a TEF repair

pre and post ductal pulse oximeter, a preductal arterial line, precordial stethoscope

What kind of monitors would you want for a carcinoid pt

pre induction arterial line and a central line

Whats the main cause of emergence delirium

pre op anxiety

What are the numbers for bun/cr for prerenal, post and intra

pre renal >20 intra < 10 post 10-20 or normal

Post op oliguria work up

pre-renal: fluid challenge, use monitors Intrinsic: OR drugs, check record Post: flush foley, bladder scan don't forget post op oliguria in a cabg the patient likely got diuretics

What is the ddx for a htn in a preggo mother

preclampsia drug abuse gestational htn chronic htn

What are you looking for preop with a hyperthyroid patient

preop Hypertension, tachycardia, tremor, anxiety, fever, heat intolerance, weight loss, exophthalmos, myopathy, atrial fibrillation/CAD/CHF, all suggest poor preoperative control. check labs because they should be euthyroid check difficult airway due to edema or recurrent laryngeal nerve palsy from prior surgery In both hypo and hyper thyroid rule out a goiter as it is a cause of respiratory difficulties more than a difficult airway. CXR is warranted to look at tracheal deviation Intraop avoid agents that are sympathomimetic like ephedrine and ketamine post op beware that thyroid storm can still occur

Preop considerations for a pheochromocytoma patient

preop medications vital signs, orthostatic BP, ekg these patients have chronic hypovolemia

Post op oliguria

prerenal: fluid challenge if no comorbidities, optimize BP Intra check chart for antibiotics (vanc), contrast and cease them Post renal: think ureter/plumbing as the ureter might have been cut

Anytime you have a restrictive lung picture, what vent setting should you have?

pressure control

What is the mechanism of dexamethasone

prevents neutrophil migration, decreased production of inflammatory mediators, suppresses immune response metabolized by liver

What is your ddx for third trimester bleeding

previa and abruption, other causes are polyps, carcinoma, vasa previa, genital tract lesions, abuse

Contraindications to cabg fast tracking

prexisting respiratory compromise large left to right shunts phtn prolonged bypass time unstable pts low EF

Hypermagnesemia

prolonged QT, widen QRS, prolong PR

What does your TEG look like when the patient is on anti-coagulants

prolonged R prolonged K decreased alpha decreased MA

What are the risk factors fo uterine atony

prolonged labor, induction, twins, polyhydramnios, macrosomia, twins, chorioamnitis, retained placenta

What is the mechnanism of amiodarone

prolongs cardiac repolarization, blocks sodium channels, beta blocker, and calcium channel blockade

Whats your treatment for tight dura

propofol bolus head of bed 30 degrees hyperventilate diuretics decrease blood pressure spinal drainage paralysis to increase venous return

What does K centra stand for

prothrombin complex concentrate

What are the side effects from epidural opioids

pruritis, N/V, urinary retention, respiratory depression

BP drops to 60/30 and peak pressure jumps, whats your ddx

ptx, anaphylaxis, bronchospasm, endobronchial intubation, kinked ett

How to perform a pudendal block and paracervical

pudendal: inject behind both sacrospinous ligaments transvaginally for S2-4 coverage for phase 2 of labor for paracervical block inject into the fornix of the vagina paracervically

How does PTT work

pull blood centrifuge draw plasma add calcium and activator and visible see how long it takes for the blood to clot

Tell me your concerns with a down syndrome patient

pulm/airway: macroglossia, subglottic stenosis, leading to a difficult airway, atlantoaxial subluxation so neutral neck position. Obstructive airway disease, more prone to snoring, apnea GI: duodenal atresia -> full stomach cardiac: baseline echo Neuro: aspiration risk, increase risk of seizure and spasms

What is your work up and ddx for blood in the ET tube

pulmonary contusion, drop a bronchoscope and look for a bleed, if you see one place a bronchial blocker and call CT surgery

Ddx for decreased compliance

pulmonary edema, effusion, CHF, mucus plugging, ptx, aspiration

What are some effects of scoliosis in a pt

pulmonary hypertension and right ventricular hypertrophy on exam look for signs of right heart failure Pre op labs should include an echo, coags, ABG, cxr These pts require electrofizz monitoring

Side effects of retrobulbar block

puncture of posterior globe-pain central retinal artery occlusion which is painless stim of occular cardiac reflex allergy hemorrhage side effects of peri contralateral eye spread ecchymoses transient blindness

Why not perform a caudal in a pregnant woman

puts the fetal head in close proximity and there's actual risk of injecting local anesthetic into the fetus

A child has non-bilious projectile vomiting, whats your ddx

pyloric stenosis, achalasia of esophagus, duodenal atresia, malrotation, hernia, meckels diverticulum (a remnant of the umbilical cord

What are some disadvantages of HFJV

rapid increase in airway pressure reliance on surgeon for ventilation No ETCO2, PAWP, No control over FIO2

What are some risk factors for emergence delirium in a child

rapid wake up pain pre op anxiety age 2-5

Tell me about symptoms of mitral prolapse and intraoperative management

rarely have symptoms, can have regurgi-chest pain and arrhythmias tx: avoid hypovolemia, and excessive contraction, and decrease afterload.

What is the first step to atrial fibrillation?

rate control them if stable with dilt, esmolol

What is a blood gas coefficient and list them for the volatile anesthestics. How does the blood gas coefficient correlate with MAC value

ratio of equilibrium of the gas in blood to vapor. The higher the number the more soluble and potent the anesthetic is. The higher blood gas coefficient the low the MAC value. The more lipophilac the more potent but slower the onset

how does protamine and heparin

react to form an inactive salt

What are some of the long term effects for a congenital diaphragmatic hernia repair

reactive airway disease, obstructie lung disease, chest wall deformities

acute febrile transfusion reaction

recipient antibody reaction

Whats the best way to place a gastrostomy tube

rectus sheath/tap block

What are some complications associated with post op PDA ligation

recurrent laryngeal nerve paralysis, chest wall deformities, scoliosis, rib fusions

What is your primary treatment for fat emboli

reduce the fracture

What causes mitral valve prolapse

redundant tissue or myxomatous tissue (connective tissue embedded in mucus) degeneration of valve leaflets

What anesthesia technique should you avoid with a HOCM patient

regional, a drop in preload could be death

For a carotid endarterectomy would you perform a general or regional technique for this case?

regional, the best monitor for neuro status is an awake pt. The only time this won't fly is if the patient is agitated and unable to lie still

A patient has GERD, what two preop medications can you give?

reglan and bicitra

What are some ways to minimize PONV through your case

reglan, decadron, zofran, minimize opioids, regional instead of general, fluid hydration, avoid etomidate and nitrous

ASA 3 category

regular dialysis, greater than 3 months on MI, CVA, TIA, or CAD/Stents

What are your hemodynamic goals for mitral regurg

relative tachycardia and reduced afterload and preload increased Up, UP, Down

How do you induce a trauma patient with a c collar

remove it and have someone hold in line stabiliztion, difficult airway cart in room, and ENT ready to cric

Major complications from AAA repair

renal MI Distal limb ischemia from emboli spinal cord ischemia

What are some concerns about placing a cross clamp

renal perfusion, heart load spinal cord perfusion

You are unable to intubate an epiglottis patient....and now mask ventilation is becoming more difficult

reposition the patient, change blade, downside tube, alert ENT for possible airway, transtracheal jet ventilation

With a mediastinal mass, what two surgical procedures can be performed

resection or mediastinoscopy for bx

What pulmonary changes do you see with obesity?

restrictive changes

What kind of information can you get from PFTS

restrictive or obstructive pattern You can use VO2max, DLCO, and FEV1 for extubation criteria See if the patient responds to bronchodilators

What is your next step if sat is 83%

resume two lung ventilation and clamp the pulmonary artery of the surgical lung

Risk of pudendal block

retroperitoneal hematomas and intravascular injection

What techniques can you use to prevent hypertension on extubation of a crani patient

reverse paralytics after fully bandaged labetolol lidocaine 1.5 mg/kg

What's the major worry of a mitral stenosis patient?

right heart failure

What is your setup for an anterior mediastinal mass

rigid bronchoscope in room, surgeon, ent, perfusionist in the room, microlaryngeal tubes, endobronchial tubes, and size down tubes, cannulate the femoral artery and vein to crash bypass if need be.

What is the procedure of choice for pediatric fb retrieval?

rigid bronchoscopy, if unable to obtain, a thoracotomy may need to be performed

Why don't we give ace inhibitors preoperatively in anesthesia

risk of hypotension. The risk of death, myocardial injury, and stroke are all decreased when ace inhibitors are stopped 24 hours before procedure.

Pyloric stenosis case, how do you induce

roc RSI

What kind of induction is pyloric stenosis

roc RSI after decompressing with an OGT prior to preoxygenation and pretreat with atropine 0.02mg/kg

As surgery starts you notice changes in SSEP, what would you?

rule out h/h/mrhythm, and be sure to have a MAC of gas less than 0.5. Then rule out profuse bleeding/anemia

Post op carotid bp is 180/110, will you treat it

rule out other cause such as pain, anxiety and then titrate something short acting like nitroglycerine

A patient is oliguric, whats the first thing you say

rule out rhythm, hypoxia, hypercarbia

For orals whats a better way of saying i'm uncomfortable?

say I'm concerned. For example i'm concerned that the patient would be at risk for reintubation

What kind of peripheral nerve injury can you get from sitting crani

sciatic nerve

Side effect of scopalamine

sedating and can cause tachy

How would you managing preop wheeze

see pt treat, O2 reassess cxr abg depending on therapy response determines proceeding with elective case

What are your post op concerns for a SAH pt

seizures rebleeding vasospasm days 3-15 hydrocephalus

Why keep someone sedated while intubated ? especially a child?

self extubation

How to diagnose TRALI and how long does it last

self limiting, 3-4 days. fever, hypoxia, hypotension

What is your management for digoxin toxicity

send a dig level get digibind and it your don't have that use magnesium

Indications for aortic balloon pump and contraindications

severe LV dysfunction, unable to come away from bypass, cardiogenic shock, bridge to transplant Contraindications: severe peripheral vascular disease severe AI

What is a myasthenic crisis?

severe bulbar or respiratory symptoms. Occurs by infection, surgery, emotional stress, drugs like antibiotics, ketamine, hypothermia, acidosis, hypokalemia. Drugs that increase risk of myasthenic crisis are: ketamine, local anestheics, aminoglycosies, clindamycin. Ester local anesthetics.

What is causalgia

severe burning pain caused by a nerve injury similar to CRPS2

When would you avoid using normovolemic hemodilution

severe cardiac disease as anemia will create myocardial work and increase demand . Avoid in renal patients due to incease volume load. This may then lead to electrolyte abnormalities Lastly avoid with a low baseine hgb already

Is this severe or mild myasthenia gravis?

severe is greater than 750mg and mild is below that

Contraindication to balloon pump

severe peripheral vascular dz, AI, aortic dissection, severe neuro brain damage

Whats bad about not taking clonidine

severe rebound hypotension

EKG changes hypercalcemia

shortened QT Widen QRS

Indicators of the severity of an epidural hematoma

size, shift absence of corneal reflex age over 70 hydrocephalus

What is the negative sequelae to intraoperative awareness

sleep disturbance, night mares, difficulties at work those at risk gyn, cardiac, regional

What are the symptoms of superior laryngeal nerve and recurrent laryngeal nerve palsy

sln single: minimal sln b/l: hoarseness rln single: stridor, hoarseness rln b/l: aphonia

How do you diagnose WPW and how do you treat it

slurred QRS upstroke or delta wave The QRS can be narrow or wide if wide there's retrograde flow the AV node if narrow there's anterograde flow through the AV node Procainamide 10mg/kg for antidromic over 10 minutes Adenosine for anterograde

How do you do an exam on an uncooperative kid

small jaw, small mouth, overbite

What are the symptoms of epiglottis

sore throat, drooling, fever, dypsnea, dysphonia. high fever 104

A sickle cell pt starts to having abdominal pain...whats the ddx

splenic sequestration where sickle rbcs are stuck in the spleen tx is transfusion mesenteric thrombosis

How will you maintain anesthesia in a foreign body retrieval

spontaneous inhalational to tiva

What is Von Willebrand disease

stabilizes factor 8 and helps with platelet aggregation

Paracervical is good for which stage and what are the side effects

stage 1 labor fetal brady, uteroplacental insufficiency-not good for preclamptics

Stages of labor

stage 1 pain is usually T10-L1 stage 2 is S2-S4

The patient is confused, what's the ddx

start with the basics, hypercarbia, hypoxia, hypotension

What are the relative strengths of steroids compared to hydrocortisone and what is hydrocortisone and why is it the steroid of choice

steroid of choice: short acting and has a good amount of mineralcorticoid activity. It is the synthetic version of cortisol Strength decadron 25:1 Prednosone 4:1 Methylprednisolone 5:1 If you have to give over 100 mg of hydrocortisone consider methylprednisolone as it has less mineralcort activity side effects of these things are low K and sodium

How does procrit work?

stimulates bone marrow to make more

How does a TENS unit work

stimulation that releases endogenous opioids, and inhibition of presynaptic terminals

MH treatment

stop offending agent call for help MH cart, dantrolene 2.5 mg/kg bolus every 5 minutes until symptoms abate flush circuit with 10 liters of O2, switch to TIVA Hyperventilate central line arterial line remove CO2 cannister, vaporizers and circuit cooling measures sodium bicarb 1-2meq/kg lasix 1mg/kg or mannitol .25g/kg for urine output 2ml/kg/hr Insulin 10 units in D50 in 50ml Foley Draw ABG every 15 minutes and electrolytes from art line procainamide 200 mg to treat arrhythmias

Whats your work up tx to oculocardiac reflex

stop offending agent lido to rectus sheath atropine epi

What is the treatment of TURP syndrome

stop offending agent normal saline ABG for electrolytes diuretics 3% NS if less than 120 too fast leads to central pontine myelinoysis which is damage to the myelin of the pons

Which antibiotics do you want to avoid in a patient with myasthenia gravis

streptomycin, polymyxins, fluoroquinolones, tetracycline as they do have some NMJ depression and may activate myasthenia

What some early complications from TEF repair

strictures, anastomotic leaks

Post op CABG, mediastinal drains stop, whats your next step

strip drains and go back to OR

During aortic cross clamping there's decrease perfusion to the spinal cord, will hypothermia help?

studies have not shown this to be true

What kind of central line would you place in a trauma

subclav or femoral as the patient could have C spine issues

What is the difference between epidural and subdural hematoma

subdural is venous, concave and immediate. Epidural has a period of lucidity, biconvex, arterial

Whats the mechanism for epidural opioids

substantia gelinatinosa dorsal horn for spinal and dorsal roots for epidural

best paralytic for liver transplant case

succ for induction and cisatra for maintenance. No nitrous as it causes bowel distension

What is the best anesthesia method for resection of a thyroidectomy

superficial and deep cervical plexus blocks

Best block for elbow surgery

supraclavicular

What are the 3 layers to a epidural

supraspinal, interspinous, ligamantum flavum

What is your next step in a bleeding airway/trauma in a patient who isn't intubated

surgical airway

What are your techniques for avoiding fire in airway

surround patient with wetpack FiO2 down methylene blue in ETT intermittent apnea

How does oxytocin work?

synthetic stuff can cause hypotension while the natural stuff causes htn works by activing g protein coupled receptors that leads to calcium release Hemabate works as it is a prostaglandin and will increase smooth muscle contraction Ergots have serotonin and alpha 1 activity

You start a nitroglycerine gtt and the patient develops tachycardia, whats going on?

tachy is a side effect of nitro

What are some signs of digoxin toxicity

tachyarrhythmias, bradyarrhythmias, sinus brady, AV block, PVCs, N/V, diarrhea, blurred vision, yellow/green discoloration, palpitations, syncope, dyspnea, confusion, dizziness delirium fatigue PVCs are the most common and earliest signs of toxicity

What do you do if a child aspirates during induction

turn the head to the side, suction and trendelenburg, intubate, suction thru catheter.

What are some risk factors for preclampsia

twins hx of it BMI Diabetes first birth chronic htn

what is alpha stat and pH stat

two different methods for managing pH during hypothermia

Digoxin toxcity treatment and risk factors

tx digibind, and magnesium risk factors renal failure because its excreted in the kidneys unchanged RFs: glucose, low K, low Mag, high Ca

What the one lab you should ask for that doesn't seem like a lab?

type and cross

What can PFTs tell you in general

type and severity of the disease (obstructive vs restrictive), present of a reversible component, baseline function Specifically Cardiopulmonary reserve VO2max 15ml/kg/min ppoFEV1 you want greater than 30% DLCO-Diffusion capacity of CO > 40%

Will you use an ultrasound or a nerve stimulator for your block

ultrasound, neither is better than the other

What are the causes of metabolic acidosis

uremia, keto lactic ethylene, salicylates, methanol, are all causes of gap

Post op oliguria electrolytes, pre-renal vs intra renal, urine osmolality

urine osm >500 in pre-renal and less than 350 in intrarenal when thinking about oliguria electrolytes, place sodium and water together. Osmolytes are a bunch of other kidney shit

What medical conditions are associated with latex allergy

urogen-spina bifida allergy to banas kiwi. pediatric surgery patients

How can you mitigate the increase in ICP from using succinylcholine?

use defasciculating dose

Why perform a caudal over lumbar epidural

used for analgesia below umbilicus can be used for urgent procedures like a hernia in the young chronic pain implications

After you have intubated an epiglottis patient, when are you good to extubate this patient

usually 24-48 hours and the criteria will be normal temperature after appropriate antibiotics, air leak, decreased edema and erythema

How long after a stroke can you not receive succ

usually 3-6 months

What are your cardiac concerns in a cirrhotic

usually high output but low SVR meaning low perfusion to the kidneys and coagulopathy

Do you extubate a stroke patient, do you extubate a crani for a mass?

usually not for a stroke, but yes for a mass

ddx for intraoperative bradycardia

vagal, insufflation hypoxia hypercarbia drug effets high spinal ICP

When a patient has diabetes what are three factors that come to mind

vasculopathy, neuropathy, nephropathy

What are the pharmacodynamic effects of magnesium

vasodilation-hypotension anticonvulsant sedative tocolytic

After SAH repair, what are your main concerns

vasospasm, rebleeding. 1/3 of people will get vasospasm

What are a few ways that cerebral aneurysm patients die

vasospasm/infarct, initial bleed, rebleed, edema-> herniation, hydrocephalus

What is ddx for random bradycardia in OR

vasovagal, orthostasis, bezold jarisch, carotid sinus sensitivity

What are your induction goals for a TEF case

ventilating the lungs without ventilating the fistula and creating gastric distention therefore we would perform an inhalation induction and place the ETT down the right mainstem and pull back until bilateral breath sounds are head. The goal is to have the ETT distal to the fistula but proximal to the carina.

What are the indications for digoxin

ventricular rate control of patients with afib/flutter, treatment of PSVT, CHF.

If vasospasm were to occur what would be the next steps

ventriculostomy (burr hole) and mannitol to lower ICP. Then surgery if doesn't resolve

What premedications would you give to an OSA patient

verbal reassurance, then maybe small dose of ketamine or versed and some reglan.

You just placed a interscalene block and the pt seizes, why?

vertebral artery injection

causes of low calcium

vitamin D and parathyroid hormone

When dropping a TEE for low bp what do you look for

volume status with IVC, effusion, regional wall abnormalities with multiple views, valvulopathy.

How would eval a pt for pheo resection

volume status, cardiac eval, medications-alpha and beta blockers. It should always be alpha followed by beta antagonists Cardiac eval: mets, sob/cp, ekg, echo, stress test

Aside from the obvious difficult airway, what is your other worry about a child with epiglottis

volume status: drooling, not eating etc

What are some conditions that are assoc with pheo

von hippel lindau, von recklinghaus aka neurofibromatosis, MEN

Whats the ACLS for neonatal resus

warmth dry suction stimulate reposition If HR > 100 but cyanotic then supp O2 If HR < 100 but apnea, PPV If HR < 60 compressions first and epinephrine 10mcg per and via ET tube is 100mcg per

Firt step to a needle stick

wash with soap and water

You have pt with a bowel rupture, and they starting getting a fever

whats your ddx they're getting septic!!

down sides to albumin

when given in large amounts it can create more of an acidosis than NS Expensive associated with histameine release Bad for TBI

What is cerebral hyperperfusion syndrome

when previously underperfused areas of the brain lose the ability to autoregulate, it can lead to headache, seizure, edema

When do you use a wake up test

when sseps, meps are not looking good and your conservative measures aren't working.

When do you investigate more about PVCs?

when there's more than 6 a minute If you see 3 in a row and the heart rate is greater than 120 this is technically vtach, give lidocaine 1-2mg/kg and investigate cause

With a renal transplant whats something you should always think of preop

when was the last dialysis

SAH ekg changes

wide qrs, t wave inversion, long QT

What is the best way to treat hypotension with severe aortic regurgitation

with increase in heart rate and contractility. An increase in afterload will just increase the regurg fraction

Trauma patient, how do you induce this pt

with inline stabilization

You have a carotid hematoma going back to the OR, how will you induce

with ketamine

How do you perform a cuff leak

with the balloon up take the the three lowest expiratory volumes and average them, then drop the cuff and do the same. If there's a difference of 10% then you can extubate

While placing an epidural catheter you see clear fluid come back into the catheter what do you do

withdraw it and place one level higher

Does PA pressure increase with tamponade?

yes

Is a spinal cord injury a contraindication to a neuraxial technique

yes

At the end of a case an MH child is having laryngospasm, would you give sux

yes and treat MH

Will you give this neonate a glucose containing solution

yes as neonates have decreased glycogen stores and are proneto hypoglycemia. Maintenance fluids will be D51/2NS at 4ml/kg/hr. Third space losses replaced with 8ml/kg/hr.

Would you pre-medication myasthenia patients with bzs?

yes because If their symptoms were only occular then yes. This is because myasthenia can be exacerbated with stress.

Is intraoperative heparin allowed after placing a catheter?

yes but lovenox is not

Pt is allergic fo PCN can you give them cephalosporins

yes if not an anaphylactic reaction. There is a 10% cross over. I'd play it safe and go clindamycin and gentamcin

If a preggo is on methadone should they be kept on it?

yes, only thing to be careful of is fetal withdrawal syndrome

Why not cannulate the ulna artery?

you can but beware if the radial was stenosed and now you going to somewhat occlude the ulna

While the OB doc is attempting a delivery in delivery she tells you she needs more relaxation

you can give nitroglycerine spray or IV

What is the main advantage of an epidural vs spinal

you can titrate it

You have a ruptured triple A bleeding like crazy, what do you think about putting him on bypass

you can't, you'd have to give a large heparin dose

What is the treatment for CSWS and SIADH

you fluid restrict with SIADH-possible diuretics and you replete with CSWS with isotonic fluids

What values of a predicted post op DLCO do you worry about on PFTs for a lobectomy or pneumonectomy

you want greater than 40%

What is the usual dose of mannitol

.25-1g/kg

What your dose of pediatric succinylcholine dose and IM doseIf

.25mg/kg IV and 4-6 IM

What is a good infusion rate for rib fractures with 0.125% bupivacaine?

1ml/kg/hr

Whats a dose for an esmolol bolus and whats the dose for an esmolol gtt

0.2 mg/kg and 50-200mcg/kg/min

What value of MAC do you need to keep your volatile agent so it doesn't affect SSEP

0.5

For pediatric cardioversion whats the dose of joules?

0.5-1 joule/kg

If you do a regional technique in an obese patient what do you avoid?

I would avoid narcotic

Treatment for VAE

Lower the head of bed, jugular compression, aspirate RA, flood field, 100% O2, pressors

3y/o going for an MRI head, what kind of anesthesia would you use

MAC if cooperative and general if not

What are you MAP goals in aortic cross clamp for a dissection

MAP of 100 above clamp and 50 below

In TE Fistula repair do you use paralytic at all?

Once the fistula track is ligated

Signs of opioid w/d

fever, lacrimation, N/V, restlessness, agitation

In TEF repair what is your ddx for hypoxia

tube migration is the most likely

Why did the ED doctor stop mag on a preggo car crash

turn it on as a tocolytic

Key points about triple A repair

-Apply a shunt to help with spinal cord ischemia -keep the patient cool distally so no lower bear hugger, -give mannitol and run dopamine before clamp is applied -patient will need to be heparinized so place epidural preop and don't use it until after clamp is released so you don't get a double hit of hypotension -mortalitiy of elective AAA 0.5%, emergent 50% -place a TEE -If thoracic AAA place a DLT**

Why do patients with congenital diaphragmatic hernia develop respiratory disease and cyanosis?

-Atelectasis from compression from the abdomen -Pulmonary hypoplasia -Persistent pulmonary htn -shunting thru PDA and foramen ovale

When doing ambulatory surgery for ex a hernia, what are your thoughts on an epidural or spinal technique

-Discuss against this as it will delay discharge and the OR schedule Post op urination is a huge problem with hernia surgery so be sure they urinate before leaving. These patients must be able to walk or crutches before discharges

What are some considerations for patients with hemophilia

-Surgery should take place at a center with recombinant factor 8 -considering an infusion of ddavp to increase factor 8 -Normal replacement level is 100IU/dl -How much factor 8 would you give in an emergency? desired level ie 100 IU/dl * 0.5 * body weight kg, then give this at 100units/min -Half life is 8-12 hours so must be administered twice daily afterwards -Avoid IM and arterial sticks -Avoid succ because fasciculations can lead to hemorrhagic joints -Avoid nasal intubation due to mucosal hemorrhages

Digoxin toxicity symptoms

-Yellow tint -Hyperkalemia is the first sign -afib/flutter/AV block -hypotension -PVCs and sinus brady

Differences between gastrochisis and omphalocele

-omphalocele is located within the umbilical cord where gastrochisis is to the right of it -omphalo has a membranous sac covering it and gcheesis does not -omphalocele is associated with conditions like downs syndrome, and congenital heart disease

When do you use a reinforced tube

-when reintubating as they're less likely to melt -when there's potential for kinking because they're more flexible -Head and neck surgery if the neck is contracted -Tubes in stomas -Nasal intubations that don't have raes prone cases down side is they don't bend back so can't have patient biting

How do you replete magnesium and phosphate

.05g/kg, .15mmol/kg

What is the dose window for digoxin, what conditions increase the risk of digoxin toxicity

0.5-2.5ng/ml hypoxia, renal disease, hypothyroidism, hypokalemia, hypomagnesemia, hypercalcemia, hypoglycemia,

Pediatric adenosine bolus

.1mg/kg don't exceed 6mg then .2 mg/kg

Whats the pediatric epi dose

.1ml/kg of a 1:10,000 concentration box IV .1ml/kg of a 1:1000 box ET amiodarone 5mg/kg lidocaine 1mg/kg

Preop pheo concerns

1. 2 weeks alpha 1 blocker 2. optimized? no st changes on EKG, BP less than 165 3. BBs ok if already on an alpha 4. Hydrate because they're volume down 5. Epidural for post pain 6. Etomidate use, avoid succ and propofol 7. Sevo is the choice 8. Place TEE 9. Uppers and downers ready Intra op 1. Tumor removal low bp and low blood sugar Post 1. low bp and high bp

Tell me 3 automatic points or things you think of when you have a sickle cell patient

1. A line and central line 2. Regional techniques do not increase risk of crises 3. Chronic anemia as a sickled rbc half life is 10 days.

What monitors would you want in the OR for a sickle cell patient for example an appendectomy?

1. ASA 2. Arterial line for serial Hct and pH sampling to avoid acidosis 3. Central line to assess volume status, give blood products, and keep the patient euvolemic 4. Foley catheter 5. Blood products on standby if the anticipated EBL was minimal

When you hear laryngeal papillomatosis or laryngeal papillomas what do you think?

1. Airway obstruction 2. source of cor pulmonale and RVH

What are your concerns about hypothermia

1. Airway-goiter, large tongue, gerd 2. Poor hemodynamics 3. Adrenal insuff-give steroids 4. Pericardial effusions 5. Avoid propofol, cardiac down already Only delay a case for severe hypothyroidsm

What are some key aspects in shoulder surgery in the beach chair position

1. Alters CPP as the cuff no longer accurately represents cerebral perfusion 2. Venous pooling due to gravity, so the patient needs to maintain well hydrated

Name 5 key concepts when doing anesthesia for anterior mediastinal masses

1. Always have the surgeon or ENT present on induction. 2. Have a rigid bronchoscope available on induction 3. Always maintain spontaneous ventilation 4. Avoid general anesthesia 5. Never rely on cardiopulmonary bypass standbyd

What are your concerns with a pyloric stenosis patient post op

1. Aspiration 2. Croup: post op extubation croup is always a worry in this age group 3. Post op apnea, as these kids are at risk due to their alkalotic state 4. Post op hypoglycemia due to inadequate glycogen stores

How would you induce a pt with carcinoid syndrome?

1. Avoid medications that are histamine and catecholamine releasing 2. Avoid hypotension as it's a trigger 3. Then use fentanyl, etomidate, roc

The 4 major post op complications from liver transplant

1. Bleeding 2. Bile leak-ultrasound, check drains, maybe stent 3. Graft rejection-Increasing LFTs, immune suppress 4. Hepatic artery or vein thrombosis-doppler, surgical re-explore

What are three major concerns with carcinoid syndrome

1. Bronchospasm 2. RV failure 3. Carcinoid syndrome

What is your differential for pulmonary edema during a case

1. CHF 2. Fluid shift 3. TRALI 4. ARDS 5. Fluid overload

What is your acute treatment of hyperkalemia

1. Calcium Cl 500 mg or Calcium gluconate 1 gram over 3 minutes and repeat every 10 minutes as needed or until you see EKG changes back 2. 10 U of regular insulin bolus, d50 bolus followed by d50 infusion in 500 of D20% infused over 1 hour. 3. albuterol 20 mg in 4 ml nebulized 4. lasix 40-80 mg

What is your ddx for post op oliguria in a renal transplant? How do you work it up

1. Categorize at prenal, intra, and post Pre renal due to: usually hypovolemia. Check inputs and outputs, check CVP readings, and send for electrolytes to assess dehydration like hypernatremia. Physical exam for skin turgor, membranes 2. Post renal: kinked foley, bladder obstruction. 3. Intra-renal nephrotoxic substances: urinalysis with casts would confirm this diagnosis. 4. Graft rejection may be occurring, vessel occlusion

How to place and confirm DLT?

1. Check both balloons 2. Lubricate the inside and outside of the tube and place stylet down bronchial lumen 3. Place through the cords and rotate 90 degrees to the left. 4. Look down the tracheal lumen and see the carina and the bronchial balloon. 5. The right upper lung should display 3 orifices (right upper/middle/lower bronchi), and the left upper and lower bronchi. 6. Inflate tracheal balloon and verify b/l lung inflation 7. Inflate bronchial balloon and connect both to the circuit and inflate to verify the endobronchial balloon is not obstructing the carina. 8. Clamp each tracheal lumen and verify unilateral lung movement.

If the surgeon insufflates and the blood pressure drops what would be your next step?

1. Check monitors and make sure oxygenating, ventilating, and sinus rhythm 2. open fluids, temporarize with neo 3. Look at ppv, cvp to see if patient is volume depleted 4. Change to a pressure controlled mode at a lower pressure 4. If these monitors didn't work then desufflate abdomen

What is the treatment for pulmonary edema in a renal patient?

1. Dialysis 2. Restrict fluid 3. Peep

Tell me 5 key points about burn patients

1. Difficult airway-facial injury 2. Inahalation injury -can causes CO poisoning/cyanide, V/Q mismatch pulmonary hypertension 3. Hypovolemia, possible shock -need a pre-induction arterial line 4. They're a full stomach (curling ulcers) 5. Sepsis risk due to the skin disruption 6. Hypothermia (which causes metabolic acidosis)

Name the different kinds of stress tests

1. Dypyramidole scintigraphy 2. Dobutamine echo (if unable to exercise) 3. Treadmill EKG

Name the three biggest worries of preclampsia

1. Edematous and difficult airway 2. CNS altered/cerebral edema 3. HELLP syndrome If a patient has altered mental state with preclampsia, intubate them as they now have become an aspiration risk.

What are some major issues ATN causes while performing anesthesia

1. Electrolyte abnormalities 2. Can't assess adequacy of resuscitation 3. Drug metabolism and clearance

Name some malpositioning errors with double lumen tubes

1. Endobronchial cuff too deep which makes diminshed breath sounds. 2. Position into the opposite bronchus which may make the opposite lung collapse. 3. Underinsertion of the DLT where the endobronchial cuff is at the carina

What are the factors that determine survivability of a burn patient?

1. Size 2. Extent-full vs partial thickness 3. Mental status change requiring mechanical ventilation 4. Sepsis 5. Fluid loss leading to renal dysfunction

Criteria to postpone a case due to a URI

1. Fever 38.5 2. Purulent nasal discharge and wet cough 3. Systemic sx, not eating, not drinking 4. Wheezing that doesn't clear with coughing. History of a URI that should be canceled not able to eat drink go to school, high fevers, cbc white count and neutrophils

Give me 4 ways to check volume status in the OR

1. Foley 2. Pulse pressure variation 3. Area of the curve of an arterial line 4. Heart cavities diameter on TEE.

What is your work up for an airway fire

1. Gas off 2. Pull tube out 3. Pour saline into the airway 4. Cxr and bronchoscopy to determine extent of injury 5. Reintubate with smaller size ETT 6. Give steroids 7. Give racemic epi

Delay emergence/disorientation work up

1. Go to bed side, make sure patient is oxygenating, ventilating and vital signs are stable. 2. Next a focused H&P, review OR and PACU notes. What meds and when symptoms began, perform focused neuro physical. 3. Stat labs: cbc, abg, electrolytes, glucose 4. Stat head CT if suspect stroke and neuro consult. 5. Order my delay emergence panel: twitch monitor, neuro exam, glucose, temperature, abg, electrolytes, cbc, reverse meds, CT scan.

Drugs to avoid in pheo resection

1. Histamine releasing drugs: morphine, atracurium 2. Drugs that cause an increase in sympathetic outdrive: atrophine, ephedrine, ketamine, desflurane 3. Succinylcholine due to fasciculations

What are some symptoms of a tracheal tear?

1. Hypotension, tachycardia 2. Bilateral pneumothoraces 3. Subcutaneous emphysema This is emergency surgery

What your ddx for post op disorientation

1. Hypoxia, hypercarbia, hypotension, malignant arrhythmia 2. Anesthesia causes: gas, opioids, paralytics, reactions 3. Withdraw or intoxication from substance abuse 4. Metabolic changes: electrolytes, temperature, thyroid 5. Neurologic: post ictal, edema, stroke

What is controlling glucose in the OR important

1. Impaired immune response 2. Increased risk of infection 3. Impaired wound healing 4. Dehydration secondary to osmotic diuresis 5. Risk of fatal nonketotic hyperosmolar coma

During laryngeal papilloma resection, you induce and place your IV will you convert to a TIVA? why?

1. Inhalation will lead to OR gas contamination 2. Variable depths of anesthesia 3. TIVA dose for a child would be 200-300 mcg/kg/min and remi .1 mcg/kg/min

Why is an epidural better than spinal for a regular case?

1. Less sympathectomy 2. Continuous infusion that is titratable 3. Post op pain

What cardiovascular concerns would you have on child for laryngeal papilloma resection

1. Long-standing chronic airway obstruction 2. RVH or cor pulmonale 3. We would want a baseline ekg and cardiac echo to determine the presence of cardiac remodeling

One lung ventilation indications

1. Lung isolation to prevent damage or contamination of the healthy lung. 2. Isolation of ventilation ie bronchopleural fistula 3. Single lung lavage 4. Helps with lung drainage and improves operative field 5. Surgical exposure for : thoracotomy, pneumonectomy, esophagectomy

During laryngeal papilloma resection the child desaturates, what do you do?

1. Mask ventilate with 100% 2. Have the surgeon intubate until improvement

How do you treat acute chest syndrome

1. Mechanical ventilation 2. Serial ABGs, chest xrays 3. Broad spectrum antibiotics 4. Transfusion to maintain Hgb >30 5. Incentive spirometry, bronchodilators

What are the safety precautions to using a CO2 laser during ENT surgery

1. Minimize flammable surgical drapes 2. Wet towels should cover face, neck, and shoulders to absorb laser energy 3. Cover eyes with moist gauze 4. Safety glasses by OR personnel 5. Low FiO2 and avoid nitrous 6. Laser safe tube with a saline filled cuff. 7. Fire extinguisher should be present in the room. 8. Special masks to prevent spread of virus upon vaporization.

What factors do you consider when deciding if a surgery can be down on an outpatient basis for a patient OSA

1. Potential for undiagnosed sleep apnea 2. Extent and severity for any co-existing disease 3. Nature of the procedure 4. Intraoperative anesthetic requirements 5. Capabilities of the center: emergency airway, radiology, lab, transfer to tertiary center. 6. Discharge, are family or tranport available. 7. Will the patient requirement a lot of post op opioids If the patient has OSA and you have 1-7 then you may proced with the procedure.

What are the stages of liver transplantation

1. Prehepatic phase: period where the native liver is mobilized and dissected 2. Anhepatic phase: period when the native liver is removed. 3. Reperfusion phase period during which the new donor liver is anastomosed

What are the 5 risk components for post op intubation with myasthenia patients

1. Pyridostigmine dose greater than 750 mg daily 2. Disease burden > than 6 years 3. Concomitant pulmonary disease 4. Peak inspiratory pressure > 25 5. Vital Capacity less than 40ml/kg

Indications for PA catheter

1. RV or LV failure 2. Cardiac output in a patient with an arrhythmia 3. Differentiate cardiogenic from non-cardiogenic 4. Cardiac and pulmonary hypertension workup 5. Determine the type of shock 6. Assessment of valvular heart disease 7. Assessment of cardiac tamponade

How would you evaluate a child for laryngeal papilloma resection

1. Review all last anesthesia records and confirm with the ENT surgeon that his condition is not worse than before. 2. Review neck CT to confirm the size, shape, and location of the lesions.

What would an EKG for RVH show?

1. Right atrial hypertrophy by having peaked p waves in 2, 3, and AVF. 2. RVH would be seen by right axis deviation and partial or complete rbbb.

What are your anesthetic concerns for a pt with carcinoid syndrome

1. Routine cardiac and pulmonary precautions 2. Ask about triggers for carcinoid, stress, anxiety, exercise or certain foods, pain 3. Caution about histamine release medications, morphine, succinylcholine, atracurium, and certain beta agonist or indirect acting agents. You want to avoid stimulation. (avoid ephedrine) 4. Careful with surgical manipulation of the tumor as it can trigger carcinoid crisis. 5. Be prepared with alpha and beta antagonists for swings in blood pressure. 6. Avoid histamine releasing agents

TE fistula tips

1. VACTER-order pre op for this 2. Consider DLT 3. Pre op G tube and suction blind end of esophagus 4. Intubate past fistula 5. Precordial stethoscope for bronchial obstruction 6. Inhalational induction 7. AVOID PPV is top 8. Don't extubate, these kids are electively paralyzed and ventilated for 5 post op days

When a sickle cell patient has a crisis, what are some of the different crises that can occur

1. Vaso-occlusive crises with microinfarcts 2. Aplastic anemia 3. Splenic sequestration 4. Hemolytic crisis will occur if the pt has G6P deficiency 5. Acute chest syndrome

Briefly describe a CPB machine

1. Venous reservoir: deoxygenated blood collects from pt 2. Oxygenator: next step 3. Oxygenator to the arterial filter 4. Arterial filter to arterial cannula Would you prefer a membrane or bubble oxygenator: membrane as its less traumatic to the blood so less hemolysis

What are some measures you would take to prevent a sickle cell crisis in the OR

1. Warm up the OR 2. Use a fluid warmer 3. Use a Bair hugger 4. Blood products need to be available 5. Place invasive monitors for hypotension and acidosis ABG check

After the clamp comes off in a renal transplant and the blood pressure drops, what do you do and whats your ddx

1. Wash-out of vasoactive substances 2. Pulmonary embolism. 3. Diffusion of calcium channel blocker 4. Divert blood volume or redistribution

What is the dose of magnesium for torsades

1. With no pulse 1-2 g over 10 minutes 2. With a pulse same but then infusion 1g/hr

How would you induce a 12 y/o child for laryngeal papilloma resection?

1. Without an IV perform a slow inhalation induction while maintaining spontaneous ventilations. 2. If there was an IV in place midazolam, ketamine, lidocaine. 3. Either way we would have emergency airway equipment in the room and ENT on standby for emergency trach 4. After inhalation, place an IV and convert to a TIVA technique.

3 liver implant techniques

1. clamp it all, preload drops like crazy 2. Bypass, portal and IVC to the axillary vein Downside is lymphedema, vascular injury, air embolism, heparinize? 3. Piggy back where part of the native IVC is oversewn and kept

When you think of renal failure, what are your top concerns

1. gastroparesis/full stomach 2. coagulopathic-dysfunctional plts, and prolonged PT levels, 3. Cardiomyopathy

In a TURP the surgeon will use a laser, what are your concerns with this

1. proper laser depth so there's minimal damage to surrounding tissue 2. Risk of fire in OR if mishandled, so place in standby when not being used 3. Personnel must wear appropriate eye protection

How do you calculate PBW

1.3IBW

What is the dose of hydrocortisone for refractory hypotension in a chronic steroid pt

1.5 mg/kg

What is winter's formula and what do you use it for

1.5*bicarb +8 +/-2. Tells you the expected PCO2 for a metabolic acidosis

What is your dose for local anesthestic toxicity

1.5ml/kg bolus .3ml/kg/min for 30 min

With neonatal resuscitation, how deep do you do compressions

1/2 the AP diameter 2 inches in adults

How much potassium can you give po and through a cental line

10 meq and 20 meq. normal reference range is 1.5-2.5 mg/dl

How much of a burn do you need for succinylcholine to cause hyperkalemia risk?

10% BSA

What is the ruleof 10s in pheochromocytomas

10% bilateral, 10% outside adrenals, 10% malignant, 10% familial

Whats a normal PTT and PT value

10-13 for PT and 25-35 PTT

What is the normal hemoglobin in a pregnant patient

10-14 g/dl

As youre doing a case of DLT the sat drops to 93%, whats your next step

100% O2, auscultate the chest, manually ventilate, check with fiberoptic scope, peep the D and CPAP nonvent, suction both lungs for mucus plug

Contraindications to carotid endarterectomy

100% occlusion less than 60% asymptomatic occlusion, complete prior hemispheric stroke ie nothing to save. -severe stenosis contralateral stenosis is relative -poor EF

What is a normal LV pressure

100-140/2-10

If you do decide to give narcan to newborn whats the dose whats the fluid bolus at this age ?

100mcg/kg 10ml/kg fluid

Ratio of heparin and protamine

100u to 1 mg

how do you do a superficial and deep cervical block ?

10ml along the posterior border of the sternocleidomastoid muscle Deep: feel for the C2 transverse process felt just below the mastoid process. Then injection from C2 transverse to C6 10 ml divided up. For superficial do the same thing but along the posterior border the SCM

What is the normal Hgb in a 3 month old

12

How long do you have to hold lovenox before epidural

12 hours

ACLS for unstable Vfib and Vtach

120-200 biphasic compressions epinephrine 1mg q3min

What is the definition of gestation htn

140/90 without proteinuria after 20 weeks. It's considered chronic htn if before 20 weeks.

What is the definition of a fetal accel

15 bpm increase over 15 seconds

What is the normal Hgb in a neonate

15-21 g/dl

What is the dose of amiodarone for afib and for ACLS

150 over 10 minutes. 300 mg for coding.

What's your end goal for the treatment of hyperglycemia intraoperatively

180

Different stages of sarcoidosis 1-4

1: Bilateral hilar adenopathy 2. B/LHA with pulmonary infiltrates 3. pulm infiltrates without b/l ha 4. fibrosis

How do you calculate the amount of body surface area burned in an adult and in a child. Tell me the different percentages per body part

Adult: whole head: 9% whole arm 9x2 whole leg 18x2 whole torso+stomach+back: 36 Baby whole head 18 whole arm 9x2 whole leg 14x2 body 36

For stable a fib, whats a low risk vs a high risk pt when considering electrical cardioversion

Afib for less than 48 hours or on more than 3 weeks of OAC

POCD causes and treatment in ICU

After ruling out life threatening causes of delirium such as hypotension, hypercarbia, hypoxia. Ruling out medications that can cause delirium benzodiazepines and antipsychotics, antihistamines, anticholinergics, antibiotics, steroids, pain. Treatment can be precedex

What are the risk factors for post intubation croup

Age 1-4, repeat intubation attempts, prolonged surgery, oversized tube, oversize cuff, more likely with uncuffed tube. Treatment is with racemic epi and they receive this they have to be monitored for 4 hours due to rebound.

What are some physiologic risk factors that increase the gradient between PaCO2 and Peco2.

Age, empysea, age, hypovolemia, a PE

List the aldrete score and phase 2 discharge criteria

Aldrete ABCCO 9 out of 10 point score Activity Moves all 4 on command, moves 2 on command Breathing freely-2 dypsnea-1 apnea-1 no dyspnea, mild Circulation <20% 2 and 20-50% 1 Consciousness fully awake 2, arousable 1 Oxygen >92% RA, greater than 90% with device Phase 2 NAPS Nausea/vomiting Activity here will be gait, without assistance 2, with 1 Pain, controllable -2 Surgical bleeding

What is the mechanism of demerol for shivering and what do you do if the patient has an allergy to it

Alpha 2 agonist. give morphine as alternative

What are the side effect of taking your morning pyridostigmine pre-op in a myasthenia pt

Altered drug requirements after surgery increased vagal reflexes possibility of disrupting bowel anastomosis due to hyperperistalsis Risk of cholinergic crisis when give neostigmine for reversal

For intraoperative hyperglycemia, would you run an insulin gtt or a bolus?

Always a bolus + gtt. The only time to use an insulin bolus is for hyperkalemia

How would you assemble a transtracheal jet ventilator

Ambu, 3cc, ETT connector, 14 gauge catheter

On a TEG if you see fibrinolysis what do you give

Amicar if lysis at 30 minutes is greater 3%

When would you avoid using cell saver

Amniotic, tumor cells, fecal material, tumor cells, bone cement

What kind of intubation will you use if the patient is in a halo?

An awake fiberoptic intubation

With liver transplant why would you monitor CVP post op

An elevated CVP can lead to venous congestion and bleeding fo the new liver and graft dysfunction.

In neonate where would you place a central line, for example ina diaphragmatic hernia

An umbilical central line which connects to the portal vein for monitoring volume status giving ionotropic agents administering blood TPN Ductus venosus shunts umbilical directly to the IVC You wouldn't place the central line in the fem or IVC No femoral because as the diaphragmatic hernia gets reduced they'll be compression on the IVC and not in the neck as you may want to save that for ECMO

Tell me about dexmedetomidine

Analgesic Onset in 15 minutes metabolized by the liver half-life is 2 hours infusion is .1-1 mcg/kg/hour

What a quick ddx for hypotension during a case?

Anaphylaxis, cardiac shock, arrhythmia, hypovolemia, anesthesia overdose.

Whats the difference between anesthesia recall and paralytic recall?

Anesthesia recall can occur even with proper depth paralytic recall occurs due to inappropriate depth

How does HIV status effect your anesthesia

Antiviral therapy interactions Substance abuse Opportunistic infections Tumors like airway papillomas peripheral neuropathies myocarditis and dilated cardiomyopathies Some agents like AZT can suppress bone marrow and lead to a coagulopathic state

Contraindications to IABP

Aortic valve regurg, aortic dissection, aortoiliac disease, vascular aneurysm at the site of insertion

Review APGARS

Appearance No cyanosis, blue peripheral, blue central Pulse >100, <100, none Grimace Stimulation to morrow or suction, minimal response, none Activity- flexed arms and legs, some flexion, none good cough, weak cough/cry, none

What is your worry in a CEA about applying a clamp and then what is your worry about removing the clamp

Applying: having a MAP high enough for collateral flow, FIO2, assess neuro status Removing: having a MAP low enough so you do hyperperfuse

With a mediastinal mass, you can get SVC syndrome, what are some symptoms of SVC syndrome

Arm edema, face swelling, cyanosis, JVD, HA, confusion, mental status

How do arm burns and torso burns affect your anesthesia

Arm: no lines in that arm. Torso-inadequate tidal volumes

What is an EF for a patient with HOCM

Around 80%, which is hypercontractile

During a regular case with a pt with carcinoid syndrome what is your ddx for hypotension

Arrhythmia, RV failure, anaphylaxis, anesthesia overdose, carcinoid crises, vasovagal if in preop

What is the most likely cause of post op oliguria after a TURP

As Dr. Deangelis said, always think of plumbing. Probably a bunch of clots in the urethra. Flush the foley and place it on continuous irrigation.

How would you prepare for reperfusion in liver transplant? What is the danger from this

As blood comes through the new liver metabolites are flushed into the systemic circulation. This will lead to decrease in cardiac output, SVR, and abrupt rises in pulmonary vascular resistance, increase K and H+. 1. Keep volume optimized 2. Check ABG and H/H, electrolytes, transfuse accordingly. 3. Pressors lined 4. Insulin, calcium, bicarb

What is the difference between CVVH and dialysis

CVVH: Continous veno-venous hemofiltration uses convection to remote solutes and toxins while hemodialysis uses diffusion. Dialysis can cause hypotension and cerebral edema CVVH tends to cause less hypotension and doesn't require heparin

What are the two ways to test for MH

Caffeine halothane contraction test and muscle biopsy ie genetic testing Of note MH can occur in the PACU. POST OP MH!

In routine case your patient's heart rate drops to 23 bpm, what do you do

Call for a crash cart, 100% O2, alert surgeon, open fluids, feel pulse, atropine, epi, while preparing bed for compressions

What's the major reason bronchial blockers suck

Cannot deflate the lung you're working on

How do you diagnose CO poisoning

Carboxyhgb level in blood symptoms CNS n/v disorientation, headache myocardial depressant half life of CO is 5 hours but a non-rebreather will take that down to 1 hour

With a dissection patient was are your preop concerns

Cardiac Pulm: is there a history of smoking or COPD because this patient will likely need to remain on the ventilator post operatively Renal: Bun/Cr Renal dysfunction preop due to the dissection is the single most predictor for kidney failure after repair

What can you measure with a PA catheter

Cardiac output Cardiac index PA Pressure CVP Calculation of oxygen delivery Assessment of cardiac work Mixed venous oxygen saturation pulmonary capillary wedge pressure SVR

What are the preop considerations for sickle cell disease

Cardiac: risk of LV failure and cardiac myopathy- so ensure proper EF Pulmonary: can demonstrate restrictive pattern, acute chest syndrome, -so cxr GI: mesenteric ischemia, hepatic dysfunction Renal: UTI, priapism, nephritic syndrome so normal BUN/Cr Neuro: developemental delay, hx of stroke, seizures, mental retardation Hematologic: aplastic crises, may have alloantibodies in blood so want to talk to blood bank to ensure they have the supplies Immunologic: neutropenic, predisposes to infection

What kind of edema is TACO and TRALI

Cardiogenic and non-cardiogenic edema

What are your concerns about a pt with chronic alcoholism

Cardiomyopathy arrhythmia like a fib Cirrhosis or fatty liver may be present so avoid hepatic metabolism Avoid neuraxial techniques due to prolonged coags Gastritis so consider a full stomach higher MAC valus will be needed

What is your treatment of a patient with 95/45 BP in SVT

Cardiovert. Below 100 systolic, consider cardioversion

What is pectus carinatum and excavatum?

Carinatum is protruding chest and manubrium, sternum, and ribs. Causes include marfans, ehlers danlos, turners syndrome Excavatum is inward both of these can cause filling defects of the heart, lungs, right heart failure, obstruction of greater vessels.

How would you plan for a placenta accreta patient?

Central line, typical EBL is 2.5-5L, so type and cross and units available. Ask IR to get access to the uterine arteries to balloon if needed. Usually the uterus or a hysterectomy will occur. This is a planned cesarean hysterectomy

How does decadron decrease PONV

Centrally mediated process of inhbition of opioid action and inhibition of prostaglandin synthesis

Patient is not peeing, are they dry or overloaded?

Chest xray, TTE-the view is called subxiphoid view and measure diameter and collapse with inspiration. If the IVC is flat with over 50% collapse then that's dehydration

What are the 5 classifications of myasthenia gravis

Class I:Ocular weakness Class 2: mild weakness of other muscles. Other being axial, limb, oropharyngeal or all Class 3: Moderate impairment Class 4: Severe Class 5: requires intubation

If a patient develops TURP syndrome, how do you know its not just bladder perforation?

Classically there should be shoulder pain.

What is the K value in a teg

Clot strength

What are the side effects of ECMO

Clot, air bubbles, increase risk of stroke

Management and ddx fo oozing

Coagulopathy hypothermia platelet dysfunction check the field check much protamine reversal labs ffp, teg

What is a cobb angle and how do you use it in anesthesia

Cobb angle is where you take the two most deviated vertebrae and draw two intersecting lines. Then drop a perpendicular line from each and the intersection of those is the cobb angle. A cobb angle of 40 requires surgery cobb angle of 65 interferes with respiratory function These patients start hypoventilating and get hypoxia and hypercarbia

For a congenital diaphragmatic hernia how would you maintain anesthesia

Combination of air/O2 to keep sat 92-95%, sevo, fent, vec. Be aware that volatiles block pulmonary hypoxic vasoconstriction and can create shunt Avoid nitrous as it can diffuse in bowel creating even more respiratory compromise,

How is carcinoid tumor diagnosed

Confirmed about 5-hydroxyindoleacetic acid

Whats your approach for regional blocks in the OR?

Consent pre-op and place post operatively

Adult retarded patient going to MRI, will you do LMA or ETT

Consider is there involuntary facial twitching, uncontrollable seizures, aspiration risk or history and if so intubate.

How would you induce a pt for a renal transplant

Consider their gastroparesis and cardiac risk. 1. Preoxygenate 100% O2 2. Do a RSI and use fentanyl, etomidate, and succinylcholine

What are your anesthetic considerations for a patient with myasthenia gravis

Considerations = manifest! so HPI H: hx of pneumonia, prior intubations, medication, post op intubations, how well controlled, respiration function/exercise tolerance, medications, well controlled, problems with chewing, swallowing, or speech, associated with an tumor or thymus hyperplasia, dyshagia, difficulty taking a deep breath. Any GERD and feeling of acid as myasthenia patients always have GERD. Any plasma exchange or IVIG P: dysphagia, ocular, proximal limb weakness, wheezing, rales Imaging: MRI for airway compression from thymus

What are the clinical factors of how one gets a tet spell

Decreased SVR-hypovolemia, acidosis, hypocarbia Increased cardiac contractility and sympathetic activity lead to more right ventricular outflow obstruction. That's why it is triggered by crying the crying, defecating, causes a fall in oxygen and increase in hypercarbia

What are the effects of hypothermia in OR

Decreased drug metabolism wound infection due to impaired immune system and decreased cutaneous blood flow impaired platelets and coags cardiac arrhythmias and death increased O2 consumption from shivering

How does a balloon pump work

Decreases myocardial oxygen demand, increases cardiac output, decreased after, improved EF

Whats the difference between a premature infant and one that is normal

Decreasing sucking/swallowing/latching -> vitamin deficiencies ADEK-> coagulation deficiencies, temperature regulation, hypoglycemia,

What anesthetic gas would you use with an OSA pt and why

Desflurane, insoluble in fat, rapid wake up, fast return of reflexes, reduce amount of post op sedation.

How would you evaluate a pt with carcinoid syndrome

Determine if the tumor is functional or non-functional via history and physical exam. Functional tumors secrete substances causing diarrhea, flushing, where non-functional tumors do not. From history has pt had any bronchospasm, diarrhea, dramatic swings in BP, palpitations. From exam listen for bronchospasm, heart murmurs ie right heart region. Then look at imaging studeis to see where the tumor is located and the size.

What are your concerns for a child with cleft palate

Difficult airway, aspiration risk, nutritional deficiencies cause post op apnea and anemia

What is DLCO

Diffusing capacity of carbon monoxide increases with asthma, polycythemia, left to right shunt, exercise decrease fibrosis, copd, anemia, heart failure, pulmonary hypertension

Why is hypokalemia a RF for digoxin toxicity, what other drug should you closely monitor?

Digoxin competes with potassium for the Na/K ATPase receptor on cells. So if less potassium digoxin will bind more. The other drug you have to monitor is diuretics. Digoxin is metabolized via kidneys

Advantages and disadvantages of HFJV

Disadvantages airway must be wide enough to allow passive exhalation Each breath leads to air entrapment due to the venturi effect Each breath leads to the entrapment of air possible PTX Dries out airway leading to cilia impairment barotrauma

Tips and verbage

Do not push answers to consults, do not say interesting, use opioid and not the term narcotic. Don't say patient is light, say they're awake

Verbage

Do not say slow and controlled and use clinically unstable

Drugs that do and don't cross the placenta

Don't paralytics insulin heparin glyco do cross opiates bzs locals atropine beta blockers Drugs that pass through that have minimal effect is prop ketamine thiopental epidural opiates

What is the best way to detect an MI in the OR

Drop a TEE probe and look for hypokinesis is the gold standard

How do you diagnose VAE

Drop in CO2 Drop in blood pressure Millwheel murmur

What is the hypotensive technique with shoulder surgery

Dropping BP w/i 30% helps with 1. Arthro visualization 2. Decreases ebl 3. Shortens surgery time It is ok to do this even with a cardiac history if ekg and stress test is normal.

How do you evaluate fluid status in a ESRD

Dry weight cxr dry or weight

Why do we use the parkland formula in burn pts?

Due to leaky capillaries

Why are cirrhotics hyponatremic?

Due to splanchnic vasodilatation the kidneys become less perfused and the RAAS increases but ADH is increased more so leading to hyponatremia

You place your interscalene block and the patient becomes hypotensive and bradycardic, why?

Dural placement. You also can get recurrent laryngeal nerve palsy, and/or horner syndrome

What cardiac abnormalities are present for a patient with HOCM

Dynamic left ventricular outflow obstruction, mitral regurgitation due to systolic anterior motion of the mitral valve, diastolic dysfunction, myocardial ischemia, arrhythmia

What is the difference from ECMO to CPB

ECMO circuits do not have a venous reservoir Difference in placement of cannulas complete bypass of the lungs

What is the difference between ECMO and CPB circuit

ECMO is a closed circuit where bypass you can drain the heart and add blood to the heart ECMO is also a treatment for refractory vtach

Compare EEG and SSEPs

EEG is a global measure of blood flow to the brain which is normally 50ml/100g/min. SSEPs are just of the dorsal column

How do you know if hyperkalemia is normal in a patient or not?

EKG changes

What two preop labs do you need for a htn pt

EKG, Cr to check for kidney damage

Name the 5 ASA monitors

EKG, blood pressure, pulse ox, capnography, temperature

What preoperative tests will you order with a renal patient

EKG, cxr, coags, LFTs (pancreatitis)

How do you induce epiglottis

ENT in the room cric kit inhalational induction in the sitting position

What gear/personnel do you need for an epiglottis pt

ENT surgeon, difficult airway cart. Do everything you can to prevent from upsetting the child. This includes separating the child from the parents

What are the 4 indications of HFJV

ENT-Provides unobstructed view, minimal movement of vocal cords, Critical care thoracic surgery-can be added to the surgical lung like CPAP radiology

Why would somone get post op hypoventilation after a carotid endarterectomy

If the surgeon infiltrated the carotid body with local

During a TEF repair case the pt starts to desat during positioning, whats your ddx

ETT into the right mainstem, increased gastric distention, ETT kink/disconnected, mucus plug or secretions, surgical compression of vital structures. Lastly pneumoperitoneum can cause desaturation

What a good estimate for ET tube depth

ETTX3

Benefits of oxygen analyzer

Easy to calibrate, last line of defense against hypoxic mixtures Reliable

Whats ebstein anomaly and eisenmenger syndrome

Ebstein is atrialization of the right ventricle Eisenmenger is pulmonary htn that develops into R->L shunting

During retrieval of a foreign body in a child the O2 sat begins to drop as the surgeon is pulling the fb, what do you do?

Either pull it out of push it distally

What are some ways you can tell if a patient is having end organ perfusion in the OR

EkG changes urine drop off acidosis

During a crani the surgeon complains of a tight dura, how can you help

Elevate the head hyperventilate, check oxygenation, turn down volatile agent and paralyze, give a bolus of propofol, give muscle relaxants to decrease ICP and increase venous return, give diuretics, CSF drainage

What are some down sides to veno venous bypass for liver transplant

Emboli, local edema, vascular injury.

What is your ACC 360 guidelines for pre op chest pain

Emergent? No then ACS work up. If this has been performed then perform an RCRI, if less than 1% then they are considered low risk and proceed. If greater than 1 factor then perform METS. If greater than 4 METs then proceed, if unknown or not greater than 4 then stress test for possible intervention.

What is your differential for a decrease in O2 saturation

Endobronchial intubation, cuff deflated, 4Ps, CHF, aspiration/mucus plug, extubation, bronchospasm

How do you control a carcinoid pt pain post op

Epidural is fine but beware of hypotension as it is a trigger for carcinoid syndrome.

If you had to pick a regional technique of spinal or epidural for a case which would you pick?

Epidural. The spinal tends to have more sympathectomy

Patient in stage 3 shock, whats your induction agent

Etomidate

How do you know if a burn pt is a difficult airway

Exam, vitals. Stridor, wheezing, soot around the nares, body surface area of the burn, facial burns, headache, n/v, dizzy

What's the fena value for oliguira pre and intra-renal

FENA for intra-renal >2% and less than 1% for pre-renal

What are the values for ppFEV1 that you worry about after pneumonectomy or lobectomy

FEV1 > 40 No anticipated post op issues FEV 1 <40% > 30 Likely to require post op ventilation and there's an increase risk of death and morbidity

Whats the FEV1 and FVC on a restrictive lung and obstructive lung disease

FEV1 and FVC they're normal or increase because both values are down. With obstructive the ratio is decreased

What does PFTs provide you with?

FEV1, DLCO, VO2max. These values can help with deciding if the patient is prone to bronchospasm, vent settings, responsive to bronchodilators, baseline for extubation

What is your immediate actions for post op c section bleeding

FIO2 Left uterine displacement Call for hemorrhage cart Stop epidural Stop mag

Work up for blood in the ETT

FIO2 100, DLT, Fiber optic, call CT surgery

How would you induce a liver patient ?

Factors Increased intrabdominal pressure-full stomach Catecholamine depleted Intravascular depleted Preinduction arterial line, prehydration I would perform an RSI with cricoid using etomidate and succinylcholine

What are some triggers to acute chest syndrome?

Fat embolism, fluid overload, hypoventilation, thrombi, bone infarction, infection, pneumonia.

Side effects of TPN

Fatty liver, venous thrombi, work of breathing, acidosis

When working up oliguria, diuretics will only effect which lab?

Fena and other shit. It won't change BUN/CR

What would you do for pain for a neonate/toddler post op

Fentanyl 0.5mcg/kg q1hr prn, tylenol 10mg/kg q4 or a caudal

What are the symptoms of DIC

Fever, low blood pressure, IV site bleeding, bleed from catheter and mucus membranes

First two steps to any OB issue

FiO2 to 100 and left uterine displacement

First step with an increased peak pressure?

FiO2 to 100%

What is your differential for unable to wean from bypass

Filling issue contractility issue vasoplegic syndrome periprosthetic complications hypothermia dissection SVR problem

Troubleshooting hypoxia in a double lumen tube

First step is to increase FIO2 and to grab your bronchoscope and verify placement. Too far can obstruct the left upper lobe. Suction out mucus, verify no kinks in the tube. Peep to the ventilated lung. CPAP to surgical lung. Resume two lung ventilation. Clamping of the pulmonary ARTERY, only really helps with pneumonectomy. Last resort is putting patient on bypass.

Three major things to tx VAE

Flood the field, PPV, FIO2 100

Name a couple of mechanisms that help avoid TURP syndrome

Fluid bag less than 30cm above patient empty bladder frequently limit resection time to less than 1 hour avoid hypotonic fluids

What are the symptoms of carcinoid syndrome

Flushing, diarrhea, abdominal pain, bronchospasm, htn or hypotension, and heart failure

How would you maintain anesthesia in this patient

I would choose a balanced technique for unconsciousness I'd use for amnesia I'd use propofol for analgesia I'd use for paralysis I'd use

What is the Hunt Hess Grading Score

For SAH, 1-5 The purpose of it is that it portends mortality Grade 1-asymptomatic or mild HA Grade 2-Moderate to severe HA ocular nerve palsy, nuchal Grade 3-Confused, mild signs in limbs like paresthesias or weakness Grade 4-hemiparesis, stupid, localizes pain Stupor, hemiparesis, some Grade 5-deep coma decerebrate

What is your management of preoperative steroids

For replacement to be considered the patient should be on at least 2 weeks of steroids If mild surgery then take normal daily dose or .2-.5 mg/kg day of surgery If moderate surgery then 0.5-1mg/kg 1-2 days if severe then 1-1.5mg/kg 2-3 days

What is the R value in a TEG

From the start to te initial fibrin formation

What kind of ETT would you use for a neonate

Full term newborn 3.0 At 6 months 3.5 At 1 year 4.0 Once 2 you can use the age/4 + 4

What are late complications of TEF repair

GERD is the most common and long standing problem after this repair. Also strictures, recurrent aspiration, pneumonia, recurrent lower and upper respiratory infections, reactive airway disease, feeding issues, and oral aversion. Decreased mucociliary clearance due to lack of epithelial cells where surgery took place. Esophageal dysmotility. tracheomalacia abnormal pulmonary flow rates, reduced lung volumes, airway hyperreactivity, winged scapula from thoracotomy

Pregnant patient has an altered mental status, do you do an epidural or general?

General, as they now have become an aspiration risk. Also, by being altered they may have low cerebral perfusion and now if you provide a sympathectomy you will drop their cerebral perfusion pressure. Plus if they're obtunded they can't cooperate with positioning. Also, platelets can be low or dysfunctional with HELLP syndrome.

What is a major side effect of giving a pt octreotide

Glucose intolerance

How will you assess the bleeding that just occurred or the patient's breathing etc

Go see the patient do history and physical order labs/blood gas

Why did you choose bupivacaine for your epidural?

Good differential blockade in terms of sensory without motor

Problems with EVAR

Graft migration occlusion collapse

What is the urine osmolality in a pt with pre-renal oliguria

Greater than 500 mOsm/L

How do you preoperatively assess a myasthenia patient?

H&P History: 1. Are their symptoms ocular or do they involve laryngeal and respiratory muscles evidence by difficulties with chewing, talking, swallowing. 2. Pyridostigmine and how much 3. Surgery before and any prolonged intubations or problems with anesthesia, any episodes myasthenia crises and whether her symptoms have been stable over the past few weeks. 3. Duration of disease 4. Physical exam assess bilateral motor strength in all extremities and see if there's pharyngeal or respiratory muscle involvement

How would you evaluate a patient with carcinoid syndrome cardiac status

H&P activity level, chest pain, sob. Physical exam heart murmurs signs of CHF, edema, elevated JVD. Then look at EKG, echo, or stress test. TR is the most common finding on echo.

How would you evaluate an airway of a pt with a goiter

H&P dysphagia, dyspnea, prior intubations. Flow volume loop for intra/extrathoracic compression, and look at CT scan to evaluate extent and size of tumor.

How would you evaluate the pulmonary status of a carcinoid pt

H&P cough, sputum production, dyspnea, exercise tolerance, oxygen requirement, cyanosis, history of pna. Physical accessory muscles, wheezing, rhonchi, rales. CXR and EKG H&P

How would assess a neonates volume status say for pyloric stenosis

H&P, frequency and quantitiy of wet diapers, vomit, IV fluids so far. Exam vitals, membranes, sunken fontanel, cap refill, skin turgor, mental status.

in a trauma pt how do you assess for fluid status

H&P, paramedics notes, vitals, blood and fluid products. STAT bmp, abg such as hypernatremia and acidosis

Pre op ICP eval

H: somnolence, gait disturbance, N/V, mentation, HA P: cushings reflex, neuro exam, nuchal rigidity Imaging: mass defect, hydrocephalus

What are your parameters for post SAH triple H therapy

HCT 33, CVP 10-20, Systolic 160

Tips for preclampsia

HELLP DIC Type and cross volume depletion

What is the treatment of vasospasm occurs

HHH Hypervolemia Wedge 18, CVP 12 Hemodilution-Hct in the 30s HTN

Treatment for cerebral vasospasm

HHH and nifedipine

How do you reduce ICP preoperatively

HOB up, quiet room ask the pt to hyperventilate apply 100% oxygen give mannitol and lasix

What is the tx for pulmonary edema

HOB, lasix, milrinone to drop preload, fluid restrict

What are the normal pediatric vital signs for a newborn-1 year

HR 110-160 BP 70-90/50-60 RR 30-40

What are the normal pediatric vital signs for a 5 year old

HR 70-120 BP 100/70 RR 20-25

What are some sequelae of cocaine abuse in the OR

HTN, tachycardia, arrhythmia, myocardial ischemia, hyperreflexia, and convulsions

What is the major sign you will receive intraoperative for hypoglycemia

HYPOTENSION!!!

How would you induce a patient with a mediastinal mass

Have people ready for bypass, inform the surgeon that we need a rigid bronchoscope in the room ready for ventilation and a sternal saw if life threatening airway compromise occurs, then we will perform a spontaneous breathing awake intubation

Whats your ddx when unable to ventilate coming off bypass

Have surgeon look at the lung directly if barely moving, take a fiberoptic bronch and look down for a mucus plug

How would you induce a pheo pt?

I would have pressors and dilators on standby and perform an induction with etomidate while observing my pre-induction arterial line.

Most trauma patients are going to be considered c spine precautions how do you determine if he patient needs to be an awake fiberoptic ex for an ex lap

Have the pt verbalize and range their neck. If they can do this without pain then RSI. If extremely cooperative and time permits then awake.

If a pt has elevated ICP whats are some treatments you can do?

Head of bed 15-30 degrees, hypervent to PaCO2 of 30. Administer mannitol and lasix, bolus of propofol

During a renal transplant the surgeon is about to clamp the iliac vessel. What medication do you want to administer

Heparin-prevents clotting Ca channel blocker-prevents vasospasm when clamp comes off Diuretic-for perfusion

Discuss the serology of hepatitis

Hepatitis surface antigen indicates active disease vs Ig which is immunity vs vaccine surface antigen usually disappears but it they have it they're currently infectious

What is hepatorenal syndrome

Hepatorenal syndrome is a fatal condition that is associated with rapid functional renal failure secondary to fulminant lliver failure disease and cirrhosis.

What is a Hgb AA fraction?

HgAA or HgA2 is alpha+alpha+delta+delta. A fraction of 50% is desired to prevent sickling. HgA2 is high in beta thalassemia. This fraction is elevated in patients with sickle cell disease

What are the risk factors for uterine atony

High birthweight, labor induction/augmentation, chorioamnionitis, magnesium sulfate, previous PPH, retained products, DIC, lacerations, rupture, inversion, genital tract hematomas

Sequelae of hyperthyroidism

High output heart failure, arrhythmias, dehydration, tachy, htn, fever.

What is the difference between a premature infant and a regular one

Higher risk of infection, hypoglyemia

How do you evaluate the patients cardiac status

History: does history show a sedentary or active lifestyle. Symptoms of CHF, rhythm, CAD Exam:listen to heart, evaluate, evidence of right or left heart failure-elevated JVD, rhonchi Imaging:Review EKG and if see signs then CXR for right or left cardiomegaly. Look for at stress test and if positive signs then consult cardiology

Whats a good way to think about ASRA guidelines

How long do you have to wait after medication is given? 6 for heparin and 12 for lovenox, 24 hours for treatment lovenox How long do you have to wait after medication is given zero-1 for heparin and 6 hours for lovenox, 6 hours for therapeutic lovenox its ok to give heparin and lovenox with a catheter in place but usually not therapeutic lovenox

What are some symptoms of pheochromocytoma ?

Htn, sweating, tachycardia, headaches, chest pain

Hunt Hess score 1-5

Hunt Hess 1-2 you can extubate

What are some ways to tell someone is hypovolemic preop aside from exam

Hypernatremic, elevated H/H

Whats the contraindication to drug X

Hypersensitivity is always first

What are some factors that increase MAC

Hyperthermia, hypernatremia,

What is your ddx for hypotension during the anhepatic stage

Hypocalcemia from products, cardiac origin, check TEE to evaluate for wall abnormalities.

What do you look for on an echo to determine if the patient is having an MI

Hypokinesis and regional wall abnormalities

Does lasix make you hyper or hyponatremic

Hyponatremic and hypokalemic

What are some triggers for a carcinoid crisis

Hypotension, pain, stress, anxiety, exercise

What is the difference between hypoxemia and hypoxia

Hypoxia is defined as end organ damage due to lack of adequate oxygen and perfusion Hypoxemia is PaO2. So a crummy PF ratio is hypoxemia only. If stroke/MI/ or renal damage occurs then that is considered hypoxia

Causes of hypoxia vs peak pressure

Hypoxia will be v/q mismatch, diffusion impairment, shunt, hypovent, FIO2

Factors that decrease MAC

Hypoxia, hypercarbia, pregnancy, lidocaine, anemia, hypotension, hyponatremia, age, ketamine, alpha agonists

What factors can precipitate a sickle cell crisis

Hypoxia, hypotension, anemia, hyperthermia, infection, stasis, acidosis, pain, stress, hypovolemia

Would you add clonidine to your epidural?

I don't because it can cause hypotension and sedation

How do you give maintenance fluid

I use 4:2:1 as a guideline but I trend and see how their vital signs respond

If a stroke pt comes in and your center doesn't have rocuronium, what do you do for induction

I use sux as the effect of hyperkalemia is unlikely to occur when under 48 hours and this is for a difficult airway For a non-difficult airway then I would rocuronium without RSI

How do you induced a child for a foreign body aspiration who is also a full stomach who just ate

I will do an inhalational induction to prevent further migration of the foreign body into the airway which could bring on total airway obstruction or make retrieval impossible. If its a complete emergency then RSI is the way to go

The patient is hypotensive...should you use fluid or blood

I will let the Hct guide transfusion. If the HCT is stable and adequate then crystalloids

What is the key phrase when asked how will you evaluate this patients blah blah status

I will perform a detailed history and phyical exam I will compare the old labs to the new labs I will compare the old imaging to the new imaging and review the nephrologist notes On history I'll ask blah blah On exam I'll ask blah blah

After aortic cross clamping the bp rockets, whats your treatment

I will use something titratable and short acting like nitroglycerine

With this diabetic, how will you keep their insulin controlled in the OR

I will use the hospital insulin protocol and titrate it

Would you obtain PFTs for this patient?

I would not unless there was a severe respiratory component as evidenced by my H&P. I would then use the PFT NIF and FVC as a reference to determine optimal conditions for extubation or the need for post operative mechanical ventilation.

How would you do induction for epiglottis pt with a full stomach

I would still do an inhalation anesthetic with sevoflurane and oxygen until the child is drowsy and then place an IV and use ketamine to deepen the child and then DL. No RSI as it hurts to swallow thus unlikely full stomach Would not perform an awake as irritation can lead to complete collapse, anxiety, cooperation

If resuscitating a patient would you use LR or NS?

I would use NS because the calcium in LR can bind with the citrate and cause clotting. Also if it's a TBI LR is known to be slightly hypotonic.

What agents would you use for maintenance and why

I would use a balanced technique....

in renal failure if you have to used local anesthetics, which kind would you use and why?

I would use amide local anesthetics because renal failure pts can have a pseudocholinesterase deficiency

Why would you use a video laryngoscopy scope before a regular DL

I would want to also be able to look for occult injuries to the larynx and glottis in this trauma patient

If you have a patient with COPD and they're undergoing a thoracic surgery, is an epidural a good choice

I'd be worried that if they're having difficulty at baseline and now I take away some of their thoracic muscle strength that they'll be unable to have adequate tidal volumes.

What is the first thing you say when given a rhythm strip, cxr, or ekg

I'd first correlate this with my other vital signs

A patient has mitral stenosis, would you perform a regional technique?

I'd have to check oral anticoagulation status as most patients with MS have a fib

How would you place an IV in a pediatric patient with a history of MH

IM ketamine induction

Youre struggling to get a line in a neonate/kid, what do you ?

IO line

Would you prefer to administer your lidocaine for induction IV or transtracheal

IV because the first thing that occurs with transtracheal is coughing

IV vs mask induction

IV is more controllabled and quicker to prevent aspiration Mask induction is less traumatic and they maintain spontaneously ventilation

How do you perform an infraclavicular block

Identifiy the pec major and minor and below the minor you should see the axillary artery

What is asymmetric septal hypertrophy

Idiopathic hypertrophic subaortic stenosis, venturi effect of the anterior mitral valve

Treating torsades if there's a blood pressure and a good pulse vs without BP and pulse

If BP and pulse then magnesium 2-4 mg without blood pressure and pulse then defib

What is the ACLS treatment for atrial fibrillation

If arrhythmia is greater than 48 hours and OAC has been used for more than 3 weeks then cardioversion is ok, or no evidence via TEE. Rate control can be achieved with diltiazem, verapamil, mtp. If unstable cardioversion 100-200 joules, then 300 then 360. Diltiazem dose is .25mg/kg or 15 mg over 2 minutes and then add dilt gtt

How can you use an ACT and TEG post op to ddx bleeding in a CABG

If bleeding and ACT is normal, then use TEG If bleeding and ACT is abnormal then give protamine

What is an ascending bellows better?

If circuit disconnects you'll know

What are some cardiac effects of neuromuscular blockers

If histamine release or pancuronium causing tachycardia

While doing a carotid the patient comes to the OR with a BP 160/90, will you lower it

If it were chronic then no as this may be critical carotid stenosis and the patient may requirement this pressure. If acute I would figure out the cause; anxiety, pain. Also EKG to rule out myocardial events

What are the only times you can leave a tube in place during a fire

If it's immediately extinguished or the airway is beyond difficult

Do you give the pre-op pyridostigmine dose in a myasthenia patient

If mild-moderate then no If severe disease then yes if severe you do because you don't want any respiratory issues if you have to return to spontaneous ventilation in the beginning of the case If you give it, there's a risk of cholinergic crises towards the end when you have to give reversal If you don't give then theres a risk of anti-cholinergic side effects after reversal

What's the ACLS algorithm for PSVT

If no pulse go to PEA algorithm vagal maneuvers adenosine verapamil 5-10mg Amiodarone 150 over 10 If unstable go straight to cardioversion in doses of 50, 100, 200, 300, 360 unstable being a sbp less than 75

For a crani case would you use succinylcholine

If the airway is reassuring I would do an RSI with rocuronium with sugg ready. If it was not I would do Sux

Is the ABG compensated or uncompensated?

If the pH is 7.35 which i in the normal range then it is a compensated ABG

Why can't you place an IV or BP cuff on a stroke patient who has hemiparesis deficit of the arm

If the patient has flaccid hemiparesis then the values will be artificially low and spastic plegia will be high but the neuromuscular tone is completely off

What is your worry about performing an epidural in a patient with COPD

If the patient is already having accessory muscle use and your epidural reaches the T6 level then you'll begin to have issues of difficulty breathing, anxious, tachycardia and htn.

Preclamptic patients are risk of coagulopathy...do you place an epidural why or why not?

If the patient is stable ie doesn't have HELLP then place the epidural. If the patient is unstable then general. This is to avoid a spinal hematoma and a sympathectomy that can decrease cerebral

What is TURP syndrome

Its a complex of symptoms resulting from acute volume expansion and dilutional hyponatremia manifested as htn followed by hypotension, refractory bradycardia, and neurologic symptoms that range from vague to severe. These may show signs of lethargy, tonic clonic seizures, coma, and death

What are some indications for digibind

K > 5, arrhythmias

In kidney failure what are the only two electrolytes that increase?

K and phosph

How would you induce a patient with tetralogy of fallot

Ketamine, the benefit is increase in SVR thus avoiding a tet spell, avoid positive pressure ventilation. 1. Avoid succinylcholine due to risk of Malignant hyperthermia 2. Succinylcholine is associated with histamine release, which can decrease SVR and promote further right to left shunting of blood

What is your setup for a stem

Key Points Urgency Optimize Room setup Induction Pre op Post op Pain ROS Physical Exam Findings What is it? KUORI PPPP What

What are the contraindications to mannitol

Kidney failure Progressive CHF Hyperkalemia severe dehydration mannitol is a 6 carbon sugar

Tell me about pediatric blood transfusion

Kids are transfused rarely and dose in ml/kg rather than in units of blood to prevent volume overload.

What kind of spinal would you perform for a TURP, what level and with what drug

L4/5, 2ml of 0.5% hyperbaric bupivicaine

what are the 3 most important side effects of a regional technique

LAST High spinal sympathectomy

Contraindications to PA catheter

LBBB, tricuspid or pulmonary vegetations, endocarditis, right heart mass

What size LMA would you place for a child who is 30-50 kg and what age is this?

LMA 3 and this is a 9 year old

Whats the kind of airway you want to avoid in an obese pt

LMA, they're going to aspirate

For neonate having pyloric stenosis surgery, what fluids do you want?

LR should be avoided since it contains lactate which converts to bicarb in liver and this can worsen the acid base status. If the urine output is low then i would use normal saline until normal urine and switch to 1/2 normal with some potassium.

What is the fluid of choice in burn patients?

LR, since you're going to do massive resuscitation you want to avoid creating an acidosis

Balloon pump indications

LV failure, cardiogenic shock, mechanical complications of acute MI, unstable angina after medical management, stunned myocardium, failure to wean, bridge to transplant

What are your concerns about doing anesthesia on someone who abuses cocaine

Labile BP, HTN, difficult IV access. Acutely: v fib, seizures, catastrophic MI. So you will need preinduction art line 5 lead EKG Have of nitro, esmolol two large IV use only direct acting medications like phenylephrine and avoid indirect as they may have an exaggerated response. Lastly universal precautions as pt may have HIV or Hep C from other drug use

When evaluating a pulmonary status of an OSA or COPD patient. You do a history and physical but what else?

Labs and EKG look for RVH, baseline cxr, and PFTs

Whats the difference with NMBDs with lambert eaton and with myasthenia

Lambert is sensitive to both depol and non-depol. Myasthenia is sensitive to NMBDs and not to succinylcholine.

Describe tricuspid regurg on CVP

Large V waves

What are the major advantages of lasix over mannitol in decreasing ICP

Lasix doesn't increase cerebral blood volume or ICP, it can be used in CHF and renal patients Metabolism of lasix is some liver, and renal unchanged

What pre op information would you want for a patient with diabetes and htn?

Last AIC, any signs of uncontrolled diabetes such as nephropathy, neuropathy, vasculopathy. Resting tachycardia, early satiety, neurogenic bladder, postural hypotension, lack of sweating, impotence. With the htn has it been controlled, any nephropathy or LVH. Need a cbc for anemia of chronic disease, electrolytes for bun/cr and glucose and baseline ekg

What are some benefits post op PCA vs regional techniques

Less opioid use resulting in less side effects such as respiratory complications like chest wall rigidity.

Which coronary artery gives off the posterior descending artery

Like people, most hearts are right dominant 50% of people are right dominant 30% are codominant 20% of left dominant

What would a flow volume look for a laryngeal papilloma or upper airway mass?

Likely diminished limb on inspiration because it is an extrathoracic mass. If it is a fixed mass you may see both diminished inspiratory and expiration limbs.

EKG Changes hypocalcemia

Long QT, short QRS

How do you know if the EKG changes in an SAH patient are cardiac related or not?

Look at prior EKG and see if the changes are new or not. If they are then do a history and physical that pertains to chest pain, but either way I wouldn't delay the case

What's a clear sign on echo of cardiac tamponade aside from blood in pericardium

Look at the septum and see if it bulges left during inspiration and bulges right during exhalation. Normally when the ventricles fill the pericardium expands instead of septal bounce

How do you diagnose a venous air embolism

Loss of ETCO2, hypotension, most sensitive way is a TEE probe, increased etN2, decreased sat, clinical suspicion

What the positive and negatives about dopamine

Low dose activiates Dopamine receptors .5-3ug/kg/min Middle : beta receptors High: alpha 7.5mcg/kg/min Negative: arrhythmogenic and can cause tachycardia. So don't use with a fib. Positive: low dose: can increase blood flow to mesenteric, cerebral, and kidneys Middle dose: some ionotropic activity High dose: known for vasoconstricting while maintaining cardiac output. Second line therapy in septic shock and useful for cardiogenic shock with dobutamine

Would you intubate a child for laryngeal papilloma resection?

No, I would keep the child ventilating while the surgeon works if the child is a simple airway. If the airway was difficult then a laser safe ET tube would be used. Some of the problems with intubation include airway bleeding, fire, obstruction of surgical field, airway resistance-due to smaller diameter ET tube, and seeding of papillomas into the distal airways.

A patient who is about to have a cabg is on chronic heparin gtt for MI, would you stop the heparin preoperatively?

No, I would place my lines quickly and use an ultrasound probe vs the risk of stopping the heparin and having an MI

You're going to place a PA catheter but the patient is unable to tolerate lying flat and gets short of breath, do you place it with the head up?

No, because they're already breathing heavy and now with the head up they're at a much higher risk of VAE

After starting an epidural with bupi can you bolus with chloroprocaine?

No, it reduces bupi analgesia

What are the only indications for FFP? If you give 2 units of pRBCs should you give two units of FFP?

No, just because you give 1-2 units of blood doesn't mean you should match it. Indications massive transfusion protocol to avoid a dilutional coagulopathy replacement of a factor deficiency reversing warfarin heparin resistance or AT3 deficiency

If you have a down syndrome patient adult, will you need a c spine xray?

No, the incidence of atlanto axial instability is 10-15% and this can cause cord compression. Ask the kid to range their neck and if they get light headed they have vertebral artery compression. clonus, parasthesias the cervical xray will be based off of history and exam

The patient is in the pacu with a glucose of 209, would you treat it

No, we only treat over 250 as this is the level where the patient can have enuresis.

What is a normal Aortic valve area and gradient. What is aortic stenosis area and gradient

Normal 2.5-6 with a gradient less than 15. Severe is <1cm2 and gradient greater than 40 cm/s

What are the side effects of mag treatment

Normal is 1.5-3 For treatment you want 3.5-7 4-5 Diminished deep tendon reflexes 4-7 EKG changes prolonged PR and ST intervals, widened QRS 5-7 Somnolence 8-10 loss of DTRs 15-respiratory arrest 20-heart block

What is your fluid of choice with transfusion?

Normal saline, LR can cause blood to coagulate due to the calcium binding with citrate

What is a normal aortic valve area/velocity/gradient

Normal valve area is 3-4cm2, velocity 1.5m/s

A patient is hyperkalemic, which maintenance fluids would you choose?

Not LR as it has some K in it

What are your thoughts on performing regional anesthesia on a patient with chronic kidney disease?

Not an option because these pts are likely coagulopathic due to decreased VWF

A patient with SAH has t wave flattening and U waves, are you concerned?

Not if she does not complain of chest pain and there's no current arrhythmias. These are likely SAH related

Would you perform a regional anesthetic on a CHF patient?

Not if the block requires them to lie flat and they are unable to do so

In a mitral stenosis patient, is an epidural for pain a great idea?

Not in severe mitral stenosis, I'd be worried about dropping their preload due to sympathectomy, and if they're having pulmonary congestion, they may have an increase issue with breathing if I take away their thoracic muscle tone if it's a thoracic epidural

For a patient in the ICU with aortic stenosis what would you use for sedation

Not propofol, it can cause life threatening hypotension. Use an infusion of midazolam and fentanyl.

What are your thoughts about placing a TEE probe in a full stomach?

Not the best idea due to you're constantly keeping the GE junction open. The food would obscure the image as well.

How do you diagnose RBBB

Notched R wave in V1-3 Deep S V6 (M) Wide QRS Absent q waves

Treatment of TRALI

Notify blood bank, stop infusion, and treat supportively

In a preggo what are the first 3 things you do with a decreasing sat or blood pressure

O2 cannula left uterine displacement fluid/pressors

What is the only time its ok to do regional due to a difficult airway

OB as we want mother to baby contact

How is OSA different from Obstruction Sleep Hypopnea

OSA complete cessation of airflow for greater 10 seconds, 5 times per hour. OSH is decrease in airflow greater than 50% for more than 10 seconds, 15x more per hour. They both have a 4% decrease in O2 sat.

How is OSA distinguished from Pickwickian Syndrome

OSA is complete stop of airway, while Pickwickian is chronic hypoventilation that worsens during sleep. This results in elevated PaCO2 levels.

Why do obese or OSA patients have post op apnea?

OSA makes one have hypersomnolence due to interrupted sleep patterns and they actually have a decreased MAC requirement.

What are some risk factors for post op hoarseness

Obesity duration difficult intubations old age and case length Ways to prevent include low cuff pressures, small ET tube size

What are some intraoperative risks for the asthmatic patient

Obstructive lung disease chronic hypoxemia -> polycythemia bronchospasm arrhythmias cor pulmonale post op intubation If there's active wheezing order ABG

How do you diagnose TRALI, what are the symptoms

Occurs in less than a 6 hour window fever hypoxia hypotension diuretics and steroids have not shown to be helpful

How long do you monitor an OSA patient in the PACU

On continuous pulse ox and able to maintain sat on RA. If the patient has an apneic episode then we admit with continous pulse ox

Will you use paralytic in a TEF repair case?

Once I have confirmed the tube is in the correct position with auscultation then I can paralyze.

What concerns do you have about placing a TEE in a cirrhotic

TEE may cause rupture and bleeding of esophageal varices. If extreme caution is used during insertion and minimal manipulation the likelihood of rupture is extremely low. It also can guide therapy intraoperatively.

If you're doing a case with a valvulopathy what is a great tool to have in place?

TEE probe

With neuro procedures, what is one monitor that you should always place?

TEE, to pick up VAE

What is the maintenance of choice when doing non-obstetric surgery on a paturient

TIVA

While doing a TURP a patient becomes disoriented and restless, whats your ddx and how would you proceed

TURP, MI, bladder perf, hyperglycemia, hyponatremia, hypoxia, pulmonary edema. Alert the the surgeon to stop irrigating. Intubate the patient as the disorientation makes them unable to protect the airway. Then ABG for serum sodium and glucose if glycine is being used, then a high ammonia level is also on the differential

Names a couple of surgeries you can get an air embolism

TURP, crani, spine, chest trauma, laparoscopic

What are the EKG changes with hyperkalemia

Tall waves, wide QRS, prolong PR, flat p waves

How do you diagnose myasthenia gravis

Tensilon test or edrophonium test. It is a reversible achesterase inhibitor. These patients also need a full cardiac work up as myasthenia can cause arrhythmias, myopathies and disease cardiac muscle

What is the definition of base excess

The amount of bicarb required to take 1 L of whole bloo back to a pH of 7.4

In a cervical laminectomy case, would you put a central line in?

The answer is yes! because whenever you get a cervical laminectomy case you may have a VAE

Where does reglan and all the anti-emetics work

The area postrema in 4th ventricle of the brainstem

In a cervical laminectomy case, where are the arms and what are the position of the hands

The arms are tucked at the sides with the palms facing the sides

How do you know if a bladder cuff is the right size?

The bladder length should cover 80% of the circumference of the arm

What effects happen to the body when placed in a lateral decubitis position

The bowel content of the dependent side is pushed in to the thorax decreasing FRC, cardiac output is disproportionate to 65 to 35% and this creates dead space ventilation in the non-dependent lung Once the chest is opened on the operative lung it is no longer constrained by the chest wall and can receive more ventilation even worsening V/Q mismatch even more so. Opening of the chest more shift the mediastinum toward the dependent lung creating even more deadspace ventilation Just being in the lateral position creates a redistribution of ventilation toward the non-dependent lung. This is worsened by an open chest and the hemidiaphragm in the dependent lung

What are your thoughts on a BP of 196/116 in a closed head injury patient

The bp is secondary to an increased ICP. The key is lowering the ICP, 1. Make sure the pt is oxygenating and ventilating appropriately and the airway is not obstructed 2. Elevate the patient's head 30 degrees 3. Have neurosurgeon drain some CSF 4. Mannitol and lasix 5. Mechanical ventilation and hyperventilate

Why do a spinal at only L5?

The cord ends at L1, so going above this could cause major cord injury if doing a spinal

How does the presence of an AICD effect your anesthetic management

The danger is when cautery is taking place 6 inches from the device. Place magnet on the device but when you do this you need to place pads on the patient too. -Ask surgeon to use short intermittent bursts -Use bipolar cautery

How do you manage post op pain with a sickle cell patient

The difficulty is that both pain and acidosis cause sickle cell crises. 1. Check HHM (hypoxic, hypercarbic, malignant rhythm) 2. Focuses H&P. Physical should include surgical site for bleeding. 3. Proceed with non-opioid anaglesics and then to po narcs, and then fentanyl if need be.

What is the etiology of TRALI

The donor Igs came along for the ride

With a sickle cell patient what is your stance on exchange transfusions?

The evidence on them is loose when compared to a regular transfusion in terms of perioperative mortality. I do exchange transfusion when the hemoglobin is low and the patient is unstable or showing signs of end organ dysfunction because at that point the benefit outweighs the risk. For example if a patient has a hgb of 7 I wouldn't transfuse but if they have a 7 with hypotension and tachycardic then I would. The recommended target would be a a Hgb of 10. But note that anemia is a trigger for crises.

Why use a pressure control setting with a restrictive lung picture

The goal is to recruit alveoli that have been collapsed by the restrictive lungs, maintaining FRC, improving oxygenation, while avoiding barotrauma Pressure control has shown to reduce peak pressures, improve oxygenation, improve gas exchange

From adult to pediatric with burns what changes are there with body surface area

The head becomes 18% instead of 9% a leg goes from 18% to 14%

A patient has rales or prominent vascular markings at baseline. They then become hypertensive and now have an increased peak pressure and increased rales on auscultation, they now are becoming hypotensive...what's happening

The hypertension unmasked a poor LV and they have increased pulmonary edema. Treat and be decrease afterload and decrease preload. Nitrogylcerine is a great choice for this. Also can increase peep to drop preload. You can also hyperventilate the patient to drop the bp. CVVH to take off volume as a last resort

What are some factors that you have to consider when preoperatively transfusing a sickle cell pt

The idea to transfuse is individual to the practitioner. My factors include 1. Pt comorbidities 2. Starting Hgb 3. The actually surgery planned, does it have a large EBL?

If a LIMA to the LAD is performed what must be opened?

The left pleural space and chest tube will usually go in there.

What happens immediately after you ligate the PDA in this surgery to the patient

They can become hypertensive as there once was a left to right shift but no longer. This then increases afterload and elevates the cerebral perfusion pressure. If is not transient you have to give a vasodilator at times. Sometimes this can unmask LV dysfunction

Why not use 100% oxygen in neonatal resuscitation

They can get hyperoxic injury to their optic nerves

Why are RA patients a difficult intubation

They can have atlantoaxial instability stiffened, cricothyroid, TMJ, costochrondral joints restrictive lung pattern due to interstitial fibrosis GI: ulcers due to RA therapy Renal: same

What happens you have a patient who is hyponatremic and you begin repleting their sodium too fast

They can incur central pontine myelinolysis a severe demyelinating condition of the brainstem

What does a flow volume loop look like for a thymoma?

They can tell you if it's extrathoracic or intrathoracic. It can also tell you if it's fixed or dynamic.

Why can't you use glucose containing solutions in patients with a neurologic injury?

They cause hyperglycemia, diuresis, and cerebral acidosis. Even with D5W the glucose is typically metabolized leaving the fluid hypotonic causing cerebral edema.

What don't we take NSTEMI to the cath lab?

They due to blockages severe ST depression but their usually not

What happens to PA pressure and CVP with a PE and tamponade

They elevate

What are the recommendations for continuing beta blockers in the perioperative setting

They have shown that pts who undergo intermediate risk or vascular surgery with a hx of CAD, DM, cerebrovascular disease or CHF will benefit from them

Why can a diabetic be a difficult airway

They may have increased atlanto occipital joint stiffness

What substances are released by a carcinoid tumor

They release serotonin, histamine, catecholamines, bradykinin, prostaglandins, vasointestinal peptide. catecholamines, serotonin, histamine

What extra monitor do you need for a HOCM patient and what kind of rhythms are they prone to

They're prone to junctional rhythms, so have pacer pads on them. They're heavy reliant on atrial kick which a junctional rhythm will take away also avoid epi in blocks

When pre op evaluation a TE fistula what labs would you want to order?

Think VACTERL Order cxr, abg, echo, renal ultz, spine films, electrolytes,

What are the symptoms of a thoracic aortic aneurysm

Think compression hoarseness due to compression of the recurrent laryngeal nerve stridor due to tracheal compression dysphagia due to esophageal SVC syndrome

Uvulopalatopharyngoplasty precautions

This is a bloody and very vascular surgery that requires art line and the surgeon may ask for permissive hypotension Nasal intubation Planned extubation because usually the jaw will be wired shut surgeon bedside, awake patient wire cutters present blood gas minimal opioid use or short acting patient sitting up and following commands

How do you make the diagnosis of a PDA

This is a left to right shunt bounding pulses, widened pulse pressure, CHF symptoms with intercostal retractions, decreased breath sounds, rales, S3 heart sound. Increasing respiratory failure with decreases in PaO2, increasing PaCo2, and then can be confirmed with echo Overall a CHF like picture

What is your DIC treatment

This is a life threatening emergency fluids O2 blood products through rapid infuser and invasive monitors

What does the artery of adamkiewicz do

This is a tributory around T11 from the aorta to the anterior spinal artery

Is propofol a good choice for this patient?

This is an induction question

After unclamping of a new kidney in a transplant the patient develops hyperkalemia, why?

This is hyperkalemia from preservative fluid of the new kidney. Thus give calcium and treat BIGCK. So after reperfusion you want to monitor your EKG closely.

During laryngeal papilloma resection the surgeon tells you the cords are closed, how do you proceed

This is larygospasm 1. Positive mask ventilation with 100% O2 2. Ask surgeon to provide aerolized lidocaine onto vocal cords 3. Deepen anesthetic with proprofol 4. Succinylcholine .25mg/kg

After closing the skin in a congenital diaphragmatic hernia the BP begins to drop, what will you do

This is likely due to IVC compression after reduction fo the hernia back into the abdominal cavity, we will open the abdominal cavity and due a temporary closure with a patch

While doing an anterior mediastinum tumor resection, you secure the airway and then sudden lose etCO2, How do you respond

This is most likely mass compression of the trachea by the mass. Therefore, we would take the rigid bronchoscope and attempt to place it beyond the level of mass. Move the patient into the prone position or lateral to relief the compression. If this didn't work we then open the chest and relieve the compression or begin bypass

What is significant about a T6 level with an epidural?

This is the level where you start to have some affect on the thoracic musculature

If a stem provides you with someone who has chronic back pain what does this trigger for you

This means it's likely they have a high opioid tolerance and you should start thinking about regional pain control

What is hemolytic disease of the newborn

This occurs when IgG antibodies from the mother pass the placenta. The presenting fetus symptoms are either mild to heart failure death (hydrops). This should be on your ddx when a baby is delivered with jaundice

On intubation you fail and you place and LMA which is now failing, what are your next steps

This would be the non-emergency pathway. Options include a ventilation with a rigid bronchoscope, ventilation with a combitube, and transtracheal jet ventilation

How do you diagnose tamponade in OR

Xray, echo, ekg xray-enlarged cardiac silhoutte, widened mediastinum, the heart will have a water bottle appearance ekg will appear electrical alternans, low voltage and high voltage and overall low voltage You'll also see equalization of the wedge pressure and the CVP Echo collapse of the right atrium

Would you place a TEE probe in a pt who is asymptomatic with a cardiac hx in a carotid endarterectomy case?

YES! the major risk of this surgery is a myocardial infarction

Would you exchange a double lumen tube in the ICU? why or why not?

Yes 1. Easier to suction through. 2. ICU team is more familiar 3. Lower airway pressures with the same tidal volume

For a large vascular bypass case would you place an epidural for post op pain?

Yes I would post op pain and could use it to offset when a cross clamp is placed.

Would you place a central line

Yes and here's why Fluid and blood resuscitation Multiple infusions or pressors evaluating fluid status treating VAE

SAH or aneurysm rupture...can you have an lung or heart problems

Yes arrhythmias, long QT and neurogenic pulmonary edema

If the patient has a first degree relative of malignant hyperthermia, can you perform their surgery in an outpatient center?

Yes as long as there's an MH cart available and all necessary precautions are in place

Would you want an echo for every liver transplant?

Yes because I would want to know if the patient has pulmonary hypertension as it is a contraindication >50mmHg to a transplant

You reverse your paralytic and now the surgeon wants to go back in, would you reparalyze

Yes but notify that we will have unpredictable reversibility

Would you use a nerve stimulator in myasthenia patients in the OR

Yes but remember that different muscles may give you different results.

Would you use a twitch monitor in a succinylcholine patient?

Yes even though it's unreliable I would still trend it.

Would you drain the ascites before OR

Yes if new sob, but you will drop your preload

Can you do a fiberoptic through an intubating LMA?

Yes there's room

Should a pt on oral benzos take it before the OR why?

Yes they should take it so we don't precipitate a withdrawal syndrome

Before starting an abdominal/thoracic aneurysm repair the blood pressure is elevated, will you lower it?

Yes while being cognizant of keeping it within 20%

In the pacu after a carotid the patient has a BP of 210/120, will you lower this?

Yes with nitroglycerine or something short acting and titratable.

Pt coming to OR for fem nail, pt has a trach and g tube feeds were left on. Can you do the case?

Yes you can, if you insert a g tube, give bicitra, give reglan, check ET balloon. You can do it because the surgery is away from the airway. If it was like a trach revision then you can't do it

When adminstering mannitol or lasix, what is your treatment parameter...can you give 10000mg?

You administer as the BP tolerates

What is the danger of mediastinoscopy

You can compress the right brachiocephalic artery

With a neurofibromatosis patient, why would a cxr be a good idea

You can have an intracardiac neurofibroma, RVH, pulmonary markings, increased cardiac silhoutte these pts are at risk of arrhythmias

If you place a interscalene block for shoulder surgery and it works just a little, how can you supplement this?

You can place a superficial cervical block

How does ATN or kidney failure modify your anesthesia evaluation?

You cannot use urine output as a tool for volume status

How do you assess for atlanto-axial instability on exam?

You examine for cord compression -clonus, hyperreflexia, parasthesias, numbness. If you have this you should delay case and neurosurgery consult

If given a blood gas and asked would you extubate this patient, what would be your next step

You have to look at the entire patient. SOAR + blood gas. Some patients can be extubated with a Pao2/FIO2 of 200 if they're SOARING

Coming off bypass you have trouble inflating the lungs, what's your ddx

You would go down your regular routine steps of listen to the patient feel the chest, suction the ET tube etc

In a kidney transplant case what are your drugs of choice with induction

You'll have to do an RSI but you have to decide if you'll use succinylcholine because the patient is likely to have an increased potassium. However, if you use rocuronium RSI then you might have a hypotonic patient at the end when you reverse....ie use suggammandex Also mention to avoid drugs that are renally excreted like morphine.

What size LMA would you place for a 2 year old

a 2

What size LMA would you place for a child who is 5-10kg

a 2

What device do you need to ind carboxyhemoglobin

a co-oximeter is needed to see the true O2 sat.

What is eliquis and how could you reverse it?

a direct factor ten inhibitor and you could use K centra if it is an emergency

What is the most likely cause of a diastolic murmur with a pt with aortic dissection

a dissection that extended into regurg

What is pulsus paradoxus and why do you get it

a drop of systolic blood pressure more than the normal 10 during inspiration

What conditions would you want to keep a PDA open

a ductal dependent condition like hypoplastic left heart syndrome, aortic valve stenosis, interrupted arch reopen with prostanglandin E1 So if you give indomethacin and the baby turns blue, then reopen it

Tell me your ddx for chronic anemia and how would you evaluate

anemia of chronic disease GERD! can lead to iron deficient anemia nutritional deficiencies chronic blood loss Evaluation: Reticulocyte count is never elevated in acute blood loss as it takes 2-3 days to make reticulocytes Low retic count is production vitamin deficiencies, malignancy, drugs, toxins, aplastic anemia If high reticulocyte count then anemia due to chronic blood loss of hemolysis If normal usually due to acute blood loss 1. Follow serial cbcs 2. Send MCV, normoctyic is usually acute bleeding or ACDZ 3. Screening labs for DIC 4. FAST scan, cxr for aortic rupture 5. EGD, sigmoid

Name all the factor that could change SSEPs

anemia, O2, CO2, temperature, blood pressure

What are the risk factors for post op vision loss in spine surgery

anemia, hypotension smoking, diabetes, htn, atherosclerosis the head down position thus increasing CVP ALSO consider transfusing at a much higher level

Why do ace inhibitors cause refractory hypotension

anesthesia blunts ANS and ace blunts RAAS

What is one thing you must do before sliding the tube off of your fiberoptic in an awake intubation, especially in someone with a cardiac issue or c spine injury

anesthetize the cords with lidocaine or induce the patient. As you do not want them to buck and hurt their neck or have a massive sympathetic discharge and have an MI

What are the sidefx of milrinone

angina, arrhythmias, low platelets, low K, low BP

Where is angiotensinogen made and where is ACE made

angiotensinogen is made by the liver and renin cleaves it to A1 ACE cleaves A1->A2 ACE is made in the endothelial cells of the lung and kidney

If a patient has an acidosis, how do you know if it's an acidosis of accumulation or failure to excrete

anion gap non gap is excrete gap is accumulation ketoacid lactic uremia toxins non gap renal tubular acidosis hyperalimentation diarrhea

When a patient has mitral stenosis, atrial fibrillation or stents what should you look out for

anti-coagulation. They will try to trick with this and regional techniques

What the indication for an ICP monitor

any TBI with a glascow less than 8

Indications for verapamil

arrhytmia, AV blocks, SVR

PAC complications

arterial puncture arrhythmia RBBB Complete heart block post operative neuropathy ptx air emboli infection thrombophlebitis pulmonary infarct endocarditis valve injury pulmonary artery rupture

What is the biggest pulmonary concern in a cirrhotic?

ascites creatng a restrictive pt

Pt neonate becomes hypotensive during repair, whats your ddx

aside from the normal stuff hemorrhage from tearing of the ductus, lung retraction, ptx

What are the two ways to fix blood pressure on bypass

ask the perfusionist to increase the flow rate or administer a vasoconstrictor

Something can cause cause increased peak pressure, rales, and wheezing

aspiration

What's your quick ddx for peak pressure

aspiration bronchospasm endobronchial intubation mucus plug ptx

How do you tell aspiration vs bronchospasm

aspiration is rales in the right lung, bronchospasm is in both

What your ddx for foreign body aspiration in kids

aspiration, croup, reactive airway, anaphylaxis, epiglottis, pneumonia, seizure, cardiac abnormality

What is a big risk with retropharyngeal abcess in terms of extubation

aspiration, that thing is slowly leaking puss

What are the contraindications to beta blockers

asthma, COPD, brady, AV block

How is a TEF diagnosed

at birth when the NGT is unableto be passed beyond 9-10 cm and there's increased drooling or when there's coughing, choking, or cyanosis with feeding, so made clinically.

what are your thoughts about using volatile anesthetics in a crani

at less than 1 mac your autoregulation is usually still preserved

Tell me about a down syndrome airway

atlanto axial instability macroglossia small mouth opening short neck adenoidal hypertrophy OSA

What is an S4 heart sound

atrial contraction-blood rushing in forcefully in diastole against a stiff ventricle (atrial gallop). S4 gallop is rarely normally like an S3. S4 is a sign of diastolic heart failure. Both S3 and S4 are heart best over the apex with the bell The S3 sound occurs with blood coming into a ventricle that is too compliant S3 can be normal in athletes, young people, pregnant women Otherwise it means heart failure or severe MR

In congenital diaphragmatic hernia what are you goals for ventilation

avoid barotrauma and oxygenate adequately. Do not exceed 25-30cmH2O, PEEP 5-7 O2 sat of 90-95% and permission hypercarbia with a pH greater than 7.3

Concerns about sickle cell patient post op

avoid cold avoid hypoxia hypercarbia avoid hypotension

What are your hemodynamic goals in a patient with tetralogy of fallot

avoid conditions that lower SVR and/or increase PVR. Increase in preload is a good idea as it opens the RVOT tract

Colleague stealing fentanyl, whats your management and what do you want to avoid

avoid direct one on one confrontation because after this they usually are at highest risk of suicide

How do you intubate a pt with a head injury

avoid gagging and bucking as this can worsen ICP. After prepping and draping the neck, trach kit open and ENT in room, perform an RSI with in line neck stabilization with etomidate and succinylcholine

Tips for pericardial effusion

avoid paralysis dx with echo

Whats the key to cardiac tamponade

avoid ppv as it is life threatening. If patient is very unstable then local and some ketamine. If patient becomes apneic then succ and intubate for a paracardiocentesis

What is the treatment for digoxin toxicity

draw a digoxin level release the patient now is likely to have hyperkalemia but don't treat with usual hyperkalemia as digibind will redue the k level DO NOT GIVE CALCIUM, as the patient already has intracellular calcium and its high and can worsen the situation If you know the amount of digoxin ingested give two vials of digibind for each mg

If you have a non-cardiac pt, with chest pain and its emergent what do you do?

drop a TEE

How would you verify that you're in the epidural space

drop column method like with a central line

What is your major concern about an ESRD dialyzed yday

drop of BP with induction

difference between drug eluting and bare metal

drug eluting are less likely to restenosis but longer period of platelet therapy

Predictors of intraop renal failure

duration of cpb Use of balloon pump blood transfusions need for pressors

How would evaluate the effect or status of a mediastinal mass on a pt?

dysphagia, SVC symptoms, deviations of oropharyn, facial cyanosis, venous distention in the neck or arm, edema of upper extremity. Imaging location and size of mass and degree of tracheal deviation

The three monitors that make you savvy

echo probe, foley, twitch

What is the dose of decadron for airway edema in peds and for ponv

edema 0.5mg/kg with a max of 8, ponv .15mg/kg

Ddx high peak and plateau pressure

edema, pneumonia, endobronchial intubation, ptx

What is refeeding syndrome

electrolyte imbalance seen when a pt gets TPN after starvation This leads to hypophosphatemia, hypokalemia, hypomagnesemia, glucose imbalance. Low K, mag, phosph

Causes of atrial flutter

electrolytes, medications, CAD, thyroid, PE

During a carotid the surgeon cross clamps and eeg goes down, and you ask to place a shunt, why not just do this in all pts

emboli risk

Ddx for increased peak pressure

endobronchial intubation, aspiration, ptx, PE, bronchospasm

What are the changes in airway with a acromegalic pt

enlarged epiglottis subglottic stenosis narrow opening making them a difficult intubation and ventilation diabetes

With sinus bradycardia what do you do if atropine doesn't work and you're waiting for a pacer

epi or dopamine gtt

After a thoracic surgery the patient has b/l lower extremity weakness, what's the likely cause

epidural hematoma can be on the ddx.

What are some factors that differ epiglottis from croup

epiglottis 3-6 y/o, thumb print sign, bacteria, antibiotics croup 2-5 y/o, steeple sign, virus

How would you evaluate the cardiac status of a pt with HOCM

episodes of chest pain, light head, sob, baseline exercise status mets, physical exam would be heart sounds, CHF-pedal edema, JVD, pulmonary edema. Then imaging EKG, echo for pulmonary hypertension.

What is your acute treatment for stable atrial fibrillation

esmolol gtt, 50-200mcg/kg/min

Neuro case, what is your induction agent

etomidate

Which anesthetic should you avoid if the patient has a h/o seizure?

etomidate as it causes seizures and myoclonus

What drug will you induce a burn pt with?

etomidate, have to remember that they are very hypovolemic and so you don't want to drop their SVR. Also, CO is a cardiac depressant. You also would want a pre-induction arterial line.

What do you use PFTs for ?

evaluate signs and symptoms of chronic cough, respiratory difficulties exertional dyspnea, follow response to therapy, determine treatment goals, assess severity

With a PA catheter you can use to evaluate several things, but how do you do this?

evaluate the hemodynamic response to ionotropes mixed venous O2 samples Cardiac index and cardiac output

What is decerebrate posturing

extension due to pain

Do you extubate a stroke pt or a mass pt

extubate a mass pt but not a stroke pt

What does it mean to fast track a cabg

extubating a CABG in the first 1-6 hours post operatively. This is usually younger patients with a single jump

Down sides to a wake up test

extubation, pulled lines, recall

What is Chvostek sign

facial nerve excitability

Tell me about gastrochisis and the anesthetic concerns

failure of omphalomesenteric artery no membrane -> more dehydration, temperature loss, and infection risk Put a pulse ox on the foot to eval for venous congestion during the surgery. The main concern is returning the bowel content leading to decreased pulmonary compliance.

BP drops after induction to 60/30, next step

feel a pulse

Whats some good post op techniques to use in a neonate

fentanyl infusion 1mcg/kg/hr

Issues with preggo laparoscopy

fetal acidosis from CO2 absorb the pressure can decrease placental perfusion so run the insufflation low, art line and good map

Risk of paracervical block

fetal brady and uteroplacental insufficiency

What are signs of uterine rupture

fetal brady, abd pain, vaginal bleeding, hypotension changes in uterine pattern of contraction, cessation of labor.

Whats another good tool for fetal heart monitoring

fetal scalp pH

What are the symptoms of neuroleptic malignant syndrome

fever encephalopathy unstable vital signs rigidity autonomic instability rhabdo, renal failure, acidosis treatment dantrolene 2.5mg/kg

What is EMLA cream and what are the contraindications to it?

mixture of lidocaine and prilocaine takes 1 hour to kick in contraindications hypersensitivity to LA amides patients already being treated with anti-arrhythmics pts with congenital or acquired methgb Need a co-oximeter to see methgb

What is the definition of post partem hemorrhage

more than 500 ml vaginal and 1000 c section

RFs for uterine atony

multip, polyhydramnios, prolonged use of oxytocin, twins, macrosomia

What are the side effects of lithium

muscle weakness, slurred speech, ataxia, arrhythmogenic, potentiates NMDB and succinylcholine diabetes insipidus!

When there's a mediastinal mass, what do you automatically think of

myasthenia gravis and SVC syndrome

Prolonged block after defasciculation dose ddx

myasthenia, lambert eaton, myotonia, neuromuscular dz tx trend TOF at different muscles, warm patient, manage in the ICU

What is a immediate worry right when you place an aortic cross clamp

myocardial ischemia due to sudden increase in SVR The higher the clamp is placed the more pronounced the SVR will be If placed below the celiac artery blood will be shunted into the splanchnic circulation If placed infrarenally we may not notice at all This also increases cerebral blood flow and may increase ICP and decrease perfusion to the spinal cord

whats the biggest worry with a hypothyroid patient

myxedema coma decreased DTRs Decreased heart contractility Decreased volume decreased breathing/hypoventilation electrolytes hypothermia

Digoxin toxicity

n/v/d arrhythmias vision changes/halos

Will you place a PAC in a liver case?

nahh just TEE and let that and TEG guide my therapy

causes of narrow pulse pressure and wide pulse pressure

narrow (decreased pumping): aortic stenosis, bleeding, hypovolemia, heart failure, tamponade wide: increased volume or high output, aortic regurg, exercise, dissection, pregnancy, anemia, fistulas normal is 30-40 mmhg

What are the differences from an adult to a pediatric airway

narrow nasal passages large tongue, floppy epiglottis, funnel shaped larynx, glottis is at C4 instead of C6. -The narrowest part of the airway is BELOW the vocal cords where the narrowest part in an adult is the glottis

What kind of intubation for a peritonsillar abscess

nasal awake, opposite side. Review ct imaging for which side.

Disadvantages of ECMO

need for anticoag, increased bleeding potential at surgical sites, intracranaial hemorrhage, sepsis

What are the positives and negative epinephrine

negative increases ketoacid lactic acid without ischemia lipolysis hypermetabolism by 35% which can become an issue with poor tissue oxygenation arrhythmogenic severe hypertension on those on beta blockers due to unopposed alpha

What effects does digoxin have on the heart

negative chronotropy, positive ionotropy.

For a pyloric stenosis patient why might it not be a great idea to use lactated ringers?

neonates are unable to metabolize lactate as of yet. Use NS until urination is normal and then switch to 1/2NSD5 and supplement it with potassium and dextrose

Surgeries that might benefit for a thoracic epidural

nephrectomy, laparotomy, thoracotomy, radical prostatectomy, thoracic and abdominal AAA, hepatic resection, pancreatectomy, bowel resection, esophagectomy, gastrectomy

What are your thoughts on neuraxial anesthesia in HIV patients

neuraxial decreases the chance of vertical transmission Introducing the virus into the CNS via epidural/spinal or blood patch is not of particular concern as the virus penetrates the CNS early in the infection course.

After a crani, the patient has pink frothy sputum in the ICU, what is it

neurogenic pulmonary edema

How do you place an IV in an MH kid?

nitrous

Can you give toradol to an asthma patient

no

Is a thyroidectomy ever an emergency surgery?

no

Is jaundice due to inhalational agents

no

The patient has a hx of stroke with right arm weakness, can you apply a BP cuff there?

no

Would you extubate an epiglottis kid?

no wait to days of antibiotics and no fever then perform a nasal fiberoptic and eval the airway

Can you be DNI and have an order that compressions is ok?

no Intubation is part of CPR

If your pheo patients has not been on alpha blockade before OR can you use a beta blocker

no but you can use phentolamine IV

What are the positives and negatives to vasopressin

no cardiac effects vasodilates the cerebral vasculature Increase SVR, decreases PVR

Benefits of regional

no instrumenting airway lower opioid side effects lower risk of DVT avoid muscle relaxants

how to clear a C spine, list 3 criteria

no report of pain or tenderness, MRI, range the neck

MH vs thyroid dz

no rigidity, no CK, no increased K

why is lovenox better than heparin

no risk of HIT and longer duration

Will you place a PAC in this liver pt?

no, TEE Then how will you know if they have phtn? I'll get this information from a previous cardiac cath

Would you give dig in the OR

no, easily toxic, very narrow therapeutic window

Is PEEP good in neuro anesthesia?

no, impairs venous return from the brain....so avoid it

What are the symptoms of malignant hyperthermia?

tachypnea tachycardia rigidity acidosis rhabdomyolysis hyperthermia Intraop signs Increased CO2 that does not respond to ventilation masseter muscle rigidity fever arrhythmia Muscle contraction -> rhabdo and hyperkalemia Mixed metabolic and respiratory acidosis

Name the 5 ASA monitors

temp, capno, blood pressure, pulse ox, 5 lead ECG

two indicators of a working stellate ganglion block

temperature, horners

In a trauma what do you think of?

tension ptx

What kind of masses can you get in the anterior mediastinum

teratoma, thymoma, thyroid, lymphoma

Name some teratogens that you wouldn't want to use during a pregnant anesthetic

tetracycline, streptomycin, carbamezepine

For IE Prophylaxis, name some cyanotic heart diseases

tetraology, transposition, truncus arteriosus

In terms of PFTS what can DLCO tell you

that increasing FIO2 wont increase oxygenation

What to convey during preclampsia epidural

that the patient is volume depleted, so have fluids and pressors ready. Also, trend the platelets, don't go off one time lab

You're bolusing the patient's IV and you get shocked, what do you do

that's a macroshock, EKG and check a pulse. A macroshock can induce V fib. Macro shock is 100mA and microshock is 10microA. Line isolation monitor only detects macro.

Abdominal pain in a sickle cell patient

that's splenic sequestration and you need blood transfusion

What extra step do you need to do an off pump cabg

the bypass machine primed and ready

How do you determine where the epidural should go with rib fractures?

the location of the middle fractured rib

What is an S3 heart sound

the sound of blood connecting distended ventricle, indicative of diastolic CHF

What is the K value, Whats the alpha angle

the time it takes until the clot reaches a fixed strength alpha is the speed of fibrin accumulation

Removal of aortic cross clamp, how do you prep

there will be an acute drop in blood pressure which may lead to hyperkalemia, acidosis, and can lower the core temperature. If there's too much of an acute drop, have the surgical team to drop the clamp incrementally

Whats the difference from a whitacre or sprotte needel to a quincke

these are pencil points

Why are most mitral stenosis pts tachy

they have pulmonary edema and can't breathe and panic ->catecholamine surge

Trauma in a preggo ddx

think uterine rupture, placental rupture

What is the treatment for cyanide toxicity

thiosulfate, inhaled amyl nitrate, hydroxocobalamin You want to create met hemoglobin because the kidneys will create eliminate cyanomethemoglobin

Whats ACLS algorithm sinus bradycardia

this is a heart rate of less than 60 and symptomatic ie low blood pressure atropine 0.5mg q 3-5 min up to 3mg If ineffective start pacing Consider dopamine infusion 2-20 mcg/kg/min Epi 2-10 mcg/min if no pulse go into PEA algorithm

Is a congenital diaphragmatic hernia emergency?

this is another that must be managed medically prior to coming to the OR. Stabilizing cardiorespiratory function by improving oxygenation to have a preductal oxygen saturation greater than 90%, correcting acidosis, reduction of R->L shunt, and increasing pulmonary perfusion using the lease aggressive ventilation possible.

What is your steroid dosing and who shouldn't get steroids

those on less than 3 weeks at any dose range Those on less than 10 mg daily Whats your hydrocortisone rescue dose for refrac hypotension 1.5mg/kg All patients get their normal daily dose in am and then extra dose before incision minor surgery: 25mg taper that night moderate surgery: 50mg with a 2 day taper major: 150mg with 3 day taper

Post op cabg is having a drain output of 300 ml, whats the ddx

thrombocytopenia, hypothermia, DIC, inadequate heparin reversal. Order an ACT, coags, fibrinogen why not just give protamine? all the possible reactions

Difference between anaphylactoid and anaphylaxis

toid is dose dependent red man syndrome is anaphylactoid

How do you place an intraosseous line?

topical antiseptic, insert interosseous needle into the proximal tibia two finger breadths from the tibial tuberosity and advance in a corkscrew like fashion until a loss of resistance is obtained, then connect infusion line and check for extravasation.

During a neuro patient surgery the patient bradys to 40, what could this be

traction or pressure on the brainstem it could be indicative of a brain stem herniation

What TEE view is best for assessing MI

transgastric sax

What are the possible side effects of using glycine irrigating solution in a TURP

transient blindness, elevated ammonia levels since glycine is metabolized to ammonia -> altered mental status and even coma. Check a serum ammonia level

What is the down side to mannitol

transient increased circulating volume which may not be tolerated by patients with CHF. Then decreased potassium and sodium

List some cyanotic heart lesions

transposition of great vessels truncus arteriosus tetralogy total anomalous pulmonary return

You have a patient with platelets of 99 and Hgb of 9.8 how do you proceed for a simple case

tren with older labs to see if acute or chronic. If chronic type and cross and proceed if asymptomatic and make sure blood available. If acute i would have to further investigate

You give succinylcholine and the patient's jaw clenches down, what's your ddx

trismus, tmj, malignant hyperthermia, myotonic dystrophy. Biggest later worry is rhabdo


Ensembles d'études connexes

D075 Information Technology Management Essentials

View Set

Reproductive System Study Module

View Set

Federal Tax considerations for Life and Annuities

View Set

Rational Equations Assignment Answers

View Set

Chapter 27: Antitrust Law (quiz 97.5/100)

View Set

CH 19: Wrist and Hand Conditions

View Set